Site Loader

Π‘ΠΎΠ΄Π΅Ρ€ΠΆΠ°Π½ΠΈΠ΅

ΠŸΠΎΡΠ»Π΅Π΄ΠΎΠ²Π°Ρ‚Π΅Π»ΡŒΠ½ΠΎΠ΅ ΠΈ ΠΏΠ°Ρ€Π°Π»Π»Π΅Π»ΡŒΠ½ΠΎΠ΅ соСдинСниС кондСнсаторов

Для достиТСния Π½ΡƒΠΆΠ½ΠΎΠΉ Смкости ΠΈΠ»ΠΈ ΠΏΡ€ΠΈ напряТСнии, ΠΏΡ€Π΅Π²Ρ‹ΡˆΠ°ΡŽΡ‰Π΅ΠΌ номинальноС напряТСниС, кондСнсаторы, ΠΌΠΎΠ³ΡƒΡ‚ ΡΠΎΠ΅Π΄ΠΈΠ½ΡΡ‚ΡŒΡΡ ΠΏΠΎΡΠ»Π΅Π΄ΠΎΠ²Π°Ρ‚Π΅Π»ΡŒΠ½ΠΎ ΠΈΠ»ΠΈ ΠΏΠ°Ρ€Π°Π»Π»Π΅Π»ΡŒΠ½ΠΎ. Π›ΡŽΠ±ΠΎΠ΅ ΠΆΠ΅ слоТноС соСдинСниС состоит ΠΈΠ· Π½Π΅ΡΠΊΠΎΠ»ΡŒΠΊΠΈΡ… ΠΊΠΎΠΌΠ±ΠΈΠ½Π°Ρ†ΠΈΠΉ ΠΏΠΎΡΠ»Π΅Π΄ΠΎΠ²Π°Ρ‚Π΅Π»ΡŒΠ½ΠΎΠ³ΠΎ ΠΈ ΠΏΠ°Ρ€Π°Π»Π»Π΅Π»ΡŒΠ½ΠΎΠ³ΠΎ соСдинСний.

ΠŸΠΎΡΠ»Π΅Π΄ΠΎΠ²Π°Ρ‚Π΅Π»ΡŒΠ½ΠΎΠ΅ соСдинСниС кондСнсаторов

ΠŸΡ€ΠΈ ΠΏΠΎΡΠ»Π΅Π΄ΠΎΠ²Π°Ρ‚Π΅Π»ΡŒΠ½ΠΎΠΌ соСдинСнии, кондСнсаторы ΠΏΠΎΠ΄ΠΊΠ»ΡŽΡ‡Π΅Π½Ρ‹ Ρ‚Π°ΠΊΠΈΠΌ ΠΎΠ±Ρ€Π°Π·ΠΎΠΌ, Ρ‡Ρ‚ΠΎ Ρ‚ΠΎΠ»ΡŒΠΊΠΎ ΠΏΠ΅Ρ€Π²Ρ‹ΠΉ ΠΈ послСдний кондСнсатор ΠΏΠΎΠ΄ΠΊΠ»ΡŽΡ‡Π΅Π½Ρ‹ ΠΊ источнику Π­Π”Π‘/Ρ‚ΠΎΠΊΠ° ΠΎΠ΄Π½ΠΎΠΉ ΠΈΠ· своих пластин. Заряд ΠΎΠ΄ΠΈΠ½Π°ΠΊΠΎΠ² Π½Π° всСх пластинах, Π½ΠΎ внСшниС Π·Π°Ρ€ΡΠΆΠ°ΡŽΡ‚ΡΡ ΠΎΡ‚ источника, Π° Π²Π½ΡƒΡ‚Ρ€Π΅Π½Π½ΠΈΠ΅ ΠΎΠ±Ρ€Π°Π·ΡƒΡŽΡ‚ΡΡ Ρ‚ΠΎΠ»ΡŒΠΊΠΎ Π·Π° счСт раздСлСния зарядов Ρ€Π°Π½Π΅Π΅ Π½Π΅ΠΉΡ‚Ρ€Π°Π»ΠΈΠ·ΠΎΠ²Π°Π²ΡˆΠΈΡ… Π΄Ρ€ΡƒΠ³ Π΄Ρ€ΡƒΠ³Π°. ΠŸΡ€ΠΈ этом заряд кондСнсаторов Π² Π±Π°Ρ‚Π°Ρ€Π΅Π΅ мСньшС, Ρ‡Π΅ΠΌ, Ссли Π±Ρ‹ ΠΊΠ°ΠΆΠ΄Ρ‹ΠΉ кондСнсатор ΠΏΠΎΠ΄ΠΊΠ»ΡŽΡ‡Π°Π»ΡΡ Π±Ρ‹ ΠΎΡ‚Π΄Π΅Π»ΡŒΠ½ΠΎ. Π‘Π»Π΅Π΄ΠΎΠ²Π°Ρ‚Π΅Π»ΡŒΠ½ΠΎ, ΠΈ общая Π΅ΠΌΠΊΠΎΡΡ‚ΡŒ Π±Π°Ρ‚Π°Ρ€Π΅ΠΈ кондСнсаторов мСньшС.

НапряТСниС Π½Π° Π΄Π°Π½Π½ΠΎΠΌ участкС Ρ†Π΅ΠΏΠΈ соотносятся ΡΠ»Π΅Π΄ΡƒΡŽΡ‰ΠΈΠΌ ΠΎΠ±Ρ€Π°Π·ΠΎΠΌ:

Зная, Ρ‡Ρ‚ΠΎ напряТСниС кондСнсатора ΠΌΠΎΠΆΠ½ΠΎ ΠΏΡ€Π΅Π΄ΡΡ‚Π°Π²ΠΈΡ‚ΡŒ Ρ‡Π΅Ρ€Π΅Π· заряд ΠΈ Π΅ΠΌΠΊΠΎΡΡ‚ΡŒ, запишСм:

Π‘ΠΎΠΊΡ€Π°Ρ‚ΠΈΠ² Π²Ρ‹Ρ€Π°ΠΆΠ΅Π½ΠΈΠ΅ Π½Π° Q, ΠΏΠΎΠ»ΡƒΡ‡ΠΈΠΌ Π·Π½Π°ΠΊΠΎΠΌΡƒΡŽ Ρ„ΠΎΡ€ΠΌΡƒΠ»Ρƒ:

ΠžΡ‚ΠΊΡƒΠ΄Π° эквивалСнтная Π΅ΠΌΠΊΠΎΡΡ‚ΡŒ Π±Π°Ρ‚Π°Ρ€Π΅ΠΈ кондСнсаторов соСдинСнных ΠΏΠΎΡΠ»Π΅Π΄ΠΎΠ²Π°Ρ‚Π΅Π»ΡŒΠ½ΠΎ:

ΠŸΠ°Ρ€Π°Π»Π»Π΅Π»ΡŒΠ½ΠΎΠ΅ соСдинСниС кондСнсаторов

ΠŸΡ€ΠΈ ΠΏΠ°Ρ€Π°Π»Π»Π΅Π»ΡŒΠ½ΠΎΠΌ соСдинСнии кондСнсаторов напряТСниС Π½Π° ΠΎΠ±ΠΊΠ»Π°Π΄ΠΊΠ°Ρ… ΠΎΠ΄ΠΈΠ½Π°ΠΊΠΎΠ²ΠΎΠ΅, Π° заряды Ρ€Π°Π·Π½Ρ‹Π΅.

Π’Π΅Π»ΠΈΡ‡ΠΈΠ½Π° ΠΎΠ±Ρ‰Π΅Π³ΠΎ заряда ΠΏΠΎΠ»ΡƒΡ‡Π΅Π½Π½ΠΎΠ³ΠΎ кондСнсаторами, Ρ€Π°Π²Π½Π° суммС зарядов всСх ΠΏΠ°Ρ€Π°Π»Π»Π΅Π»ΡŒΠ½ΠΎ ΠΏΠΎΠ΄ΠΊΠ»ΡŽΡ‡Π΅Π½Π½Ρ‹Ρ… кондСнсаторов. Π’ случаС Π±Π°Ρ‚Π°Ρ€Π΅ΠΈ ΠΈΠ· Π΄Π²ΡƒΡ… кондСнсаторов:

Π’Π°ΠΊ ΠΊΠ°ΠΊ заряд кондСнсатора

А напряТСния Π½Π° ΠΊΠ°ΠΆΠ΄ΠΎΠΌ ΠΈΠ· кондСнсаторов Ρ€Π°Π²Π½Ρ‹, ΠΏΠΎΠ»ΡƒΡ‡Π°Π΅ΠΌ ΡΠ»Π΅Π΄ΡƒΡŽΡ‰Π΅Π΅ Π²Ρ‹Ρ€Π°ΠΆΠ΅Π½ΠΈΠ΅ для эквивалСнтной Смкости Π΄Π²ΡƒΡ… ΠΏΠ°Ρ€Π°Π»Π»Π΅Π»ΡŒΠ½ΠΎ соСдинСнных кондСнсаторов

ΠŸΡ€ΠΈΠΌΠ΅Ρ€ 1

Какова Ρ€Π΅Π·ΡƒΠ»ΡŒΡ‚ΠΈΡ€ΡƒΡŽΡ‰Π°Ρ Π΅ΠΌΠΊΠΎΡΡ‚ΡŒ 4 кондСнсаторов Π²ΠΊΠ»ΡŽΡ‡Π΅Π½Π½Ρ‹Ρ… ΠΏΠΎΡΠ»Π΅Π΄ΠΎΠ²Π°Ρ‚Π΅Π»ΡŒΠ½ΠΎ ΠΈ ΠΏΠ°Ρ€Π°Π»Π»Π΅Π»ΡŒΠ½ΠΎ, Ссли извСстно Ρ‡Ρ‚ΠΎ Π‘1 = 10 ΠΌΠΊΠ€, C2 = 2 ΠΌΠΊΠ€, C3 = 5 ΠΌΠΊΠ€, Π° C4 = 1 ΠΌΠΊΠ€?

ΠŸΡ€ΠΈ ΠΏΠΎΡΠ»Π΅Π΄ΠΎΠ²Π°Ρ‚Π΅Π»ΡŒΠ½ΠΎΠΌ соСдинСнии общая Π΅ΠΌΠΊΠΎΡΡ‚ΡŒ Ρ€Π°Π²Π½Π°:

ΠŸΡ€ΠΈ ΠΏΠ°Ρ€Π°Π»Π»Π΅Π»ΡŒΠ½ΠΎΠΌ соСдинСнии общая Π΅ΠΌΠΊΠΎΡΡ‚ΡŒ Ρ€Π°Π²Π½Π°:

ΠŸΡ€ΠΈΠΌΠ΅Ρ€ 2

ΠžΠΏΡ€Π΅Π΄Π΅Π»ΠΈΡ‚ΡŒ Ρ€Π΅Π·ΡƒΠ»ΡŒΡ‚ΠΈΡ€ΡƒΡŽΡ‰ΡƒΡŽ Π΅ΠΌΠΊΠΎΡΡ‚ΡŒ Π³Ρ€ΡƒΠΏΠΏΡ‹ кондСнсаторов ΠΏΠΎΠ΄ΠΊΠ»ΡŽΡ‡Π΅Π½Π½Ρ‹Ρ… ΠΏΠΎΡΠ»Π΅Π΄ΠΎΠ²Π°Ρ‚Π΅Π»ΡŒΠ½ΠΎ-ΠΏΠ°Ρ€Π°Π»Π»Π΅Π»ΡŒΠ½ΠΎ, Ссли извСстно, Ρ‡Ρ‚ΠΎ Π‘1 = 7 ΠΌΠΊΠ€, Π‘2 = 2 ΠΌΠΊΠ€, Π‘3 = 1 ΠΌΠΊΠ€.

Π‘Π½Π°Ρ‡Π°Π»Π° Π½Π°ΠΉΠ΄Π΅ΠΌ ΠΎΠ±Ρ‰ΡƒΡŽ Π΅ΠΌΠΊΠΎΡΡ‚ΡŒ ΠΏΠ°Ρ€Π°Π»Π»Π΅Π»ΡŒΠ½ΠΎΠ³ΠΎ участка Ρ†Π΅ΠΏΠΈ:

Π—Π°Ρ‚Π΅ΠΌ Π½Π°ΠΉΠ΄Π΅ΠΌ ΠΎΠ±Ρ‰ΡƒΡŽ Π΅ΠΌΠΊΠΎΡΡ‚ΡŒ для всСй Ρ†Π΅ΠΏΠΈ:

По сути, расчСт ΠΎΠ±Ρ‰Π΅ΠΉ Смкости кондСнсаторов схоТ с расчСтом ΠΎΠ±Ρ‰Π΅Π³ΠΎ сопротивлСния Ρ†Π΅ΠΏΠΈ Π² случаС с ΠΏΠΎΡΠ»Π΅Π΄ΠΎΠ²Π°Ρ‚Π΅Π»ΡŒΠ½Ρ‹ΠΌ ΠΈΠ»ΠΈ ΠΏΠ°Ρ€Π°Π»Π»Π΅Π»ΡŒΠ½Ρ‹ΠΌ соСдинСниСм, Π½ΠΎ ΠΏΡ€ΠΈ этом, Π·Π΅Ρ€ΠΊΠ°Π»ΡŒΠ½ΠΎ ΠΏΡ€ΠΎΡ‚ΠΈΠ²ΠΎΠΏΠΎΠ»ΠΎΠΆΠ΅Π½.

Π‘ΠΎΠ²Π΅Ρ‚ΡƒΠ΅ΠΌ ΠΏΡ€ΠΎΡ‡Π΅ΡΡ‚ΡŒ — Заряд ΠΈ разряд кондСнсатора

  • ΠŸΡ€ΠΎΡΠΌΠΎΡ‚Ρ€ΠΎΠ²:
  • ΠšΠ°Π»ΡŒΠΊΡƒΠ»ΡΡ‚ΠΎΡ€ Смкости ΠΏΠΎΡΠ»Π΅Π΄ΠΎΠ²Π°Ρ‚Π΅Π»ΡŒΠ½ΠΎΠ³ΠΎ соСдинСния кондСнсаторов β€’ ЭлСктротСхничСскиС ΠΈ радиотСхничСскиС ΠΊΠ°Π»ΡŒΠΊΡƒΠ»ΡΡ‚ΠΎΡ€Ρ‹ β€’ Онлайн-ΠΊΠΎΠ½Π²Π΅Ρ€Ρ‚Π΅Ρ€Ρ‹ Π΅Π΄ΠΈΠ½ΠΈΡ† измСрСния

    ΠšΠ°Π»ΡŒΠΊΡƒΠ»ΡΡ‚ΠΎΡ€ позволяСт Ρ€Π°ΡΡΡ‡ΠΈΡ‚Π°Ρ‚ΡŒ Π΅ΠΌΠΊΠΎΡΡ‚ΡŒ Π½Π΅ΡΠΊΠΎΠ»ΡŒΠΊΠΈΡ… кондСнсаторов, соСдинСнных ΠΏΠΎΡΠ»Π΅Π΄ΠΎΠ²Π°Ρ‚Π΅Π»ΡŒΠ½ΠΎ.

    ΠŸΡ€ΠΈΠΌΠ΅Ρ€. Π Π°ΡΡΡ‡ΠΈΡ‚Π°Ρ‚ΡŒ ΡΠΊΠ²ΠΈΠ²Π°Π»Π΅Π½Ρ‚Π½ΡƒΡŽ Π΅ΠΌΠΊΠΎΡΡ‚ΡŒ Π΄Π²ΡƒΡ… соСдинСнных ΠΏΠΎΡΠ»Π΅Π΄ΠΎΠ²Π°Ρ‚Π΅Π»ΡŒΠ½ΠΎ кондСнсаторов 10 ΠΌΠΊΠ€ ΠΈ 5 ΠΌΠΊΠ€.

    Π’Ρ…ΠΎΠ΄Π½Ρ‹Π΅ Π΄Π°Π½Π½Ρ‹Π΅

    Π”ΠΎΠ±Π°Π²ΠΈΡ‚ΡŒ кондСнсатор

    Π’Ρ‹Ρ…ΠΎΠ΄Π½Ρ‹Π΅ Π΄Π°Π½Π½Ρ‹Π΅

    ЭквивалСнтная Π΅ΠΌΠΊΠΎΡΡ‚ΡŒ

    C ΠΌΠΈΠΊΡ€ΠΎΡ„Π°Ρ€Π°Π΄ (ΠΌΠΊΠ€)

    Π’Π²Π΅Π΄ΠΈΡ‚Π΅ значСния Смкости Π² поля C1 ΠΈ C 2, Π΄ΠΎΠ±Π°Π²ΡŒΡ‚Π΅ ΠΏΡ€ΠΈ нСобходимости Π½ΠΎΠ²Ρ‹Π΅ поля, Π²Ρ‹Π±Π΅Ρ€ΠΈΡ‚Π΅ Π΅Π΄ΠΈΠ½ΠΈΡ†Ρ‹ Смкости (ΠΎΠ΄ΠΈΠ½Π°ΠΊΠΎΠ²Ρ‹Π΅ для всСх ΠΏΠΎΠ»Π΅ΠΉ Π²Π²ΠΎΠ΄Π°) Π² Ρ„Π°Ρ€Π°Π΄Π°Ρ… (Π€), ΠΌΠΈΠ»Π»ΠΈΡ„Π°Ρ€Π°Π΄Π°Ρ… (ΠΌΠ€), ΠΌΠΈΠΊΡ€ΠΎΡ„Π°Ρ€Π°Π΄Π°Ρ… (ΠΌΠΊΠ€), ΠΏΠΈΠΊΠΎΡ„Π°Ρ€Π°Π΄Π°Ρ… (ΠΏΠ€), Π½Π°Π½ΠΎΡ„Π°Ρ€Π°Π΄Π°Ρ… (Π½Π€) ΠΈ Π½Π°ΠΆΠΌΠΈΡ‚Π΅ Π½Π° ΠΊΠ½ΠΎΠΏΠΊΡƒ Π Π°ΡΡΡ‡ΠΈΡ‚Π°Ρ‚ΡŒ.

    1 мЀ = 0,001 Ѐ. 1 мкЀ = 0,000001 = 10⁻⁢ Ѐ. 1 нЀ = 0,000000001 = 10⁻⁹ Ѐ. 1 пЀ = 0,000000000001 = 10⁻¹² Ѐ.

    Π’ соотвСтствии со Π²Ρ‚ΠΎΡ€Ρ‹ΠΌ ΠΏΡ€Π°Π²ΠΈΠ»ΠΎΠΌ ΠšΠΈΡ€Ρ…Π³ΠΎΡ„Π°, падСния напряТСния V₁, Vβ‚‚ and V₃ Π½Π° ΠΊΠ°ΠΆΠ΄ΠΎΠΌ ΠΈΠ· кондСнсаторов Π² Π³Ρ€ΡƒΠΏΠΏΠ΅ ΠΈΠ· Ρ‚Ρ€Π΅Ρ… соСдинСнных ΠΏΠΎΡΠ»Π΅Π΄ΠΎΠ²Π°Ρ‚Π΅Π»ΡŒΠ½ΠΎ кондСнсаторов Π² ΠΎΠ±Ρ‰Π΅ΠΌ случаС Ρ€Π°Π·Π»ΠΈΡ‡Π½Ρ‹Π΅ ΠΈ общая Ρ€Π°Π·Π½ΠΎΡΡ‚ΡŒ ΠΏΠΎΡ‚Π΅Π½Ρ†ΠΈΠ°Π»ΠΎΠ² V Ρ€Π°Π²Π½Π° ΠΈΡ… суммС:

    По ΠΎΠΏΡ€Π΅Π΄Π΅Π»Π΅Π½ΠΈΡŽ Смкости ΠΈ с ΡƒΡ‡Π΅Ρ‚ΠΎΠΌ Ρ‚ΠΎΠ³ΠΎ, Ρ‡Ρ‚ΠΎ заряд Q Π³Ρ€ΡƒΠΏΠΏΡ‹ ΠΏΠΎΡΠ»Π΅Π΄ΠΎΠ²Π°Ρ‚Π΅Π»ΡŒΠ½ΠΎ соСдинСнных кондСнсаторов являСтся ΠΎΠ±Ρ‰ΠΈΠΌ для всСх кондСнсаторов, эквивалСнтная Π΅ΠΌΠΊΠΎΡΡ‚ΡŒ Ceq всСх Ρ‚Ρ€Π΅Ρ… кондСнсаторов, соСдинСнных ΠΏΠΎΡΠ»Π΅Π΄ΠΎΠ²Π°Ρ‚Π΅Π»ΡŒΠ½ΠΎ, опрСдСляСтся ΠΊΠ°ΠΊ

    ΠΈΠ»ΠΈ

    Для Π³Ρ€ΡƒΠΏΠΏΡ‹ ΠΈΠ· n соСдинСнных ΠΏΠΎΡΠ»Π΅Π΄ΠΎΠ²Π°Ρ‚Π΅Π»ΡŒΠ½ΠΎ кондСнсаторов эквивалСнтная Π΅ΠΌΠΊΠΎΡΡ‚ΡŒ Ceq Ρ€Π°Π²Π½Π° Π²Π΅Π»ΠΈΡ‡ΠΈΠ½Π΅, ΠΎΠ±Ρ€Π°Ρ‚Π½ΠΎΠΉ суммС Π²Π΅Π»ΠΈΡ‡ΠΈΠ½, ΠΎΠ±Ρ€Π°Ρ‚Π½Ρ‹Ρ… Смкостям ΠΎΡ‚Π΄Π΅Π»ΡŒΠ½Ρ‹Ρ… кондСнсаторов:

    ΠΈΠ»ΠΈ

    Π­Ρ‚Π° Ρ„ΠΎΡ€ΠΌΡƒΠ»Π° для Ceq ΠΈ ΠΈΡΠΏΠΎΠ»ΡŒΠ·ΡƒΠ΅Ρ‚ΡΡ для расчСтов Π² этом ΠΊΠ°Π»ΡŒΠΊΡƒΠ»ΡΡ‚ΠΎΡ€Π΅. НапримСр, общая Π΅ΠΌΠΊΠΎΡΡ‚ΡŒ соСдинСнных ΠΏΠΎΡΠ»Π΅Π΄ΠΎΠ²Π°Ρ‚Π΅Π»ΡŒΠ½ΠΎ Ρ‚Ρ€Π΅Ρ… кондСнсаторов Π΅ΠΌΠΊΠΎΡΡ‚ΡŒΡŽ 10, 15 and 20 ΠΌΠΊΠ€ Π±ΡƒΠ΄Π΅Ρ‚ Ρ€Π°Π²Π½Π° 4,62 ΠΌΠΊΠ€:

    Если кондСнсаторов Ρ‚ΠΎΠ»ΡŒΠΊΠΎ Π΄Π²Π°, Ρ‚ΠΎ ΠΈΡ… общая Π΅ΠΌΠΊΠΎΡΡ‚ΡŒ опрСдСляСтся ΠΏΠΎ Ρ„ΠΎΡ€ΠΌΡƒΠ»Π΅

    ΠΈΠ»ΠΈ

    Если имССтся n соСдинСнных ΠΏΠΎΡΠ»Π΅Π΄ΠΎΠ²Π°Ρ‚Π΅Π»ΡŒΠ½ΠΎ кондСнсаторов с Π΅ΠΌΠΊΠΎΡΡ‚ΡŒΡŽ C, ΠΈΡ… эквивалСнтная Π΅ΠΌΠΊΠΎΡΡ‚ΡŒ Ρ€Π°Π²Π½Π°

    ΠžΡ‚ΠΌΠ΅Ρ‚ΠΈΠΌ, Ρ‡Ρ‚ΠΎ для расчСта ΠΎΠ±Ρ‰Π΅ΠΉ Смкости Π½Π΅ΡΠΊΠΎΠ»ΡŒΠΊΠΈΡ… соСдинСнных ΠΏΠΎΡΠ»Π΅Π΄ΠΎΠ²Π°Ρ‚Π΅Π»ΡŒΠ½ΠΎ кондСнсаторов ΠΈΡΠΏΠΎΠ»ΡŒΠ·ΡƒΠ΅Ρ‚ΡΡ Ρ‚Π° ΠΆΠ΅ Ρ„ΠΎΡ€ΠΌΡƒΠ»Π°, Ρ‡Ρ‚ΠΎ ΠΈ для расчСта ΠΎΠ±Ρ‰Π΅Π³ΠΎ сопротивлСния ΠΏΠ°Ρ€Π°Π»Π»Π΅Π»ΡŒΠ½ΠΎ соСдинСнных рСзисторов.

    ΠžΡ‚ΠΌΠ΅Ρ‚ΠΈΠΌ Ρ‚Π°ΠΊΠΆΠ΅, Ρ‡Ρ‚ΠΎ общая Π΅ΠΌΠΊΠΎΡΡ‚ΡŒ Π³Ρ€ΡƒΠΏΠΏΡ‹ ΠΈΠ· любого количСства ΠΏΠΎΡΠ»Π΅Π΄ΠΎΠ²Π°Ρ‚Π΅Π»ΡŒΠ½ΠΎ соСдинСнных кондСнсаторов всСгда Π±ΡƒΠ΄Π΅Ρ‚ мСньшС, Ρ‡Π΅ΠΌ Π΅ΠΌΠΊΠΎΡΡ‚ΡŒ самого малСнького кондСнсатора, Π° Π΄ΠΎΠ±Π°Π²Π»Π΅Π½ΠΈΠ΅ кондСнсаторов Π² Π³Ρ€ΡƒΠΏΠΏΡƒ всСгда ΠΏΡ€ΠΈΠ²ΠΎΠ΄ΠΈΡ‚ ΠΊ ΡƒΠΌΠ΅Π½ΡŒΡˆΠ΅Π½ΠΈΡŽ Смкости.

    ΠšΠΎΠ½Π΄Π΅Π½ΡΠ°Ρ‚ΠΎΡ€Ρ‹ Π½Π° ΠΏΠ΅Ρ‡Π°Ρ‚Π½ΠΎΠΉ ΠΏΠ»Π°Ρ‚Π΅

    ΠžΡ‚Π΄Π΅Π»ΡŒΠ½ΠΎΠ³ΠΎ упоминания заслуТиваСт ΠΏΠ°Π΄Π΅Π½ΠΈΠ΅ напряТСния Π½Π° ΠΊΠ°ΠΆΠ΄ΠΎΠΌ кондСнсаторС Π² Π³Ρ€ΡƒΠΏΠΏΠ΅ ΠΏΠΎΡΠ»Π΅Π΄ΠΎΠ²Π°Ρ‚Π΅Π»ΡŒΠ½ΠΎ соСдинСнных кондСнсаторов. Если всС кондСнсаторы Π² Π³Ρ€ΡƒΠΏΠΏΠ΅ ΠΈΠΌΠ΅ΡŽΡ‚ ΠΎΠ΄ΠΈΠ½Π°ΠΊΠΎΠ²ΡƒΡŽ Π½ΠΎΠΌΠΈΠ½Π°Π»ΡŒΠ½ΡƒΡŽ Π΅ΠΌΠΊΠΎΡΡ‚ΡŒ, ΠΏΠ°Π΄Π΅Π½ΠΈΠ΅ напряТСния Π½Π° Π½ΠΈΡ… скорСС всСго Π±ΡƒΠ΄Π΅Ρ‚ Ρ€Π°Π·Π½Ρ‹ΠΌ, Ρ‚Π°ΠΊ ΠΊΠ°ΠΊ кондСнсаторы Π² Ρ€Π΅Π°Π»ΡŒΠ½ΠΎΡΡ‚ΠΈ Π±ΡƒΠ΄ΡƒΡ‚ ΠΈΠΌΠ΅Ρ‚ΡŒ Ρ€Π°Π·Π½ΡƒΡŽ Π΅ΠΌΠΊΠΎΡΡ‚ΡŒ ΠΈ Ρ€Π°Π·Π½Ρ‹ΠΉ Ρ‚ΠΎΠΊ ΡƒΡ‚Π΅Ρ‡ΠΊΠΈ. На кондСнсаторС с наимСньшСй Π΅ΠΌΠΊΠΎΡΡ‚ΡŒΡŽ Π±ΡƒΠ΄Π΅Ρ‚ наибольшСС ΠΏΠ°Π΄Π΅Π½ΠΈΠ΅ напряТСния ΠΈ, Ρ‚Π°ΠΊΠΈΠΌ ΠΎΠ±Ρ€Π°Π·ΠΎΠΌ, ΠΎΠ½ Π±ΡƒΠ΄Π΅Ρ‚ самым слабым Π·Π²Π΅Π½ΠΎΠΌ этой Ρ†Π΅ΠΏΠΈ.

    Π’Ρ‹Ρ€Π°Π²Π½ΠΈΠ²Π°ΡŽΡ‰ΠΈΠ΅ рСзисторы ΡƒΠΌΠ΅Π½ΡŒΡˆΠ°ΡŽΡ‚ разброс напряТСний Π½Π° ΠΎΡ‚Π΄Π΅Π»ΡŒΠ½Ρ‹Ρ… кондСнсаторах

    Для получСния Π±ΠΎΠ»Π΅Π΅ Ρ€Π°Π²Π½ΠΎΠΌΠ΅Ρ€Π½ΠΎΠ³ΠΎ распрСдСлСния напряТСний ΠΏΠ°Ρ€Π°Π»Π»Π΅Π»ΡŒΠ½ΠΎ кондСнсаторам Π²ΠΊΠ»ΡŽΡ‡Π°ΡŽΡ‚ Π²Ρ‹Ρ€Π°Π²Π½ΠΈΠ²Π°ΡŽΡ‰ΠΈΠ΅ рСзисторы. Π­Ρ‚ΠΈ рСзисторы Ρ€Π°Π±ΠΎΡ‚Π°ΡŽΡ‚ ΠΊΠ°ΠΊ Π΄Π΅Π»ΠΈΡ‚Π΅Π»ΠΈ напряТСния, ΡƒΠΌΠ΅Π½ΡŒΡˆΠ°ΡŽΡ‰ΠΈΠ΅ разброс напряТСний Π½Π° ΠΎΡ‚Π΄Π΅Π»ΡŒΠ½Ρ‹Ρ… кондСнсаторах. Но Π΄Π°ΠΆΠ΅ с этими рСзисторами всС Ρ€Π°Π²Π½ΠΎ для ΠΏΠΎΡΠ»Π΅Π΄ΠΎΠ²Π°Ρ‚Π΅Π»ΡŒΠ½ΠΎΠ³ΠΎ Π²ΠΊΠ»ΡŽΡ‡Π΅Π½ΠΈΡ слСдуСт Π²Ρ‹Π±ΠΈΡ€Π°Ρ‚ΡŒ кондСнсаторы с большим запасом ΠΏΠΎ Ρ€Π°Π±ΠΎΡ‡Π΅ΠΌΡƒ Π½Π°ΠΏΡ€ΡΠΆΠ΅Π½ΠΈΡŽ.

    Если нСсколько кондСнсаторов соСдинСны ΠΏΠ°Ρ€Π°Π»Π»Π΅Π»ΡŒΠ½ΠΎ, Ρ€Π°Π·Π½ΠΎΡΡ‚ΡŒ ΠΏΠΎΡ‚Π΅Π½Ρ†ΠΈΠ°Π»ΠΎΠ² V Π½Π° Π³Ρ€ΡƒΠΏΠΏΠ΅ кондСнсаторов Ρ€Π°Π²Π½Π° разности ΠΏΠΎΡ‚Π΅Π½Ρ†ΠΈΠ°Π»ΠΎΠ² ΡΠΎΠ΅Π΄ΠΈΠ½ΠΈΡ‚Π΅Π»ΡŒΠ½Ρ‹Ρ… ΠΏΡ€ΠΎΠ²ΠΎΠ΄ΠΎΠ² Π³Ρ€ΡƒΠΏΠΏΡ‹. ΠžΠ±Ρ‰ΠΈΠΉ заряд Q раздСляСтся ΠΌΠ΅ΠΆΠ΄Ρƒ кондСнсаторами ΠΈ Ссли ΠΈΡ… Смкости Ρ€Π°Π·Π»ΠΈΡ‡Π½Ρ‹, Ρ‚ΠΎ заряды Π½Π° ΠΎΡ‚Π΄Π΅Π»ΡŒΠ½Ρ‹Ρ… кондСнсаторах

    Q₁, Qβ‚‚ and Q₃ Ρ‚ΠΎΠΆΠ΅ Π±ΡƒΠ΄ΡƒΡ‚ Ρ€Π°Π·Π»ΠΈΡ‡Π½Ρ‹ΠΌΠΈ. ΠžΠ±Ρ‰ΠΈΠΉ заряд опрСдСляСтся ΠΊΠ°ΠΊ

    ΠšΠΎΠ½Π΄Π΅Π½ΡΠ°Ρ‚ΠΎΡ€Ρ‹, соСдинСнныС ΠΏΠ°Ρ€Π°Π»Π»Π΅Π»ΡŒΠ½ΠΎ

    По ΠΎΠΏΡ€Π΅Π΄Π΅Π»Π΅Π½ΠΈΡŽ Смкости, эквивалСнтная Π΅ΠΌΠΊΠΎΡΡ‚ΡŒ Π³Ρ€ΡƒΠΏΠΏΡ‹ кондСнсаторов Ρ€Π°Π²Π½Π°

    ΠΎΡ‚ΡΡŽΠ΄Π°

    ΠΈΠ»ΠΈ

    Для Π³Ρ€ΡƒΠΏΠΏΡ‹ n Π²ΠΊΠ»ΡŽΡ‡Π΅Π½Π½Ρ‹Ρ… ΠΏΠ°Ρ€Π°Π»Π»Π΅Π»ΡŒΠ½ΠΎ кондСнсаторов

    Π’ΠΎ Π΅ΡΡ‚ΡŒ, Ссли нСсколько кондСнсаторов Π²ΠΊΠ»ΡŽΡ‡Π΅Π½Ρ‹ ΠΏΠ°Ρ€Π°Π»Π»Π΅Π»ΡŒΠ½ΠΎ, ΠΈΡ… эквивалСнтная Π΅ΠΌΠΊΠΎΡΡ‚ΡŒ опрСдСляСтся ΠΏΡƒΡ‚Π΅ΠΌ слоТСния СмкостСй всСх кондСнсаторов Π² Π³Ρ€ΡƒΠΏΠΏΠ΅.

    Π’ΠΎΠ·ΠΌΠΎΠΆΠ½ΠΎ, Π²Ρ‹ Π·Π°ΠΌΠ΅Ρ‚ΠΈΠ»ΠΈ, Ρ‡Ρ‚ΠΎ кондСнсаторы Π²Π΅Π΄ΡƒΡ‚ сСбя ΠΏΡ€ΠΎΡ‚ΠΈΠ²ΠΎΠΏΠΎΠ»ΠΎΠΆΠ½ΠΎ рСзисторам: Ссли рСзисторы соСдинСны ΠΏΠΎΡΠ»Π΅Π΄ΠΎΠ²Π°Ρ‚Π΅Π»ΡŒΠ½ΠΎ, ΠΈΡ… ΠΎΠ±Ρ‰Π΅Π΅ сопротивлСниС всСгда Π±ΡƒΠ΄Π΅Ρ‚ Π²Ρ‹ΡˆΠ΅ сопротивлСний ΠΎΡ‚Π΄Π΅Π»ΡŒΠ½Ρ‹Ρ… рСзисторов, Π° Π² случаС кондСнсаторов всё происходит с Ρ‚ΠΎΡ‡Π½ΠΎΡΡ‚ΡŒΡŽ Π΄ΠΎ Π½Π°ΠΎΠ±ΠΎΡ€ΠΎΡ‚.

    ΠšΠΎΠ½Π΄Π΅Π½ΡΠ°Ρ‚ΠΎΡ€Ρ‹ Π½Π° ΠΏΠ΅Ρ‡Π°Ρ‚Π½ΠΎΠΉ ΠΏΠ»Π°Ρ‚Π΅

    ΠŸΠΎΡΠ»Π΅Π΄ΠΎΠ²Π°Ρ‚Π΅Π»ΡŒΠ½ΠΎΠ΅ ΠΈ ΠΏΠ°Ρ€Π°Π»Π»Π΅Π»ΡŒΠ½ΠΎΠ΅ соСдинСниС кондСнсаторов


    ΠŸΠΎΡΠ»Π΅Π΄ΠΎΠ²Π°Ρ‚Π΅Π»ΡŒΠ½ΠΎΠ΅ ΠΈ ΠΏΠ°Ρ€Π°Π»Π»Π΅Π»ΡŒΠ½ΠΎΠ΅ соСдинСниС кондСнсаторов ΠΏΡ€ΠΈΠΌΠ΅Π½ΡΡŽΡ‚ Π² зависимости ΠΎΡ‚ поставлСнной Ρ†Π΅Π»ΠΈ. ΠŸΡ€ΠΈ ΠΏΠΎΡΠ»Π΅Π΄ΠΎΠ²Π°Ρ‚Π΅Π»ΡŒΠ½ΠΎΠΌ соСдинСнии кондСнсаторов ΡƒΠΌΠ΅Π½ΡŒΡˆΠ°Π΅Ρ‚ΡΡ общая Π΅ΠΌΠΊΠΎΡΡ‚ΡŒ ΠΈ увСличиваСтся ΠΎΠ±Ρ‰Π΅Π΅ напряТСниС кондСнсаторов.
    Π•ΠΌΠΊΠΎΡΡ‚ΡŒ Π½Π°Π±ΠΎΡ€Π° ΠΏΡ€ΠΈ ΠΏΠΎΡΠ»Π΅Π΄ΠΎΠ²Π°Ρ‚Π΅Π»ΡŒΠ½ΠΎΠΌ соСдинСнии кондСнсаторов Π±ΡƒΠ΄Π΅Ρ‚ Π²Ρ‹Ρ‡ΠΈΡΠ»ΡΡ‚ΡŒΡΡ ΠΏΠΎ Ρ„ΠΎΡ€ΠΌΡƒΠ»Π΅:

    1 = 1 + 1 + 1 + …
    CC1C2C3

    А ΠΎΠ±Ρ‰Π΅Π΅ напряТСниС Π±ΡƒΠ΄Π΅Ρ‚ Ρ€Π°Π²Π½ΡΡ‚ΡŒΡΡ суммС напряТСний всСх кондСнсаторов.
    НапримСр: ΠΌΡ‹ ΠΈΠΌΠ΅Π΅ΠΌ Ρ‚Ρ€ΠΈ кондСнсатора ΠΏΠΎ 30 ΠΌΠΊΠ€ x 100 Π’ ΠΊΠ°ΠΆΠ΄Ρ‹ΠΉ. ΠŸΡ€ΠΈ ΠΈΡ… ΠΏΠΎΡΠ»Π΅Π΄ΠΎΠ²Π°Ρ‚Π΅Π»ΡŒΠ½ΠΎΠΌ соСдинСнии ΠΎΠ±Ρ‰ΠΈΠΉ кондСнсатор Π±ΡƒΠ΄Π΅Ρ‚ ΠΈΠΌΠ΅Ρ‚ΡŒ ΡΠ»Π΅Π΄ΡƒΡŽΡ‰ΠΈΠ΅ Π΄Π°Π½Π½Ρ‹Π΅: 10 ΠΌΠΊΠ€ x 300 Π’.

    ΠŸΡ€ΠΈ ΠΏΠ°Ρ€Π°Π»Π»Π΅Π»ΡŒΠ½ΠΎΠΌ соСдинСнии общая Π΅ΠΌΠΊΠΎΡΡ‚ΡŒ кондСнсаторов складываСтся, Π° допустимоС напряТСниС всСго Π½Π°Π±ΠΎΡ€Π° Π±ΡƒΠ΄Π΅Ρ‚ Ρ€Π°Π²Π½ΠΎ Π½Π°ΠΏΡ€ΡΠΆΠ΅Π½ΠΈΡŽ кондСнсатора, ΠΈΠΌΠ΅ΡŽΡ‰Π΅Π³ΠΎ самоС Π½ΠΈΠ·ΠΊΠΎΠ΅ Π·Π½Π°Ρ‡Π΅Π½ΠΈΠ΅ допустимого напряТСния ΠΈΠ· всСго Π½Π°Π±ΠΎΡ€Π°.

    C = C1 + C2 + C3 + C4 + …

    НапримСр: ΠΌΡ‹ ΠΈΠΌΠ΅Π΅ΠΌ Ρ‚Ρ€ΠΈ кондСнсатора 30 ΠΌΠΊΠ€ x 100 Π’, соСдинённыС ΠΏΠ°Ρ€Π°Π»Π»Π΅Π»ΡŒΠ½ΠΎ. ΠŸΠ°Ρ€Π°ΠΌΠ΅Ρ‚Ρ€Ρ‹ всСго Π½Π°Π±ΠΎΡ€Π° кондСнсаторов Π² этом случаС Π±ΡƒΠ΄ΡƒΡ‚ ΡΠ»Π΅Π΄ΡƒΡŽΡ‰ΠΈΠ΅: 90 ΠΌΠΊΠ€ x 100 Π’.

    Π‘ΠΎΠ΅Π΄ΠΈΠ½Π΅Π½ΠΈΠ΅ Π±ΠΎΠ»Π΅Π΅ Π΄Π²ΡƒΡ… кондСнсаторов ΠΏΠΎΡΠ»Π΅Π΄ΠΎΠ²Π°Ρ‚Π΅Π»ΡŒΠ½ΠΎ Ρ€Π΅Π΄ΠΊΠΎ встрСчаСтся Π² Ρ€Π΅Π°Π»ΡŒΠ½Ρ‹Ρ… схСмах. Π₯отя для увСличСния ΠΎΠ±Ρ‰Π΅Π³ΠΎ напряТСния Ρ‚Π°ΠΊΠΎΠΉ Π½Π°Π±ΠΎΡ€ ΠΌΠΎΠΆΠ΅Ρ‚ Π²ΡΡ‚Ρ€Π΅Ρ‚ΠΈΡ‚ΡŒΡΡ Π² Π²Ρ‹ΡΠΎΠΊΠΎΠ²ΠΎΠ»ΡŒΡ‚Π½Ρ‹Ρ… источниках питания. А Π²ΠΎΡ‚ Π² Π½ΠΈΠ·ΠΊΠΎΠ²ΠΎΠ»ΡŒΡ‚Π½Ρ‹Ρ… источниках довольно часто встрСчаСтся ΠΏΠ°Ρ€Π°Π»Π»Π΅Π»ΡŒΠ½ΠΎΠ΅ соСдинСниС Π½Π΅ΡΠΊΠΎΠ»ΡŒΠΊΠΈΡ… кондСнсаторов для сглаТивания ΠΏΡƒΠ»ΡŒΡΠ°Ρ†ΠΈΠΉ послС выпрямлСния ΠΏΡ€ΠΈ Π±ΠΎΠ»ΡŒΡˆΠΈΡ… Ρ‚ΠΎΠΊΠ°Ρ… потрСблСния.

    ΠžΠ±Ρ€Π°Ρ‚ΠΈΡ‚Π΅ Π²Π½ΠΈΠΌΠ°Π½ΠΈΠ΅, Ρ„ΠΎΡ€ΠΌΡƒΠ»Ρ‹ вычислСния Смкости ΠΏΠΎΡΠ»Π΅Π΄ΠΎΠ²Π°Ρ‚Π΅Π»ΡŒΠ½ΠΎΠ³ΠΎ ΠΈ ΠΏΠ°Ρ€Π°Π»Π»Π΅Π»ΡŒΠ½ΠΎΠ³ΠΎ соСдинСния кондСнсаторов Π² точности ΠΎΠ±Ρ€Π°Ρ‚Π½Ρ‹ Ρ„ΠΎΡ€ΠΌΡƒΠ»Π°ΠΌ вычислСния сопротивлСния ΠΏΡ€ΠΈ ΠΏΠΎΡΠ»Π΅Π΄ΠΎΠ²Π°Ρ‚Π΅Π»ΡŒΠ½ΠΎΠΌ ΠΈ ΠΏΠ°Ρ€Π°Π»Π»Π΅Π»ΡŒΠ½ΠΎΠΌ соСдинСнии рСзисторов.


    Π—Π°Π΄Π°Ρ‡ΠΈ с кондСнсаторами: сборная солянка

    Π’ эту ΡΡ‚Π°Ρ‚ΡŒΡŽ вошли Π·Π°Π΄Π°Ρ‡ΠΈ всСх Ρ‚ΠΈΠΏΠΎΠ²: здСсь ΠΈ ΠΎΠΏΡ€Π΅Π΄Π΅Π»Π΅Π½ΠΈΠ΅ эквивалСнтных СмкостСй, ΠΈ напряТСний ΠΌΠ΅ΠΆΠ΄Ρƒ ΠΎΠΏΡ€Π΅Π΄Π΅Π»Π΅Π½Π½Ρ‹ΠΌΠΈ Ρ‚ΠΎΡ‡ΠΊΠ°ΠΌΠΈ схСмы, ΠΈ бСсконСчныС Ρ†Π΅ΠΏΠΎΡ‡ΠΊΠΈ, ΠΈ Π΄Π°ΠΆΠ΅ исчСзновСниС кондСнсаторов ΠΈΠ· схСм (бСсслСдноС ΠΈ Π±Π΅Π· послСдствий).

    Β 

    Π—Π°Π΄Π°Ρ‡Π° 1. Плоский кондСнсатор Ρ€Π°Π·Ρ€Π΅Π·Π°ΡŽΡ‚ Π½Π° Ρ€Π°Π²Π½Ρ‹Π΅ части вдоль плоскостСй, пСрпСндикулярных ΠΎΠ±ΠΊΠ»Π°Π΄ΠΊΠ°ΠΌ. ΠŸΠΎΠ»ΡƒΡ‡Π΅Π½Π½Ρ‹Π΅ кондСнсаторов ΡΠΎΠ΅Π΄ΠΈΠ½ΡΡŽΡ‚ ΠΏΠΎΡΠ»Π΅Π΄ΠΎΠ²Π°Ρ‚Π΅Π»ΡŒΠ½ΠΎ. Π§Π΅ΠΌΡƒ Ρ€Π°Π²Π½Π° Π΅ΠΌΠΊΠΎΡΡ‚ΡŒ ΠΏΠΎΠ»ΡƒΡ‡Π΅Π½Π½ΠΎΠΉ Π±Π°Ρ‚aΡ€Π΅ΠΈ кондСнсаторов, Ссли Π΅ΠΌΠΊΠΎΡΡ‚ΡŒ исходного кондСнсатора ΠΌΠΊΠ€?

    К Π·Π°Π΄Π°Ρ‡Π΅ 1

    ΠŸΠ»ΠΎΡ‰Π°Π΄ΡŒ исходного кондСнсатора:

    Β  Β 

    ΠŸΠ»ΠΎΡ‰Π°Π΄ΡŒ Π½ΠΎΠ²ΠΎΠ³ΠΎ кондСнсатора (ΠΎΠ΄Π½ΠΎΠ³ΠΎ)Β  – Π² 4 Ρ€Π°Π·Π° мСньшС исходного (Ρ‚Π°ΠΊ ΠΊΠ°ΠΊ ΠΏΠ»ΠΎΡ‰Π°Π΄ΡŒ мСньшС):

    Β  Β 

    Π’Π΅ΠΏΠ΅Ρ€ΡŒ соСдиняСм ΠΏΠΎΡΠ»Π΅Π΄ΠΎΠ²Π°Ρ‚Π΅Π»ΡŒΠ½ΠΎ:

    Β  Β 

    Β  Β 

    ΠžΡ‚Π²Π΅Ρ‚: 1 ΠΌΠΊΠ€.

    Π—Π°Π΄Π°Ρ‡Π° 2. Π”Π²Π° плоских кондСнсатора, Π΅ΠΌΠΊΠΎΡΡ‚ΡŒΡŽ ΠΊΠ°ΠΆΠ΄Ρ‹ΠΉ, соСдинили ΠΏΠ°Ρ€Π°Π»Π»Π΅Π»ΡŒΠ½ΠΎ. Π’ ΠΎΠ΄ΠΈΠ½ ΠΈΠ· Π½ΠΈΡ… вставили Π΄ΠΈΡΠ»Π΅ΠΊΡ‚Ρ€ΠΈΡ‡Π΅ΡΠΊΡƒΡŽ пластину с ΠΏΡ€ΠΎΠ½ΠΈΡ†Π°Π΅ΠΌΠΎΡΡ‚ΡŒΡŽ , Π·Π°ΠΏΠΎΠ»Π½ΠΈΠ²ΡˆΡƒΡŽ вСсь объСм кондСнсатора. Какой Смкости ΠΈ ΠΊΠ°ΠΊ Π½Π΅ΠΎΠ±Ρ…ΠΎΠ΄ΠΈΠΌΠΎ ΠΏΠΎΠ΄ΠΊΠ»ΡŽΡ‡ΠΈΡ‚ΡŒ Ρ‚Ρ€Π΅Ρ‚ΠΈΠΉ кондСнсатор, Ρ‡Ρ‚ΠΎΠ±Ρ‹ Π΅ΠΌΠΊΠΎΡΡ‚ΡŒ систСмы стала Ρ€Π°Π²Π½ΠΎΠΉ ?

    Π’Π°ΠΊ ΠΊΠ°ΠΊ ΠΏΠ΅Ρ€Π²Ρ‹Π΅ Π΄Π²Π° кондСнсатора соСдинСны ΠΏΠ°Ρ€Π°Π»Π»Π΅Π»ΡŒΠ½ΠΎ, Ρ‚ΠΎ ΠΈΡ… Смкости Π½Π°Π΄ΠΎ ΡΠ»ΠΎΠΆΠΈΡ‚ΡŒ, Ρ‡Ρ‚ΠΎΠ±Ρ‹ ΠΏΠΎΠ»ΡƒΡ‡ΠΈΡ‚ΡŒ ΡΠΊΠ²ΠΈΠ²Π°Π»Π΅Π½Ρ‚Π½ΡƒΡŽ Π΅ΠΌΠΊΠΎΡΡ‚ΡŒ:

    Β  Β 

    ПослС ввСдСния пластины Π΅ΠΌΠΊΠΎΡΡ‚ΡŒ Ρ‚Π°ΠΊΠΎΠ³ΠΎ кондСнсатора стала Ρ€Π°Π²Π½Π° , Π° эквивалСнтная Π΅ΠΌΠΊΠΎΡΡ‚ΡŒ стала Ρ€Π°Π²Π½Π°

    Β  Β 

    Π’Π΅ΠΏΠ΅Ρ€ΡŒ ΠΊ этой конструкции Π±ΡƒΠ΄Π΅ΠΌ ΠΏΡ€ΠΈΡΠΎΠ΅Π΄ΠΈΠ½ΡΡ‚ΡŒ Π΅Ρ‰Π΅ ΠΎΠ΄ΠΈΠ½ кондСнсатор. ΠŸΠΎΠΏΡ€ΠΎΠ±ΡƒΠ΅ΠΌ ΠΏΡ€ΠΈΡΠΎΠ΅Π΄ΠΈΠ½ΠΈΡ‚ΡŒ ΠΏΠ°Ρ€Π°Π»Π»Π΅Π»ΡŒΠ½ΠΎ, Ρ‚ΠΎΠ³Π΄Π°

    Β  Β 

    Β  Β 

    Π’Π°ΠΊ ΠΊΠ°ΠΊ , Ρ‚ΠΎ , .

    Π’Π΅ΠΏΠ΅Ρ€ΡŒ присоСдиняСм ΠΏΠΎΡΠ»Π΅Π΄ΠΎΠ²Π°Ρ‚Π΅Π»ΡŒΠ½ΠΎ, Ρ‚ΠΎΠ³Π΄Π°:

    Β  Β 

    Β  Β 

    Β  Β 

    Β  Β 

    Β  Β 

    Β  Β 

    Β  Β 

    РСшим нСравСнство:

    Β  Β 

    РСшСниС – – это Ρ€Π΅ΡˆΠ΅Π½ΠΈΠ΅ Π½Π΅ ΠΈΠΌΠ΅Π΅Ρ‚ смысла, .

    ΠžΡ‚Π²Π΅Ρ‚: , ΠΏΡ€ΠΈ , ΠΏΠ°Ρ€Π°Π»Π»Π΅Π»ΡŒΠ½ΠΎ.

    , ΠΏΡ€ΠΈ , ΠΏΠΎΡΠ»Π΅Π΄ΠΎΠ²Π°Ρ‚Π΅Π»ΡŒΠ½ΠΎ.

    Π—Π°Π΄Π°Ρ‡Π° 3. Π Π°Π·Π½ΠΎΡΡ‚ΡŒ ΠΏΠΎΡ‚Π΅Π½Ρ†ΠΈΠ°Π»ΠΎΠ² ΠΌΠ΅ΠΆΠ΄Ρƒ Ρ‚ΠΎΡ‡ΠΊΠ°ΠΌΠΈ А ΠΈ Π’ Ρ€Π°Π²Π½Π° . Емкости кондСнсаторов извСстны. ΠžΠΏΡ€Π΅Π΄Π΅Π»ΠΈΡ‚ΡŒ заряды кондСнсаторов ΠΈ Ρ€Π°Π·Π½ΠΎΡΡ‚ΡŒ ΠΏΠΎΡ‚Π΅Π½Ρ†ΠΈΠ°Π»ΠΎΠ² ΠΌΠ΅ΠΆΠ΄Ρƒ Ρ‚ΠΎΡ‡ΠΊΠ°ΠΌΠΈ А ΠΈ D.

    К Π·Π°Π΄Π°Ρ‡Π΅ 3

    Π’Π°ΠΊ ΠΊΠ°ΠΊ Смкости ΠΈ соСдинСны ΠΏΠ°Ρ€Π°Π»Π»Π΅Π»ΡŒΠ½ΠΎ, Ρ‚ΠΎ напряТСниС Π½Π° Π½ΠΈΡ… ΠΎΠ΄ΠΈΠ½Π°ΠΊΠΎΠ²ΠΎΠ΅. ΠšΡ€ΠΎΠΌΠ΅ Ρ‚ΠΎΠ³ΠΎ, заряды Π½Π° Смкостях ΠΈ систСмС кондСнсаторов ΠΎΠ΄ΠΈΠ½Π°ΠΊΠΎΠ²Ρ‹, Ρ‚Π°ΠΊ ΠΊΠ°ΠΊ ΠΎΠ½ΠΈ соСдинСны ΠΏΠΎΡΠ»Π΅Π΄ΠΎΠ²Π°Ρ‚Π΅Π»ΡŒΠ½ΠΎ. ΠŸΠΎΡΡ‚ΠΎΠΌΡƒ

    Β  Β 

    Β  Β 

    Β  Β 

    Β  Β 

    ЭквивалСнтная Π΅ΠΌΠΊΠΎΡΡ‚ΡŒ , поэтому эквивалСнтная Π΅ΠΌΠΊΠΎΡΡ‚ΡŒ всСй схСмы – ΠΏΡ€ΠΎΠΈΠ·Π²Π΅Π΄Π΅Π½ΠΈΠ΅ Π½Π° сумму – .

    Π’ΠΎΠ³Π΄Π° заряд

    Β  Β 

    Но вслСдствиС (1)

    Β  Β 

    Π’ΠΎΠ³Π΄Π°

    Β  Β 

    Находим :

    Β  Β 

    ΠžΠΏΡ€Π΅Π΄Π΅Π»ΠΈΠΌ заряды ΠΈ :

    Β  Β 

    Β  Β 

    ΠžΡ‚Π²Π΅Ρ‚: , , , .
    Π—Π°Π΄Π°Ρ‡Π° 4. ΠžΠΏΡ€Π΅Π΄Π΅Π»ΠΈΡ‚ΡŒ Π΅ΠΌΠΊΠΎΡΡ‚ΡŒ Π±Π°Ρ‚Π°Ρ€Π΅ΠΈ кондСнсаторов, ΠΏΠΎΠΊΠ°Π·Π°Π½Π½ΠΎΠΉ Π½Π° рисункС, Ссли ΠΌΠΊΠ€, ΠΌΠΊΠ€, ΠΌΠΊΠ€.

    К Π·Π°Π΄Π°Ρ‡Π΅ 4

    Π‘Π½Π°Ρ‡Π°Π»Π° Π΄Π²Π° кондСнсатора ΠΏΠΎΠ΄ΠΊΠ»ΡŽΡ‡Π΅Π½Ρ‹ ΠΏΠ°Ρ€Π°Π»Π»Π΅Π»ΡŒΠ½ΠΎ, ΠΏΡ€ΠΈ этом Смкости ΡΠΊΠ»Π°Π΄Ρ‹Π²Π°ΡŽΡ‚ΡΡ: . Π’ ΠΊΠΎΠ½Ρ†Π΅ ΠΏΠ°Ρ€Π°Π»Π»Π΅Π»ΡŒΠ½ΠΎΠ΅ соСдинСниС ΠΈ : . Π’Π΅ΠΏΠ΅Ρ€ΡŒ ΠΈΠΌΠ΅Π΅ΠΌ ΠΏΠΎΡΠ»Π΅Π΄ΠΎΠ²Π°Ρ‚Π΅Π»ΡŒΠ½ΠΎΠ΅ соСдинСниС СмкостСй , ΠΈ . Π’ΠΎΠ³Π΄Π°

    Β  Β 

    МоТно ΠΏΠΎΠ΄ΡΡ‚Π°Π²ΠΈΡ‚ΡŒ числа ΠΈ довСсти Ρ€Π΅ΡˆΠ΅Π½ΠΈΠ΅ Π΄ΠΎ ΠΊΠΎΠ½Ρ†Π°:

    Β  Β 

    Β  Β 

    ΠžΡ‚Π²Π΅Ρ‚: ΠΌΠΊΠ€.

    Π—Π°Π΄Π°Ρ‡Π° 5. Найти Π΅ΠΌΠΊΠΎΡΡ‚ΡŒ систСмы кондСнсаторов, ΠΈΠ·ΠΎΠ±Ρ€Π°ΠΆΠ΅Π½Π½ΠΎΠΉ Π½Π° рисункС.

    К Π·Π°Π΄Π°Ρ‡Π΅ 5

    На рисункС a) Π΅ΠΌΠΊΠΎΡΡ‚ΡŒ оказываСтся нСзаряТСнной, Ρ‚Π°ΠΊ ΠΊΠ°ΠΊ схСма ΡΠΎΠ²Π΅Ρ€ΡˆΠ΅Π½Π½ΠΎ симмСтрична ΠΈ , поэтому .

    Рисунок 2 (Π·Π°Π΄Π°Ρ‡Π° 5)

    ΠŸΠΎΡΡ‚ΠΎΠΌΡƒ кондСнсатор Π½Π΅ заряТСн – Ρ€Π°Π·Π½ΠΎΡΡ‚ΡŒ ΠΏΠΎΡ‚Π΅Π½Ρ†ΠΈΠ°Π»ΠΎΠ² Π½Π° Π΅Π³ΠΎ Π²Ρ‹Π²ΠΎΠ΄Π°Ρ… нулСвая. Π‘Π»Π΅Π΄ΠΎΠ²Π°Ρ‚Π΅Π»ΡŒΠ½ΠΎ, ΠΈΠΌΠ΅Π΅ΠΌ Π΄Π²Π΅ Π²Π΅Ρ‚ΠΎΡ‡ΠΊΠΈ, Π²ΠΊΠ»ΡŽΡ‡Π΅Π½Π½Ρ‹Π΅ Π² ΠΏΠ°Ρ€Π°Π»Π»Π΅Π»ΡŒ: Π² ΠΊΠ°ΠΆΠ΄ΠΎΠΉ ΠΏΠΎΡΠ»Π΅Π΄ΠΎΠ²Π°Ρ‚Π΅Π»ΡŒΠ½ΠΎΠ΅ соСдинСниС ΠΈ .

    Рисунок 3 (ΠΊ Π·Π°Π΄Π°Ρ‡Π΅ 5)

    Π‘ΠΎΠΏΡ€ΠΎΡ‚ΠΈΠ²Π»Π΅Π½ΠΈΠ΅ ΠΎΠ΄Π½ΠΎΠΉ Π²Π΅Ρ‚ΠΊΠΈ (Π΅ΠΌΠΊΠΎΡΡ‚ΡŒ Π΄Π²ΡƒΡ… ΠΏΠΎΡΠ»Π΅Π΄ΠΎΠ²Π°Ρ‚Π΅Π»ΡŒΠ½ΠΎ Π²ΠΊΠ»ΡŽΡ‡Π΅Π½Π½Ρ‹Ρ… кондСнсаторов – ΠΏΡ€ΠΎΠΈΠ·Π²Π΅Π΄Π΅Π½ΠΈΠ΅, Π΄Π΅Π»Π΅Π½Π½ΠΎΠ΅ Π½Π° сумму):

    Β  Β 

    А Π΄Π²ΡƒΡ… Ρ‚Π°ΠΊΠΈΡ… Π²Π΅Ρ‚ΠΎΠΊ Π² ΠΏΠ°Ρ€Π°Π»Π»Π΅Π»ΡŒ (Смкости, Π²ΠΊΠ»ΡŽΡ‡Π΅Π½Π½Ρ‹Π΅ ΠΏΠ°Ρ€Π°Π»Π»Π΅Π»ΡŒΠ½ΠΎ, ΡΠΊΠ»Π°Π΄Ρ‹Π²Π°ΡŽΡ‚ΡΡ): .

    На рисункС Π±) – Ссли ΠΏΡ€ΠΈΠ³Π»ΡΠ΄Π΅Ρ‚ΡŒΡΡ, Ρ‚Π° ΠΆΠ΅ самая ситуация:

    К Π·Π°Π΄Π°Ρ‡Π΅ 5 – рисунок 4

    Π’Π°ΠΊ Ρ‡Ρ‚ΠΎ, Π°Π½Π°Π»ΠΎΠ³ΠΈΡ‡Π½ΠΎ ΠΏΠ΅Ρ€Π²ΠΎΠΉ схСмС, сопротивлСниС ΠΎΠ΄Π½ΠΎΠΉ Π²Π΅Ρ‚ΠΊΠΈ с двумя ΠΏΠΎΡΠ»Π΅Π΄ΠΎΠ²Π°Ρ‚Π΅Π»ΡŒΠ½ΠΎ Π²ΠΊΠ»ΡŽΡ‡Π΅Π½Π½Ρ‹ΠΌΠΈ кондСнсаторами – , Π° Π΄Π²Π΅ Ρ‚Π°ΠΊΠΈΠ΅ Смкости Π² ΠΏΠ°Ρ€Π°Π»Π»Π΅Π»ΡŒ Π΄Π°Π΄ΡƒΡ‚ .

    ΠžΡ‚Π²Π΅Ρ‚: Π°) ; Π±) .

    Π—Π°Π΄Π°Ρ‡Π° 6. ΠžΠΏΡ€Π΅Π΄Π΅Π»ΠΈΡ‚ΡŒ Π΅ΠΌΠΊΠΎΡΡ‚ΡŒ Π‘Ρ… бСсконСчно Π΄Π»ΠΈΠ½Π½ΠΎΠΉ систСмы ΠΎΠ΄ΠΈΠ½Π°ΠΊΠΎΠ²Ρ‹Ρ… кондСнсаторов, Π΅ΠΌΠΊΠΎΡΡ‚ΡŒΡŽ Π‘ ΠΊΠ°ΠΆΠ΄Ρ‹ΠΉ, соСдинСнных Π΄Ρ€ΡƒΠ³ с Π΄Ρ€ΡƒΠ³ΠΎΠΌ, ΠΊΠ°ΠΊ ΠΏΠΎΠΊΠ°Π·Π°Π½ΠΎ Π½Π° рисункС.

    К Π·Π°Π΄Π°Ρ‡Π΅ 6, рисунок 1

    Π’Ρ‹Π΄Π΅Π»ΠΈΠΌ Π² этой Ρ†Π΅ΠΏΠΈ ΠΏΠΎΠ²Ρ‚ΠΎΡ€ΡΡŽΡ‰ΠΈΠΉΡΡ элСмСнт:

    К Π·Π°Π΄Π°Ρ‡Π΅ 6, рисунок 2

    Π­Ρ‚ΠΈ элСмСнты соСдинСны ΠΏΠ°Ρ€Π°Π»Π»Π΅Π»ΡŒΠ½ΠΎ. Π’Π°ΠΊ ΠΊΠ°ΠΊ Π΅ΠΌΠΊΠΎΡΡ‚ΡŒ Ρ†Π΅ΠΏΠΈ бСсконСчна, Ρ‚ΠΎ ΠΎΡ‚ Π½Π΅Π΅ Π½Π΅ ΡƒΠ±ΡƒΠ΄Π΅Ρ‚, Ссли ΠΌΡ‹ ΠΎΠ΄ΠΈΠ½ элСмСнт ΡƒΠ΄Π°Π»ΠΈΠΌ, ΠΈΠ»ΠΈ Π²Ρ‹Π΄Π΅Π»ΠΈΠΌ. Π’ΠΎΠ³Π΄Π° справа ΠΎΡ‚ Π²Ρ‹Π΄Π΅Π»Π΅Π½Π½ΠΎΠ³ΠΎ элСмСнта Ρ†Π΅ΠΏΡŒ с Π΅ΠΌΠΊΠΎΡΡ‚ΡŒΡŽ , ΠΈ слСва – Ρ‚ΠΎΠΆΠ΅.

    К Π·Π°Π΄Π°Ρ‡Π΅ 6, рисунок 3

    МоТСм Π·Π°ΠΏΠΈΡΠ°Ρ‚ΡŒ для ΠΏΠΎΡΠ»Π΅Π΄ΠΎΠ²Π°Ρ‚Π΅Π»ΡŒΠ½ΠΎ Π²ΠΊΠ»ΡŽΡ‡Π΅Π½Π½Ρ‹Ρ… СмкостСй:

    Β  Β 

    Β  Β 

    Β  Β 

    Β  Β 

    Β  Β 

    Π—Π°Π΄Π°Ρ‡Π° 7. Найти Ρ€Π°Π·Π½ΠΎΡΡ‚ΡŒ ΠΏΠΎΡ‚Π΅Π½Ρ†ΠΈΠ°Π»ΠΎΠ² ΠΌΠ΅ΠΆΠ΄Ρƒ Ρ‚ΠΎΡ‡ΠΊΠ°ΠΌΠΈ А ΠΈ Π’ Π² схСмС, ΠΈΠ·ΠΎΠ±Ρ€Π°ΠΆΠ΅Π½Π½ΠΎΠΉ Π½Π° рисункС. Π•ΠΌΠΊΠΎΡΡ‚ΡŒ ΠΌΠΊΠ€, ΠΌΠΊΠ€, ΠΌΠΊΠ€. НапряТСниС источника Π’.

    К Π·Π°Π΄Π°Ρ‡Π΅ 7

    Π•ΠΌΠΊΠΎΡΡ‚ΡŒ Π²Π΅Ρ€Ρ…Π½Π΅ΠΉ Π²Π΅Ρ‚ΠΊΠΈ:

    Β  Β 

    Π•ΠΌΠΊΠΎΡΡ‚ΡŒ Π½ΠΈΠΆΠ½Π΅ΠΉ Π²Π΅Ρ‚ΠΊΠΈ:

    Β  Β 

    Заряд Π²Π΅Ρ€Ρ…Π½Π΅ΠΉ Π²Π΅Ρ‚Π²ΠΈ (мкКл):

    Β  Β 

    Заряд Π½ΠΈΠΆΠ½Π΅ΠΉ Π²Π΅Ρ‚Π²ΠΈ (мкКл):

    Β  Β 

    Но соСдинСн ΠΏΠΎΡΠ»Π΅Π΄ΠΎΠ²Π°Ρ‚Π΅Π»ΡŒΠ½ΠΎ с , поэтому , ΠΈ

    Β  Β 

    Β  Β 

    Аналогично Π² Π½ΠΈΠΆΠ½Π΅ΠΉ Π²Π΅Ρ‚Π²ΠΈ:

    Β  Β 

    Β  Β 

    Π’ суммС .

    НайдСм Ρ€Π°Π·Π½ΠΎΡΡ‚ΡŒ ΠΏΠΎΡ‚Π΅Π½Ρ†ΠΈΠ°Π»ΠΎΠ² ΠΌΠ΅ΠΆΠ΄Ρƒ Ρ‚ΠΎΡ‡ΠΊΠ°ΠΌΠΈ ΠΈ :

    Β  Β 

    ΠžΡ‚Π²Π΅Ρ‚: B.

    Β 

    По ΠΊΠ°ΠΊΠΎΠΉ Ρ„ΠΎΡ€ΠΌΡƒΠ»Π΅ Π½Π°ΠΉΡ‚ΠΈ Ρ‘ΠΌΠΊΠΎΡΡ‚ΡŒ (объСм) кондСнсаторов

    Π’ΠΎ всСх элСктронных устройствах ΠΈΡΠΏΠΎΠ»ΡŒΠ·ΡƒΡŽΡ‚ΡΡ кондСнсаторы. ΠŸΡ€ΠΈ ΠΈΡ… конструировании ΠΈΠ»ΠΈ ΠΈΠ·Π³ΠΎΡ‚ΠΎΠ²Π»Π΅Π½ΠΈΠΈ своими Ρ€ΡƒΠΊΠ°ΠΌΠΈ ΠΏΠ°Ρ€Π°ΠΌΠ΅Ρ‚Ρ€Ρ‹ устройств Ρ€Π°ΡΡΡ‡ΠΈΡ‚Ρ‹Π²Π°ΡŽΡ‚ΡΡ ΠΏΠΎ ΡΠΏΠ΅Ρ†ΠΈΠ°Π»ΡŒΠ½Ρ‹ΠΌ Ρ„ΠΎΡ€ΠΌΡƒΠ»Π°ΠΌ.

    ΠšΠΎΠ½Π΄Π΅Π½ΡΠ°Ρ‚ΠΎΡ€Ρ‹

    Расчёт кондСнсаторов

    Один ΠΈΠ· Π³Π»Π°Π²Π½Ρ‹Ρ… ΠΏΠ°Ρ€Π°ΠΌΠ΅Ρ‚Ρ€ΠΎΠ² Ρ‚Π°ΠΊΠΈΡ… устройств – Ρ‘ΠΌΠΊΠΎΡΡ‚ΡŒ. Π Π°ΡΡΡ‡ΠΈΡ‚Π°Ρ‚ΡŒ Π΅Ρ‘ ΠΌΠΎΠΆΠ½ΠΎ ΠΏΠΎ ΡΠ»Π΅Π΄ΡƒΡŽΡ‰Π΅ΠΉ Ρ„ΠΎΡ€ΠΌΡƒΠ»Π΅:

    C=q/U, Π³Π΄Π΅:

    • C – Ρ‘ΠΌΠΊΠΎΡΡ‚ΡŒ,
    • q – заряд ΠΎΠ΄Π½ΠΎΠΉ ΠΈΠ· ΠΎΠ±ΠΊΠ»Π°Π΄ΠΎΠΊ элСмСнта,
    • U – Ρ€Π°Π·Π½ΠΎΡΡ‚ΡŒ ΠΏΠΎΡ‚Π΅Π½Ρ†ΠΈΠ°Π»ΠΎΠ² ΠΌΠ΅ΠΆΠ΄Ρƒ ΠΎΠ±ΠΊΠ»Π°Π΄ΠΊΠ°ΠΌΠΈ.

    Π’ элСктротСхникС вмСсто понятия Β«Ρ€Π°Π·Π½ΠΎΡΡ‚ΡŒ ΠΏΠΎΡ‚Π΅Π½Ρ†ΠΈΠ°Π»ΠΎΠ² ΠΌΠ΅ΠΆΠ΄Ρƒ ΠΎΠ±ΠΊΠ»Π°Π΄ΠΊΠ°ΠΌΠΈΒ» ΠΈΡΠΏΠΎΠ»ΡŒΠ·ΡƒΠ΅Ρ‚ΡΡ «напряТСниС Π½Π° кондСнсаторС».

    ΠΠΌΠΊΠΎΡΡ‚ΡŒ элСмСнта Π½Π΅ зависит ΠΎΡ‚ конструкции ΠΈ Ρ€Π°Π·ΠΌΠ΅Ρ€ΠΎΠ² устройства, Π° Ρ‚ΠΎΠ»ΡŒΠΊΠΎ ΠΎΡ‚ напряТСния Π½Π° Π½Ρ‘ΠΌ ΠΈ заряда ΠΎΠ±ΠΊΠ»Π°Π΄ΠΎΠΊ. Но эти ΠΏΠ°Ρ€Π°ΠΌΠ΅Ρ‚Ρ€Ρ‹ ΠΌΠΎΠ³ΡƒΡ‚ ΠΈΠ·ΠΌΠ΅Π½ΡΡ‚ΡŒΡΡ Π² зависимости ΠΎΡ‚ расстояния ΠΌΠ΅ΠΆΠ΄Ρƒ Π½ΠΈΠΌΠΈ ΠΈ ΠΌΠ°Ρ‚Π΅Ρ€ΠΈΠ°Π»Π° диэлСктрика. Π­Ρ‚ΠΎ учитываСтся Π² Ρ„ΠΎΡ€ΠΌΡƒΠ»Π΅:

    Π‘=Co*Ξ΅, Π³Π΄Π΅:

    • Π‘ – Ρ€Π΅Π°Π»ΡŒΠ½Π°Ρ Ρ‘ΠΌΠΊΠΎΡΡ‚ΡŒ,
    • Π‘ΠΎ – идСальная, ΠΏΡ€ΠΈ условии, Ρ‡Ρ‚ΠΎ ΠΌΠ΅ΠΆΠ΄Ρƒ пластинами Π²Π°ΠΊΡƒΡƒΠΌ ΠΈΠ»ΠΈ Π²ΠΎΠ·Π΄ΡƒΡ…,
    • Ξ΅ – диэлСктричСская ΠΏΡ€ΠΎΠ½ΠΈΡ†Π°Π΅ΠΌΠΎΡΡ‚ΡŒ ΠΌΠ°Ρ‚Π΅Ρ€ΠΈΠ°Π»Π° ΠΌΠ΅ΠΆΠ΄Ρƒ Π½ΠΈΠΌΠΈ.

    НапримСр, Ссли Π² качСствС диэлСктрика ΠΈΡΠΏΠΎΠ»ΡŒΠ·ΡƒΠ΅Ρ‚ΡΡ слюда, «Ρ» ΠΊΠΎΡ‚ΠΎΡ€ΠΎΠΉ 6, Ρ‚ΠΎ Ρ‘ΠΌΠΊΠΎΡΡ‚ΡŒ Ρ‚Π°ΠΊΠΎΠ³ΠΎ устройства Π² 6 Ρ€Π°Π· большС, Ρ‡Π΅ΠΌ Π²ΠΎΠ·Π΄ΡƒΡˆΠ½ΠΎΠ³ΠΎ, Π° ΠΏΡ€ΠΈ ΠΈΠ·ΠΌΠ΅Π½Π΅Π½ΠΈΠΈ количСства диэлСктрика ΠΌΠ΅Π½ΡΡŽΡ‚ΡΡ ΠΏΠ°Ρ€Π°ΠΌΠ΅Ρ‚Ρ€Ρ‹ конструкции. На этом ΠΏΡ€ΠΈΠ½Ρ†ΠΈΠΏΠ΅ основана Ρ€Π°Π±ΠΎΡ‚Π° ёмкостного Π΄Π°Ρ‚Ρ‡ΠΈΠΊΠ° полоТСния.

    Устройство кондСнсатора

    Π•Π΄ΠΈΠ½ΠΈΡ†Π΅ΠΉ ёмкости Π² систСмС БИ являСтся 1 Ρ„Π°Ρ€Π°Π΄ (F). Π­Ρ‚ΠΎ большая Π²Π΅Π»ΠΈΡ‡ΠΈΠ½Π°, поэтому Ρ‡Π°Ρ‰Π΅ ΠΏΡ€ΠΈΠΌΠ΅Π½ΡΡŽΡ‚ΡΡ ΠΌΠΈΠΊΡ€ΠΎΡ„Π°Ρ€Π°Π΄Ρ‹ (1000000mkF=1F) ΠΈ ΠΏΠΈΠΊΠΎΡ„Π°Ρ€Π°Π΄Ρ‹ (1000000pF=1mkF).

    РасчСт плоской конструкции

    Если Π½ΡƒΠΆΠ½ΠΎ Ρ€Π°ΡΡΡ‡ΠΈΡ‚Π°Ρ‚ΡŒ плоский кондСнсатор, Ρ‚ΠΎ Π½Π΅ΠΎΠ±Ρ…ΠΎΠ΄ΠΈΠΌΠΎ ΡƒΡ‡Π΅ΡΡ‚ΡŒ ΠΏΠ»ΠΎΡ‰Π°Π΄ΡŒ ΠΎΠ±ΠΊΠ»Π°Π΄ΠΎΠΊ ΠΈ расстояниС ΠΌΠ΅ΠΆΠ΄Ρƒ Π½ΠΈΠΌΠΈ. Π­Ρ‚ΠΎ ΠΎΡ‚Ρ€Π°ΠΆΠ΅Π½ΠΎ Π² Ρ„ΠΎΡ€ΠΌΡƒΠ»Π΅, ΠΏΠΎ ΠΊΠΎΡ‚ΠΎΡ€ΠΎΠΉ рассчитываСтся Ρ‘ΠΌΠΊΠΎΡΡ‚ΡŒ плоского кондСнсатора:

    C=Ξ΅/d, Π³Π΄Π΅:

    • Ξ΅ – диэлСктричСская ΠΏΡ€ΠΎΠ½ΠΈΡ†Π°Π΅ΠΌΠΎΡΡ‚ΡŒ ΠΈΠ·ΠΎΠ»ΠΈΡ€ΡƒΡŽΡ‰Π΅Π³ΠΎ ΠΌΠ°Ρ‚Π΅Ρ€ΠΈΠ°Π»Π°,
    • d – расстояниС ΠΌΠ΅ΠΆΠ΄Ρƒ пластинами.

    РасчСт конструкции цилиндричСской Ρ„ΠΎΡ€ΠΌΡ‹

    ЦилиндричСский кондСнсатор – это Π΄Π²Π΅ соосныС Ρ‚Ρ€ΡƒΠ±ΠΊΠΈ Ρ€Π°Π·Π»ΠΈΡ‡Π½ΠΎΠ³ΠΎ Π΄ΠΈΠ°ΠΌΠ΅Ρ‚Ρ€Π°, вставлСнныС Π΄Ρ€ΡƒΠ³ Π² Π΄Ρ€ΡƒΠ³Π°. ΠœΠ΅ΠΆΠ΄Ρƒ Π½ΠΈΠΌΠΈ находится диэлСктрик. ΠŸΡ€ΠΈ радиусС Ρ†ΠΈΠ»ΠΈΠ½Π΄Ρ€ΠΎΠ², Π±Π»ΠΈΠ·ΠΊΠΎΠΌ Π΄Ρ€ΡƒΠ³ ΠΊ Π΄Ρ€ΡƒΠ³Ρƒ ΠΈ Π½Π°ΠΌΠ½ΠΎΠ³ΠΎ большСм, Ρ‡Π΅ΠΌ расстояниС ΠΌΠ΅ΠΆΠ΄Ρƒ Π½ΠΈΠΌΠΈ, цилиндричСской Ρ„ΠΎΡ€ΠΌΠΎΠΉ ΠΌΠΎΠΆΠ½ΠΎ ΠΏΡ€Π΅Π½Π΅Π±Ρ€Π΅Ρ‡ΡŒ ΠΈ свСсти расчёт ΠΊ Ρ„ΠΎΡ€ΠΌΡƒΠ»Π΅, Π°Π½Π°Π»ΠΎΠ³ΠΈΡ‡Π½ΠΎΠΉ Ρ‚ΠΎΠΉ, ΠΏΠΎ ΠΊΠΎΡ‚ΠΎΡ€ΠΎΠΉ рассчитываСтся плоский кондСнсатор.

    Π’Ρ‹Ρ‡ΠΈΡΠ»ΡΡŽΡ‚ΡΡ ΠΏΠ°Ρ€Π°ΠΌΠ΅Ρ‚Ρ€Ρ‹ Ρ‚Π°ΠΊΠΎΠ³ΠΎ устройства ΠΏΠΎ Ρ„ΠΎΡ€ΠΌΡƒΠ»Π΅:

    C=(2Ο€*l*R*Ξ΅)/d, Π³Π΄Π΅:

    • l – Π΄Π»ΠΈΠ½Π° устройства,
    • R – радиус Ρ†ΠΈΠ»ΠΈΠ½Π΄Ρ€Π°,
    • Ξ΅ – диэлСктричСская ΠΏΡ€ΠΎΠ½ΠΈΡ†Π°Π΅ΠΌΠΎΡΡ‚ΡŒ изолятора,
    • d – Π΅Π³ΠΎ Ρ‚ΠΎΠ»Ρ‰ΠΈΠ½Π°.

    Расчёт сфСричСской конструкции

    Π•ΡΡ‚ΡŒ устройства, ΠΎΠ±ΠΊΠ»Π°Π΄ΠΊΠΈ ΠΊΠΎΡ‚ΠΎΡ€Ρ‹Ρ… ΠΏΡ€Π΅Π΄ΡΡ‚Π°Π²Π»ΡΡŽΡ‚ собой Π΄Π²Π° ΡˆΠ°Ρ€Π°, Π²Π»ΠΎΠΆΠ΅Π½Π½Ρ‹Π΅ Π΄Ρ€ΡƒΠ³ Π² Π΄Ρ€ΡƒΠ³Π°. Π€ΠΎΡ€ΠΌΡƒΠ»Π° ёмкости Ρ‚Π°ΠΊΠΎΠ³ΠΎ ΠΏΡ€ΠΈΠ±ΠΎΡ€Π°:

    C=(4Ο€*l*R1*R2*Ξ΅)/(R2-R1), Π³Π΄Π΅:

    • R1 – радиус Π²Π½ΡƒΡ‚Ρ€Π΅Π½Π½Π΅ΠΉ сфСры,
    • R2 – радиус внСшнСй сфСры,
    • Ξ΅ – диэлСктричСская ΠΏΡ€ΠΎΠ½ΠΈΡ†Π°Π΅ΠΌΠΎΡΡ‚ΡŒ.

    Π€ΠΎΡ€ΠΌΡƒΠ»Ρ‹ ёмкости кондСнсаторов Ρ€Π°Π·Π»ΠΈΡ‡Π½ΠΎΠΉ Ρ„ΠΎΡ€ΠΌΡ‹

    ΠΠΌΠΊΠΎΡΡ‚ΡŒ ΠΎΠ΄ΠΈΠ½ΠΎΡ‡Π½ΠΎΠ³ΠΎ ΠΏΡ€ΠΎΠ²ΠΎΠ΄Π½ΠΈΠΊΠ°

    ΠšΡ€ΠΎΠΌΠ΅ кондСнсаторов, ΡΠΏΠΎΡΠΎΠ±Π½ΠΎΡΡ‚ΡŒΡŽ Π½Π°ΠΊΠ°ΠΏΠ»ΠΈΠ²Π°Ρ‚ΡŒ заряд ΠΎΠ±Π»Π°Π΄Π°ΡŽΡ‚ ΠΎΡ‚Π΄Π΅Π»ΡŒΠ½Ρ‹Π΅ ΠΏΡ€ΠΎΠ²ΠΎΠ΄Π½ΠΈΠΊΠΈ. ΠžΠ΄ΠΈΠ½ΠΎΡ‡Π½Ρ‹ΠΌ ΠΏΡ€ΠΎΠ²ΠΎΠ΄Π½ΠΈΠΊΠΎΠΌ считаСтся Ρ‚Π°ΠΊΠΎΠΉ ΠΏΡ€ΠΎΠ²ΠΎΠ΄Π½ΠΈΠΊ, ΠΊΠΎΡ‚ΠΎΡ€Ρ‹ΠΉ бСсконСчно Π΄Π°Π»Ρ‘ΠΊ ΠΎΡ‚ Π΄Ρ€ΡƒΠ³ΠΈΡ… ΠΏΡ€ΠΎΠ²ΠΎΠ΄Π½ΠΈΠΊΠΎΠ². ΠŸΠ°Ρ€Π°ΠΌΠ΅Ρ‚Ρ€Ρ‹ заряТСнного элСмСнта рассчитываСтся ΠΏΠΎ Ρ„ΠΎΡ€ΠΌΡƒΠ»Π΅:

    C=Q/Ο†, Π³Π΄Π΅:

    • Q – заряд,
    • φ – ΠΏΠΎΡ‚Π΅Π½Ρ†ΠΈΠ°Π» ΠΏΡ€ΠΎΠ²ΠΎΠ΄Π½ΠΈΠΊΠ°.

    ΠžΠ±ΡŠΡ‘ΠΌ заряда опрСдСляСтся Ρ€Π°Π·ΠΌΠ΅Ρ€ΠΎΠΌ ΠΈ Ρ„ΠΎΡ€ΠΌΠΎΠΉ устройства, Π° Ρ‚Π°ΠΊΠΆΠ΅ ΠΎΠΊΡ€ΡƒΠΆΠ°ΡŽΡ‰Π΅ΠΉ срСдой. ΠœΠ°Ρ‚Π΅Ρ€ΠΈΠ°Π» ΠΏΡ€ΠΈΠ±ΠΎΡ€Π° значСния Π½Π΅ ΠΈΠΌΠ΅Π΅Ρ‚.

    Бпособы соСдинСния элСмСнтов

    НС всСгда Π΅ΡΡ‚ΡŒ Π² Π½Π°Π»ΠΈΡ‡ΠΈΠΈ элСмСнты с Π½Π΅ΠΎΠ±Ρ…ΠΎΠ΄ΠΈΠΌΡ‹ΠΌΠΈ ΠΏΠ°Ρ€Π°ΠΌΠ΅Ρ‚Ρ€Π°ΠΌΠΈ. ΠŸΡ€ΠΈΡ…ΠΎΠ΄ΠΈΡ‚ΡΡ ΡΠΎΠ΅Π΄ΠΈΠ½ΡΡ‚ΡŒ ΠΈΡ… Ρ€Π°Π·Π»ΠΈΡ‡Π½Ρ‹ΠΌΠΈ способами.

    Π‘ΠΎΠ΅Π΄ΠΈΠ½Π΅Π½ΠΈΠ΅ кондСнсаторов

    ΠŸΠ°Ρ€Π°Π»Π»Π΅Π»ΡŒΠ½ΠΎΠ΅ соСдинСниС

    Π­Ρ‚ΠΎ Ρ‚Π°ΠΊΠΎΠ΅ соСдинСниС Π΄Π΅Ρ‚Π°Π»Π΅ΠΉ, ΠΏΡ€ΠΈ ΠΊΠΎΡ‚ΠΎΡ€ΠΎΠΌ ΠΊ ΠΎΠ΄Π½ΠΎΠΉ ΠΊΠ»Π΅ΠΌΠΌΠ΅ ΠΈΠ»ΠΈ ΠΊΠΎΠ½Ρ‚Π°ΠΊΡ‚Ρƒ ΠΏΡ€ΠΈΡΠΎΠ΅Π΄ΠΈΠ½ΡΡŽΡ‚ΡΡ ΠΏΠ΅Ρ€Π²Ρ‹Π΅ ΠΎΠ±ΠΊΠ»Π°Π΄ΠΊΠΈ ΠΊΠ°ΠΆΠ΄ΠΎΠ³ΠΎ кондСнсатора. ΠŸΡ€ΠΈ этом Π²Ρ‚ΠΎΡ€Ρ‹Π΅ ΠΎΠ±ΠΊΠ»Π°Π΄ΠΊΠΈ ΠΏΡ€ΠΈΡΠΎΠ΅Π΄ΠΈΠ½ΡΡŽΡ‚ΡΡ ΠΊ Π΄Ρ€ΡƒΠ³ΠΎΠΉ ΠΊΠ»Π΅ΠΌΠΌΠ΅.

    ΠŸΡ€ΠΈ Ρ‚Π°ΠΊΠΎΠΌ соСдинСнии напряТСниС Π½Π° ΠΊΠΎΠ½Ρ‚Π°ΠΊΡ‚Π°Ρ… всСх элСмСнтов Π±ΡƒΠ΄Π΅Ρ‚ ΠΎΠ΄ΠΈΠ½Π°ΠΊΠΎΠ²Ρ‹ΠΌ. Заряд ΠΊΠ°ΠΆΠ΄ΠΎΠ³ΠΎ ΠΈΠ· Π½ΠΈΡ… происходит нСзависимо ΠΎΡ‚ ΠΎΡΡ‚Π°Π»ΡŒΠ½Ρ‹Ρ…, поэтому общая Ρ‘ΠΌΠΊΠΎΡΡ‚ΡŒ Ρ€Π°Π²Π½Π° суммС всСх Π²Π΅Π»ΠΈΡ‡ΠΈΠ½. Π•Ρ‘ находят ΠΏΠΎ Ρ„ΠΎΡ€ΠΌΡƒΠ»Π΅:

    C=C1+C2+…Cn,

    Π³Π΄Π΅ C1-Cn – ΠΏΠ°Ρ€Π°ΠΌΠ΅Ρ‚Ρ€Ρ‹ Π΄Π΅Ρ‚Π°Π»Π΅ΠΉ, ΡƒΡ‡Π°ΡΡ‚Π²ΡƒΡŽΡ‰ΠΈΡ… Π² ΠΏΠ°Ρ€Π°Π»Π»Π΅Π»ΡŒΠ½ΠΎΠΌ соСдинСнии.

    Π’Π°ΠΆΠ½ΠΎ! ΠšΠΎΠ½Π΄Π΅Π½ΡΠ°Ρ‚ΠΎΡ€Ρ‹ ΠΈΠΌΠ΅ΡŽΡ‚ ΠΏΡ€Π΅Π΄Π΅Π»ΡŒΠ½ΠΎΠ΅ допустимоС напряТСниС, ΠΏΡ€Π΅Π²Ρ‹ΡˆΠ΅Π½ΠΈΠ΅ ΠΊΠΎΡ‚ΠΎΡ€ΠΎΠ³ΠΎ ΠΏΡ€ΠΈΠ²Π΅Π΄Ρ‘Ρ‚ ΠΊ Π²Ρ‹Ρ…ΠΎΠ΄Ρƒ элСмСнта ΠΈΠ· строя. ΠŸΡ€ΠΈ ΠΏΠ°Ρ€Π°Π»Π»Π΅Π»ΡŒΠ½ΠΎΠΌ соСдинСнии устройств с Ρ€Π°Π·Π»ΠΈΡ‡Π½Ρ‹ΠΌ допустимым напряТСниСм этот ΠΏΠ°Ρ€Π°ΠΌΠ΅Ρ‚Ρ€ ΠΏΠΎΠ»ΡƒΡ‡ΠΈΠ²ΡˆΠ΅ΠΉΡΡ сборки Ρ€Π°Π²Π΅Π½ элСмСнту с наимСньшим Π·Π½Π°Ρ‡Π΅Π½ΠΈΠ΅ΠΌ.

    ΠŸΠΎΡΠ»Π΅Π΄ΠΎΠ²Π°Ρ‚Π΅Π»ΡŒΠ½ΠΎΠ΅ соСдинСниС

    Π­Ρ‚ΠΎ Ρ‚Π°ΠΊΠΎΠ΅ соСдинСниС, ΠΏΡ€ΠΈ ΠΊΠΎΡ‚ΠΎΡ€ΠΎΠΌ ΠΊ ΠΊΠ»Π΅ΠΌΠΌΠ΅ присоСдиняСтся Ρ‚ΠΎΠ»ΡŒΠΊΠΎ ΠΎΠ΄Π½Π° пластина ΠΏΠ΅Ρ€Π²ΠΎΠ³ΠΎ элСмСнта. Вторая пластина присоСдиняСтся ΠΊ ΠΏΠ΅Ρ€Π²ΠΎΠΉ пластинС Π²Ρ‚ΠΎΡ€ΠΎΠ³ΠΎ элСмСнта, вторая пластина Π²Ρ‚ΠΎΡ€ΠΎΠ³ΠΎ – ΠΊ ΠΏΠ΅Ρ€Π²ΠΎΠΉ пластинС Ρ‚Ρ€Π΅Ρ‚ΡŒΠ΅Π³ΠΎ ΠΈ Ρ‚Π°ΠΊ Π΄Π°Π»Π΅Π΅. Ко Π²Ρ‚ΠΎΡ€ΠΎΠΉ ΠΊΠ»Π΅ΠΌΠΌΠ΅ присоСдиняСтся Ρ‚ΠΎΠ»ΡŒΠΊΠΎ вторая ΠΎΠ±ΠΊΠ»Π°Π΄ΠΊΠ° послСднСго элСмСнта.

    ΠŸΡ€ΠΈ Ρ‚Π°ΠΊΠΎΠΌ соСдинСнии заряд Π½Π° ΠΎΠ±ΠΊΠ»Π°Π΄ΠΊΠ°Ρ… кондСнсатора Π² ΠΊΠ°ΠΆΠ΄ΠΎΠΌ ΠΏΡ€ΠΈΠ±ΠΎΡ€Π΅ Π±ΡƒΠ΄Π΅Ρ‚ Ρ€Π°Π²Π΅Π½ ΠΎΡΡ‚Π°Π»ΡŒΠ½Ρ‹ΠΌ, ΠΎΠ΄Π½Π°ΠΊΠΎ напряТСниС Π½Π° Π½ΠΈΡ… Π±ΡƒΠ΄Π΅Ρ‚ Ρ€Π°Π·Π½Ρ‹ΠΌ: для зарядки устройств большСй ёмкости Ρ‚Π΅ΠΌ ΠΆΠ΅ зарядом трСбуСтся мСньшая Ρ€Π°Π·Π½ΠΎΡΡ‚ΡŒ ΠΏΠΎΡ‚Π΅Π½Ρ†ΠΈΠ°Π»ΠΎΠ². ΠŸΠΎΡΡ‚ΠΎΠΌΡƒ вся Ρ†Π΅ΠΏΠΎΡ‡ΠΊΠ° прСдставляСт собой ΠΎΠ΄Π½Ρƒ ΠΊΠΎΠ½ΡΡ‚Ρ€ΡƒΠΊΡ†ΠΈΡŽ, Ρ€Π°Π·Π½ΠΎΡΡ‚ΡŒ ΠΏΠΎΡ‚Π΅Π½Ρ†ΠΈΠ°Π»ΠΎΠ² ΠΊΠΎΡ‚ΠΎΡ€ΠΎΠΉ Ρ€Π°Π²Π½Π° суммС напряТСний Π½Π° всСх элСмСнтах, Π° заряд кондСнсатора Ρ€Π°Π²Π΅Π½ суммС зарядов.

    ΠŸΠΎΡΠ»Π΅Π΄ΠΎΠ²Π°Ρ‚Π΅Π»ΡŒΠ½ΠΎΠ΅ соСдинСниС ΡƒΠ²Π΅Π»ΠΈΡ‡ΠΈΠ²Π°Π΅Ρ‚ допустимоС напряТСниС ΠΈ ΡƒΠΌΠ΅Π½ΡŒΡˆΠ°Π΅Ρ‚ ΠΎΠ±Ρ‰ΡƒΡŽ Ρ‘ΠΌΠΊΠΎΡΡ‚ΡŒ, которая мСньшС самого мСньшСго элСмСнта.

    Π Π°ΡΡΡ‡ΠΈΡ‚Ρ‹Π²Π°ΡŽΡ‚ΡΡ эти ΠΏΠ°Ρ€Π°ΠΌΠ΅Ρ‚Ρ€Ρ‹ ΡΠ»Π΅Π΄ΡƒΡŽΡ‰ΠΈΠΌ ΠΎΠ±Ρ€Π°Π·ΠΎΠΌ:

    • ДопустимоС напряТСниС:

    UΠΎΠ±Ρ‰=U1+U2+U3+…Un, Π³Π΄Π΅ U1-Un – напряТСниС Π½Π° кондСнсаторС;

    • ΠžΠ±Ρ‰Π°Ρ Ρ‘ΠΌΠΊΠΎΡΡ‚ΡŒ:

    1/Π‘ΠΎΠ±Ρ‰=1/Π‘1+1/Π‘2+1/Π‘3+…1/Π‘n, Π³Π΄Π΅ Π‘1-Π‘n – ΠΏΠ°Ρ€Π°ΠΌΠ΅Ρ‚Ρ€Ρ‹ ΠΊΠ°ΠΆΠ΄ΠΎΠ³ΠΎ устройства.

    Π˜Π½Ρ‚Π΅Ρ€Π΅ΡΠ½ΠΎ. Если Π² Ρ†Π΅ΠΏΠΈ Ρ‚ΠΎΠ»ΡŒΠΊΠΎ Π΄Π²Π° элСмСнта, Ρ‚ΠΎ ΠΌΠΎΠΆΠ½ΠΎ Π²ΠΎΡΠΏΠΎΠ»ΡŒΠ·ΠΎΠ²Π°Ρ‚ΡŒΡΡ ΡƒΠΏΡ€ΠΎΡ‰Ρ‘Π½Π½ΠΎΠΉ Ρ„ΠΎΡ€ΠΌΡƒΠ»ΠΎΠΉ: Π‘ΠΎΠ±Ρ‰=(Π‘1*Π‘2)/(Π‘1+Π‘2).

    БмСшанноС соСдинСниС

    Π­Ρ‚ΠΎ Ρ‚Π°ΠΊΠΎΠ΅ соСдинСниС, Π² ΠΊΠΎΡ‚ΠΎΡ€ΠΎΠΌ Π΅ΡΡ‚ΡŒ Π΄Π΅Ρ‚Π°Π»ΠΈ, соСдинённыС ΠΏΠΎΡΠ»Π΅Π΄ΠΎΠ²Π°Ρ‚Π΅Π»ΡŒΠ½ΠΎ, ΠΈ Π΅ΡΡ‚ΡŒ соСдинённыС ΠΏΠ°Ρ€Π°Π»Π»Π΅Π»ΡŒΠ½ΠΎ. ΠŸΠ°Ρ€Π°ΠΌΠ΅Ρ‚Ρ€Ρ‹ всСй Ρ†Π΅ΠΏΠΈ рассчитываСтся Π² ΡΠ»Π΅Π΄ΡƒΡŽΡ‰Π΅ΠΉ ΠΏΠΎΡΠ»Π΅Π΄ΠΎΠ²Π°Ρ‚Π΅Π»ΡŒΠ½ΠΎΡΡ‚ΠΈ:

    1. ΠΎΠΏΡ€Π΅Π΄Π΅Π»ΡΡŽΡ‚ΡΡ Π³Ρ€ΡƒΠΏΠΏΡ‹ элСмСнтов, соСдинённыС ΠΏΠ°Ρ€Π°Π»Π»Π΅Π»ΡŒΠ½ΠΎ;
    2. для ΠΊΠ°ΠΆΠ΄ΠΎΠΉ Π³Ρ€ΡƒΠΏΠΏΡ‹ Π² ΠΎΡ‚Π΄Π΅Π»ΡŒΠ½ΠΎΡΡ‚ΠΈ рассчитываСтся эквивалСнтныС значСния;
    3. рядом с ΠΊΠ°ΠΆΠ΄ΠΎΠΉ Π³Ρ€ΡƒΠΏΠΏΠΎΠΉ ΠΏΠ°Ρ€Π°Π»Π»Π΅Π»ΡŒΠ½ΠΎ соСдинённых Π΄Π΅Ρ‚Π°Π»Π΅ΠΉ ΠΏΠΈΡˆΡƒΡ‚ΡΡ ΠΏΠΎΠ»ΡƒΡ‡ΠΈΠ²ΡˆΠΈΠ΅ΡΡ Π²Π΅Π»ΠΈΡ‡ΠΈΠ½Ρ‹;
    4. ΠΏΠΎΠ»ΡƒΡ‡ΠΈΠ²ΡˆΠ°ΡΡΡ схСма эквивалСнтна ΠΏΠΎΡΠ»Π΅Π΄ΠΎΠ²Π°Ρ‚Π΅Π»ΡŒΠ½ΠΎΠΉ схСмС ΠΈ рассчитываСтся ΠΏΠΎ ΡΠΎΠΎΡ‚Π²Π΅Ρ‚ΡΡ‚Π²ΡƒΡŽΡ‰ΠΈΠΌ Ρ„ΠΎΡ€ΠΌΡƒΠ»Π°ΠΌ.

    Π—Π½Π°Π½ΠΈΠ΅ Ρ„ΠΎΡ€ΠΌΡƒΠ», ΠΏΠΎ ΠΊΠΎΡ‚ΠΎΡ€Ρ‹ΠΌ ΠΌΠΎΠΆΠ½ΠΎ Π½Π°ΠΉΡ‚ΠΈ Π΅ΠΌΠΊΠΎΡΡ‚ΡŒΒ ΠΏΡ€ΠΈ ΠΈΠ·Π³ΠΎΡ‚ΠΎΠ²Π»Π΅Π½ΠΈΠΈ кондСнсаторов ΠΈΠ»ΠΈ ΠΈΡ… соСдинСнии Π½Π΅ΠΎΠ±Ρ…ΠΎΠ΄ΠΈΠΌΠΎ ΠΏΡ€ΠΈ конструировании элСктронных схСм.

    Π’ΠΈΠ΄Π΅ΠΎ

    ΠžΡ†Π΅Π½ΠΈΡ‚Π΅ ΡΡ‚Π°Ρ‚ΡŒΡŽ:

    БмСшанноС соСдинСниС кондСнсаторов

    ВсС Π²Π½ΡƒΡ‚Ρ€Π΅Π½Π½ΠΈΠ΅ ΠΎΠ±ΠΊΠ»Π°Π΄ΠΊΠΈ ΠΏΡ€ΠΈ ΠΏΠΎΡΠ»Π΅Π΄ΠΎΠ²Π°Ρ‚Π΅Π»ΡŒΠ½ΠΎΠΌ соСдинСнии ΡΠ»Π΅ΠΊΡ‚Ρ€ΠΈΠ·ΡƒΡŽΡ‚ΡΡ Ρ‡Π΅Ρ€Π΅Π· влияниС. Π˜Ρ… заряды Ρ€Π°Π²Π½Ρ‹

    ΠΏΠΎ Π²Π΅Π»ΠΈΡ‡ΠΈΠ½Π΅, Π½ΠΎ ΠΏΡ€ΠΎΡ‚ΠΈΠ²ΠΎΠΏΠΎΠ»ΠΎΠΆΠ½Ρ‹ ΠΏΠΎ Π·Π½Π°ΠΊΡƒ (Β½+qΒ½=Β½-qΒ½ = q; рис. 12).

    Π‘Π»Π΅Π΄ΠΎΠ²Π°Ρ‚Π΅Π»ΡŒΠ½ΠΎ, заряды Π½Π° всСх кондСнсаторах ΠΏΡ€ΠΈ ΠΏΠΎΡΠ»Π΅Π΄ΠΎΠ²Π°Ρ‚Π΅Π»ΡŒΠ½ΠΎΠΌ ΠΈΡ… соСдинСнии Ρ€Π°Π²Π½Ρ‹, Π° ΠΏΠΎΡ‚Π΅Π½Ρ†ΠΈΠ°Π»Ρ‹ ΡΠΊΠ»Π°Π΄Ρ‹Π²Π°ΡŽΡ‚ΡΡ,

    Dj = j 1 — j 2 = Dj 1 + Dj 2 + … + Dj n ,

    Π³Π΄Π΅ .

    Π‘Π»Π΅Π΄ΠΎΠ²Π°Ρ‚Π΅Π»ΡŒΠ½ΠΎ, . (17)

    ΠŸΠ°Ρ€Π°Π»Π»Π΅Π»ΡŒΠ½ΠΎΠ΅ соСдинСниС кондСнсаторов

    Рис. 13

    ΠŸΡ€ΠΈ ΠΏΠ°Ρ€Π°Π»Π»Π΅Π»ΡŒΠ½ΠΎΠΌ соСдинСнии всС кондСнсаторы ΠΈΠΌΠ΅ΡŽΡ‚ ΠΏΠΎΡΡ‚ΠΎΡΠ½Π½ΡƒΡŽ Ρ€Π°Π·Π½ΠΎΡΡ‚ΡŒ ΠΏΠΎΡ‚Π΅Π½Ρ†ΠΈΠ°Π»ΠΎΠ²

    j 1 — j 2 = сonst. ΠŸΠΎΠ»Π½Ρ‹ΠΉ заряд Π±Π°Ρ‚Π°Ρ€Π΅ΠΈ кондСнсаторов (рис. 1.31): q = q 1 + q 2 +…+ q n

    По ΠΎΠΏΡ€Π΅Π΄Π΅Π»Π΅Π½ΠΈΡŽ Π΅ΠΌΠΊΠΎΡΡ‚ΡŒ Π±Π°Ρ‚Π°Ρ€Π΅ΠΈ кондСнсаторов ,

    Π‘Π»Π΅Π΄ΠΎΠ²Π°Ρ‚Π΅Π»ΡŒΠ½ΠΎ,

    Π‘ = Π‘ 1 + Π‘ 2 + … + Π‘ n . (18)

    ЭнСргия элСктричСского поля

    ЭнСргия взаимодСйствия элСктричСских зарядов

    Π˜Π·Π²Π΅ΡΡ‚Π½ΠΎ, Ρ‡Ρ‚ΠΎ dW 12 = — dA 12 . Для систСмы ΠΈΠ· Ρ‚Ρ€Π΅Ρ… зарядов

    dW = — d(W 12 + W 13 + W 23)= — dA,

    W = W 12 + W 13 + W 23 . (19)

    Π­Ρ‚ΠΎ ΠΏΠΎΠ»ΠΎΠΆΠ΅Π½ΠΈΠ΅ остаСтся справСдливым ΠΈ для ΠΏΡ€ΠΎΠΈΠ·Π²ΠΎΠ»ΡŒΠ½ΠΎΠΉ систСмы Ρ‚ΠΎΡ‡Π΅Ρ‡Π½Ρ‹Ρ… зарядов. Для нахоТдСния энСргии взаимодСйствия систСмы N Ρ‚ΠΎΡ‡Π΅Ρ‡Π½Ρ‹Ρ… зарядов Ρ„ΠΎΡ€ΠΌΡƒΠ»Ρƒ (19) прСдставим Π² Π²ΠΈΠ΄Π΅

    , Π³Π΄Π΅ W ij = W ji .

    Π‘Π»Π΅Π΄ΠΎΠ²Π°Ρ‚Π΅Π»ΡŒΠ½ΠΎ, ,

    Π³Π΄Π΅ W i — энСргия взаимодСйствия i-Π³ΠΎ заряда с ΠΎΡΡ‚Π°Π»ΡŒΠ½Ρ‹ΠΌΠΈ зарядами.

    Π˜Π·Π²Π΅ΡΡ‚Π½ΠΎ, Ρ‡Ρ‚ΠΎ W i = q i j i , Π³Π΄Π΅ q i — i-ΠΉ заряд систСмы; j i — Ρ€Π΅Π·ΡƒΠ»ΡŒΡ‚ΠΈΡ€ΡƒΡŽΡ‰ΠΈΠΉ ΠΏΠΎΡ‚Π΅Π½Ρ†ΠΈΠ°Π», создаваСмый всСми ΠΎΡΡ‚Π°Π»ΡŒΠ½Ρ‹ΠΌΠΈ зарядами систСмы вмСстС нахоТдСния заряда q i . Π’Π°ΠΊΠΈΠΌ ΠΎΠ±Ρ€Π°Π·ΠΎΠΌ,

    . (20)

    Полная энСргия систСмы зарядов

    Если заряды распрСдСлСны ΠΏΠΎ ΠΎΠ±ΡŠΠ΅ΠΌΡƒ с объСмной ΠΏΠ»ΠΎΡ‚Π½ΠΎΡΡ‚ΡŒΡŽ заряда r, Ρ‚ΠΎ систСму зарядов ΠΌΠΎΠΆΠ½ΠΎ ΠΏΡ€Π΅Π΄ΡΡ‚Π°Π²ΠΈΡ‚ΡŒ ΠΊΠ°ΠΊ ΡΠΎΠ²ΠΎΠΊΡƒΠΏΠ½ΠΎΡΡ‚ΡŒ элСмСнтарных зарядов dq = rdV, Ρ‚. Π΅. dW = j dq = j rdV.

    Π‘ ΡƒΡ‡Π΅Ρ‚ΠΎΠΌ этого Ρ„ΠΎΡ€ΠΌΡƒΠ»Π° (20) послС интСгрирования ΠΏΡ€ΠΈΠ½ΠΈΠΌΠ°Π΅Ρ‚ Π²ΠΈΠ΄

    , (21)

    Π³Π΄Π΅ j — ΠΏΠΎΡ‚Π΅Π½Ρ†ΠΈΠ°Π», созданный всСми зарядами Π² элСмСнтарном объСмС dV.

    Если заряды распрСдСлСны с повСрхностной ΠΏΠ»ΠΎΡ‚Π½ΠΎΡΡ‚ΡŒΡŽ заряда s, Ρ‚ΠΎ

    . (22)

    Π€ΠΎΡ€ΠΌΡƒΠ»Ρ‹ (21) ΠΈ (22) ΠΏΠΎΠ·Π²ΠΎΠ»ΡΡŽΡ‚ Π½Π°ΠΉΡ‚ΠΈ ΠΏΠΎΠ»Π½ΡƒΡŽ ΡΠ½Π΅Ρ€Π³ΠΈΡŽ систСмы, Π° Ρ„ΠΎΡ€ΠΌΡƒΠ»Π° (20) — Ρ‚ΠΎΠ»ΡŒΠΊΠΎ ΡΠΎΠ±ΡΡ‚Π²Π΅Π½Π½ΡƒΡŽ ΡΠ½Π΅Ρ€Π³ΠΈΡŽ заряда. Π”Π΅ΠΉΡΡ‚Π²ΠΈΡ‚Π΅Π»ΡŒΠ½ΠΎ, согласно (21), W = W 1 + W 2 + W 12 , Π³Π΄Π΅ W 1 , W 2 — собствСнныС энСргии заряда q 1 ΠΈ q 2 ; W 12 — энСргия взаимодСйствия этих зарядов.

    ЭнСргия систСмы заряТСнных ΠΏΡ€ΠΎΠ²ΠΎΠ΄Π½ΠΈΠΊΠΎΠ²

    Π˜ΡΠΏΠΎΠ»ΡŒΠ·ΡƒΡ Ρ„ΠΎΡ€ΠΌΡƒΠ»Ρƒ (21) Π½Π°ΠΉΠ΄Π΅ΠΌ ΡΠ½Π΅Ρ€Π³ΠΈΡŽ ΠΈΠ·ΠΎΠ»ΠΈΡ€ΠΎΠ²Π°Π½Π½ΠΎΠ³ΠΎ (ΡƒΠ΅Π΄ΠΈΠ½Π΅Π½Π½ΠΎΠ³ΠΎ) ΠΏΡ€ΠΎΠ²ΠΎΠ΄Π½ΠΈΠΊΠ°. Если ΠΏΡ€ΠΎΠ²ΠΎΠ΄Π½ΠΈΠΊ ΠΈΠΌΠ΅Π΅Ρ‚ заряд q ΠΈ ΠΏΠΎΡ‚Π΅Π½Ρ†ΠΈΠ°Π» j = сonst Π²ΠΎ всСх Ρ‚ΠΎΡ‡ΠΊΠ°Ρ…, Π³Π΄Π΅ распрСдСлСн заряд, Ρ‚ΠΎ

    . (23)

    Π’Π°ΠΊ ΠΊΠ°ΠΊ для плоского кондСнсатора (Π΄Π²Π° заряТСнных ΠΏΡ€ΠΎΠ²ΠΎΠ΄Π½ΠΈΠΊΠ°)

    , (24)

    Π³Π΄Π΅ Β½+qΒ½=Β½-qΒ½= q; Dj — Ρ€Π°Π·Π½ΠΎΡΡ‚ΡŒ ΠΏΠΎΡ‚Π΅Π½Ρ†ΠΈΠ°Π»ΠΎΠ² ΠΌΠ΅ΠΆΠ΄Ρƒ ΠΏΠΎΠ»ΠΎΠΆΠΈΡ‚Π΅Π»ΡŒΠ½ΠΎ ΠΈ ΠΎΡ‚Ρ€ΠΈΡ†Π°Ρ‚Π΅Π»ΡŒΠ½ΠΎ заряТСнными ΠΎΠ±ΠΊΠ»Π°Π΄ΠΊΠ°ΠΌΠΈ кондСнсатора; W — полная энСргия взаимодСйствия Π½Π΅ Ρ‚ΠΎΠ»ΡŒΠΊΠΎ зарядов ΠΎΠ΄Π½ΠΎΠΉ ΠΎΠ±ΠΊΠ»Π°Π΄ΠΊΠΈ с зарядами Π΄Ρ€ΡƒΠ³ΠΎΠΉ, Π½ΠΎ ΠΈ энСргия взаимодСйствия зарядов Π²Π½ΡƒΡ‚Ρ€ΠΈ ΠΊΠ°ΠΆΠ΄ΠΎΠΉ ΠΈΠ· ΠΎΠ±ΠΊΠ»Π°Π΄ΠΎΠΊ.

    Π€ΠΎΡ€ΠΌΡƒΠ»Π° (24) остаСтся справСдливой ΠΈ ΠΏΡ€ΠΈ Π½Π°Π»ΠΈΡ‡ΠΈΠΈ диэлСктрика ΠΌΠ΅ΠΆΠ΄Ρƒ ΠΎΠ±ΠΊΠ»Π°Π΄ΠΊΠ°ΠΌΠΈ кондСнсатора.

    Если ΠΈΡΠΏΠΎΠ»ΡŒΠ·ΠΎΠ²Π°Ρ‚ΡŒ СмкостныС коэффициСнты, Ρ‚ΠΎ

    . (25)

    ЭнСргия элСктричСского поля

    Для нахоТдСния энСргии ΠΌΡ‹ использовали Ρ‚ΠΎΠ»ΡŒΠΊΠΎ заряды ΠΈ ΠΏΠΎΡ‚Π΅Π½Ρ†ΠΈΠ°Π»Ρ‹. Основной характСристикой элСктричСского поля являСтся Π²Π΅ΠΊΡ‚ΠΎΡ€ напряТСнности . Π’ΠΎΠ³Π΄Π° ΡΠ½Π΅Ρ€Π³ΠΈΡŽ элСктричСского поля ΠΌΠ΅ΠΆΠ΄Ρƒ ΠΎΠ±ΠΊΠ»Π°Π΄ΠΊΠ°ΠΌΠΈ плоского кондСнсатора ΠΌΠΎΠΆΠ½ΠΎ Π½Π°ΠΉΡ‚ΠΈ, прСобразуя Ρ„ΠΎΡ€ΠΌΡƒΠ»Ρƒ (23) с ΡƒΡ‡Π΅Ρ‚ΠΎΠΌ Ρ‚ΠΎΠ³ΠΎ, Ρ‡Ρ‚ΠΎ Dj = Π•d; .

    ПослС подстановки ΠΏΠΎΠ»ΡƒΡ‡ΠΈΠΌ

    . (26)

    Π‘ ΡƒΡ‡Π΅Ρ‚ΠΎΠΌ диэлСктрика ΠΌΠ΅ΠΆΠ΄Ρƒ ΠΎΠ±ΠΊΠ»Π°Π΄ΠΊΠ°ΠΌΠΈ кондСнсатора

    . (27)

    Π˜Π·Π²Π΅ΡΡ‚Π½ΠΎ, Ρ‡Ρ‚ΠΎ элСктричСскоС ΠΏΠΎΠ»Π΅ являСтся частным случаСм элСктромагнитного поля, ΠΊΠΎΡ‚ΠΎΡ€ΠΎΠ΅ ΠΌΠΎΠΆΠ΅Ρ‚ ΡΡƒΡ‰Π΅ΡΡ‚Π²ΠΎΠ²Π°Ρ‚ΡŒ ΠΎΡ‚Π΄Π΅Π»ΡŒΠ½ΠΎ ΠΎΡ‚ источников поля, Ρ‚.Π΅. распространСниС элСктромагнитных Π²ΠΎΠ»Π½ Π² пространствС связано с пСрСносом энСргии.

    Π‘Π»Π΅Π΄ΠΎΠ²Π°Ρ‚Π΅Π»ΡŒΠ½ΠΎ, элСктростатичСскоС ΠΏΠΎΠ»Π΅ ΠΈΠΌΠ΅Π΅Ρ‚ ΡΠ½Π΅Ρ€Π³ΠΈΡŽ, Ρ€Π°ΡΠΏΡ€Π΅Π΄Π΅Π»Π΅Π½Π½ΡƒΡŽ Π² Π½Π΅ΠΌ с объСмной ΠΏΠ»ΠΎΡ‚Π½ΠΎΡΡ‚ΡŒΡŽ w эл.

    Π’ случаС ΠΎΠ΄Π½ΠΎΡ€ΠΎΠ΄Π½ΠΎΠ³ΠΎ элСктричСского поля

    Если элСктричСскоС ΠΏΠΎΠ»Π΅ Π½Π΅ΠΎΠ΄Π½ΠΎΡ€ΠΎΠ΄Π½ΠΎ, Ρ‚ΠΎ

    Π³Π΄Π΅ .

    Π’ этом случаС объСмная ΠΏΠ»ΠΎΡ‚Π½ΠΎΡΡ‚ΡŒ энСргии элСктричСского поля

    . (29)

    Π‘Π»Π΅Π΄ΠΎΠ²Π°Ρ‚Π΅Π»ΡŒΠ½ΠΎ, полная энСргия элСктричСского поля

    . (30)

    Π’Π°ΠΊΠΈΠΌ ΠΎΠ±Ρ€Π°Π·ΠΎΠΌ, Π² ΠΎΡ‚Π»ΠΈΡ‡ΠΈΠ΅ ΠΎΡ‚ Π³Ρ€Π°Π²ΠΈΡ‚Π°Ρ†ΠΈΠΎΠ½Π½ΠΎΠ³ΠΎ поля элСктростатичСскоС (элСктромагнитноС) ΠΏΠΎΠ»Π΅ характСризуСтся объСмной ΠΏΠ»ΠΎΡ‚Π½ΠΎΡΡ‚ΡŒΡŽ энСргии, ΠΈ ΠΌΠΎΠΆΠ½ΠΎ Π³ΠΎΠ²ΠΎΡ€ΠΈΡ‚ΡŒ ΠΎ Π»ΠΎΠΊΠ°Π»ΠΈΠ·Π°Ρ†ΠΈΠΈ элСктричСской энСргии Π² пространствС.

    Π’ΠΎ ΠΌΠ½ΠΎΠ³ΠΈΡ… случаях для получСния Π½ΡƒΠΆΠ½ΠΎΠΉ элСктроСмкости кондСнсаторы приходится ΡΠΎΠ΅Π΄ΠΈΠ½ΡΡ‚ΡŒ Π² Π³Ρ€ΡƒΠΏΠΏΡƒ, которая называСтся Π±Π°Ρ‚Π°Ρ€Π΅Π΅ΠΉ .

    ΠŸΠΎΡΠ»Π΅Π΄ΠΎΠ²Π°Ρ‚Π΅Π»ΡŒΠ½Ρ‹ΠΌ называСтся Ρ‚Π°ΠΊΠΎΠ΅ соСдинСниС кондСнсаторов, ΠΏΡ€ΠΈ ΠΊΠΎΡ‚ΠΎΡ€ΠΎΠΌ ΠΎΡ‚Ρ€ΠΈΡ†Π°Ρ‚Π΅Π»ΡŒΠ½ΠΎ заряТСнная ΠΎΠ±ΠΊΠ»Π°Π΄ΠΊΠ° ΠΏΡ€Π΅Π΄Ρ‹Π΄ΡƒΡ‰Π΅Π³ΠΎ кондСнсатора соСдинСна с ΠΏΠΎΠ»ΠΎΠΆΠΈΡ‚Π΅Π»ΡŒΠ½ΠΎ заряТСнной ΠΎΠ±ΠΊΠ»Π°Π΄ΠΊΠΎΠΉ ΠΏΠΎΡΠ»Π΅Π΄ΡƒΡŽΡ‰Π΅Π³ΠΎ (рис. 15.31). ΠŸΡ€ΠΈ ΠΏΠΎΡΠ»Π΅Π΄ΠΎΠ²Π°Ρ‚Π΅Π»ΡŒΠ½ΠΎΠΌ соСдинСнии Π½Π° всСх ΠΎΠ±ΠΊΠ»Π°Π΄ΠΊΠ°Ρ… кондСнсаторов Π±ΡƒΠ΄ΡƒΡ‚ ΠΎΠ΄ΠΈΠ½Π°ΠΊΠΎΠ²Ρ‹Π΅ ΠΏΠΎ Π²Π΅Π»ΠΈΡ‡ΠΈΠ½Π΅ заряды q. Π’Π°ΠΊ ΠΊΠ°ΠΊ заряды Π½Π° кондСнсаторС находятся Π² равновСсии, Ρ‚ΠΎ ΠΏΠΎΡ‚Π΅Π½Ρ†ΠΈΠ°Π»Ρ‹ ΠΎΠ±ΠΊΠ»Π°Π΄ΠΎΠΊ, соСдинённых ΠΌΠ΅ΠΆΠ΄Ρƒ собой ΠΏΡ€ΠΎΠ²ΠΎΠ΄Π½ΠΈΠΊΠ°ΠΌΠΈ, Π±ΡƒΠ΄ΡƒΡ‚ ΠΎΠ΄ΠΈΠ½Π°ΠΊΠΎΠ²Ρ‹ΠΌΠΈ.

    Учитывая эти ΠΎΠ±ΡΡ‚ΠΎΡΡ‚Π΅Π»ΡŒΡΡ‚Π²Π°, Π²Ρ‹Π²Π΅Π΄Π΅ΠΌ Ρ„ΠΎΡ€ΠΌΡƒΠ»Ρƒ для вычислСния элСктроСмкости Π±Π°Ρ‚Π°Ρ€Π΅ΠΈ ΠΏΠΎΡΠ»Π΅Π΄ΠΎΠ²Π°Ρ‚Π΅Π»ΡŒΠ½ΠΎ соСдинСнных кондСнсаторов. Из рис. 15.31 Π²ΠΈΠ΄Π½ΠΎ, Ρ‡Ρ‚ΠΎ напряТСниС Π½Π° Π±Π°Ρ‚Π°Ρ€Π΅Π΅ U 6 Ρ€Π°Π²Π½ΠΎ суммС напряТСний Π½Π° ΠΏΠΎΡΠ»Π΅Π΄ΠΎΠ²Π°Ρ‚Π΅Π»ΡŒΠ½ΠΎ соСдинСнных кондСнсаторах. Π”Π΅ΠΉΡΡ‚Π²ΠΈΡ‚Π΅Π»ΡŒΠ½ΠΎ:

    (Ο• 1 β€’ Ο• 2) + (Ο• 2 β€’ Ο• 3) + … + (Ο• n-1 β€’ Ο• n) = Ο• 1 β€’ Ο• n

    U 1 + U 2 + … + U n = U 6

    Π˜ΡΠΏΠΎΠ»ΡŒΠ·ΡƒΡ ΡΠΎΠΎΡ‚Π½ΠΎΡˆΠ΅Π½ΠΈΡ q= CU, ΠΏΠΎΠ»ΡƒΡ‡ΠΈΠΌ:

    ПослС сокращСния Π½Π° q Π±ΡƒΠ΄Π΅ΠΌ ΠΈΠΌΠ΅Ρ‚ΡŒ:

    Из (15.21) Π²ΠΈΠ΄Π½ΠΎ, Ρ‡Ρ‚ΠΎ ΠΏΡ€ΠΈ ΠΏΠΎΡΠ»Π΅Π΄ΠΎΠ²Π°Ρ‚Π΅Π»ΡŒΠ½ΠΎΠΌ соСдинСнии ΡΠ»Π΅ΠΊΡ‚Ρ€ΠΎΠ΅ΠΌΠΊΠΎΡΡ‚ΡŒ Π±Π°Ρ‚Π°Ρ€Π΅ΠΈ оказываСтся мСньшС самой малСнькой ΠΈΠ· элСктроСмкостСй ΠΎΡ‚Π΄Π΅Π»ΡŒΠ½Ρ‹Ρ… кондСнсаторов .

    ΠŸΠ°Ρ€Π°Π»Π»Π΅Π»ΡŒΠ½Ρ‹ΠΌ называСтся соСдинСниС кондСнсаторов, ΠΏΡ€ΠΈ ΠΊΠΎΡ‚ΠΎΡ€ΠΎΠΌ всС ΠΏΠΎΠ»ΠΎΠΆΠΈΡ‚Π΅Π»ΡŒΠ½ΠΎ заряТСнныС ΠΎΠ±ΠΊΠ»Π°Π΄ΠΊΠΈ присоСдинСны ΠΊ ΠΎΠ΄Π½ΠΎΠΌΡƒ ΠΏΡ€ΠΎΠ²ΠΎΠ΄Ρƒ, Π° ΠΎΡ‚Ρ€ΠΈΡ†Π°Ρ‚Π΅Π»ΡŒΠ½ΠΎ заряТСнныС — ΠΊ Π΄Ρ€ΡƒΠ³ΠΎΠΌΡƒ (рис. 15.32). Π’ этом случаС напряТСния Π½Π° всСх кондСнсаторах ΠΎΠ΄ΠΈΠ½Π°ΠΊΠΎΠ²Ρ‹ ΠΈ Ρ€Π°Π²Π½Ρ‹ U, Π° заряд Π½Π° Π±Π°Ρ‚Π°Ρ€Π΅Π΅ q Π± Ρ€Π°Π²Π΅Π½ суммС зарядов Π½Π° ΠΎΡ‚Π΄Π΅Π»ΡŒΠ½Ρ‹Ρ… кондСнсаторах:

    q Π± = q 1 + q 2 + … = q n

    C Π± U = C 1 U + C 2 U + … + C n U

    ПослС сокращСния Π½Π° ΠΈ ΠΏΠΎΠ»ΡƒΡ‡Π°Π΅ΠΌ Ρ„ΠΎΡ€ΠΌΡƒΠ»Ρƒ для вычислСния элСктроСмкости Π±Π°Ρ‚Π°Ρ€Π΅ΠΈ ΠΏΠ°Ρ€Π°Π»Π»Π΅Π»ΡŒΠ½ΠΎ соСдинСнных кондСнсаторов :

    C Π± = C 1 + C 2 + … + C n (15.22)

    Из (15.22) Π²ΠΈΠ΄Π½ΠΎ, Ρ‡Ρ‚ΠΎ ΠΏΡ€ΠΈ ΠΏΠ°Ρ€Π°Π»Π»Π΅Π»ΡŒΠ½ΠΎΠΌ соСдинСнии ΡΠ»Π΅ΠΊΡ‚Ρ€ΠΎΠ΅ΠΌΠΊΠΎΡΡ‚ΡŒ Π±Π°Ρ‚Π°Ρ€Π΅ΠΈ получаСтся большС, Ρ‡Π΅ΠΌ самая большая ΠΈΠ· элСктроСмкостСй ΠΎΡ‚Π΄Π΅Π»ΡŒΠ½Ρ‹Ρ… кондСнсаторов. ΠŸΡ€ΠΈ ΠΈΠ·Π³ΠΎΡ‚ΠΎΠ²Π»Π΅Π½ΠΈΠΈ кондСнсаторов большой элСктроСмкости ΠΏΠΎΠ»ΡŒΠ·ΡƒΡŽΡ‚ΡΡ ΠΏΠ°Ρ€Π°Π»Π»Π΅Π»ΡŒΠ½Ρ‹ΠΌ соСдинСниСм, ΠΈΠ·ΠΎΠ±Ρ€Π°ΠΆΠ΅Π½Π½Ρ‹ΠΌ Π½Π° рис. 15.33. Π’Π°ΠΊΠΎΠΉ способ соСдинСния Π΄Π°Π΅Ρ‚ экономию Π² ΠΌΠ°Ρ‚Π΅Ρ€ΠΈΠ°Π»Π΅, Ρ‚Π°ΠΊ ΠΊΠ°ΠΊ заряды Ρ€Π°ΡΠΏΠΎΠ»Π°Π³Π°ΡŽΡ‚ΡΡ с ΠΎΠ±Π΅ΠΈΡ… сторон ΠΎΠ±ΠΊΠ»Π°Π΄ΠΎΠΊ кондСнсаторов (ΠΊΡ€ΠΎΠΌΠ΅ Π΄Π²ΡƒΡ… ΠΊΡ€Π°ΠΉΠ½ΠΈΡ… ΠΎΠ±ΠΊΠ»Π°Π΄ΠΎΠΊ).

    На рис. 15.33 соСдинСно ΠΏΠ°Ρ€Π°Π»Π»Π΅Π»ΡŒΠ½ΠΎ 6 кондСнсаторов, Π° ΠΎΠ±ΠΊΠ»Π°Π΄ΠΎΠΊ сдСлано 7. Π‘Π»Π΅Π΄ΠΎΠ²Π°Ρ‚Π΅Π»ΡŒΠ½ΠΎ, Π² этом случаС ΠΏΠ°Ρ€Π°Π»Π»Π΅Π»ΡŒΠ½ΠΎ соСдинСнных кондСнсаторов Π½Π° ΠΎΠ΄ΠΈΠ½ мСньшС, Ρ‡Π΅Π» число мСталличСских листов ΠΏ Π² Π±Π°Ρ‚Π°Ρ€Π΅Π΅ кондСнсаторов, Ρ‚. Π΅.

    C Π± = Ɛ C S(n β€’ 1)/d (15.23)

    Π’ элСктричСских цСпях ΠΈ схСмах ΠΈΡΠΏΠΎΠ»ΡŒΠ·ΡƒΡŽΡ‚ΡΡ Ρ€Π°Π·Π»ΠΈΡ‡Π½Ρ‹Π΅ ΠΌΠ΅Ρ‚ΠΎΠ΄Ρ‹ соСдинСния кондСнсаторов. Π‘ΠΎΠ΅Π΄ΠΈΠ½Π΅Π½ΠΈΠ΅ кондСнсаторов ΠΌΠΎΠΆΠ΅Ρ‚ Π±Ρ‹Ρ‚ΡŒ ΠΏΠΎΡΠ»Π΅Π΄ΠΎΠ²Π°Ρ‚Π΅Π»ΡŒΠ½Ρ‹ΠΌ, ΠΏΠ°Ρ€Π°Π»Π»Π΅Π»ΡŒΠ½Ρ‹ΠΌ ΠΈ ΠΏΠΎΡΠ»Π΅Π΄ΠΎΠ²Π°Ρ‚Π΅Π»ΡŒΠ½ΠΎ-ΠΏΠ°Ρ€Π°Π»Π»Π΅Π»ΡŒΠ½Ρ‹ΠΌ (смСшанноС соСдинСниС кондСнсаторов).

    Если ΠΏΠΎΠ΄ΠΊΠ»ΡŽΡ‡Π΅Π½ΠΈΠ΅ СмкостСй Π² Π±Π°Ρ‚Π°Ρ€Π΅ΡŽ осущСствляСтся Π² Π²ΠΈΠ΄Π΅ Ρ†Π΅ΠΏΠΎΡ‡ΠΊΠΈ ΠΈ ΠΊ Ρ‚ΠΎΡ‡ΠΊΠ°ΠΌ Π²ΠΊΠ»ΡŽΡ‡Π΅Π½ΠΈΡ Π² Ρ†Π΅ΠΏΡŒ присоСдинСны пластины Ρ‚ΠΎΠ»ΡŒΠΊΠΎ ΠΏΠ΅Ρ€Π²ΠΎΠ³ΠΎ ΠΈ послСднСго кондСнсаторов, Ρ‚ΠΎ Ρ‚Π°ΠΊΠΎΠ΅ соСдинСниС называСтся ΠΏΠΎΡΠ»Π΅Π΄ΠΎΠ²Π°Ρ‚Π΅Π»ΡŒΠ½Ρ‹ΠΌ .

    ΠŸΡ€ΠΈ ΠΏΠΎΡΠ»Π΅Π΄ΠΎΠ²Π°Ρ‚Π΅Π»ΡŒΠ½ΠΎΠΌ соСдинСниС кондСнсаторов ΠΎΠ½ΠΈ Π·Π°Ρ€ΡΠΆΠ°ΡŽΡ‚ΡΡ ΠΎΠ΄ΠΈΠ½Π°ΠΊΠΎΠ²Ρ‹ΠΌ количСством элСктричСства, хотя ΠΎΡ‚ источника Ρ‚ΠΎΠΊΠ° Π·Π°Ρ€ΡΠΆΠ°ΡŽΡ‚ΡΡ Ρ‚ΠΎΠ»ΡŒΠΊΠΎ Π΄Π²Π΅ ΠΊΡ€Π°ΠΉΠ½ΠΈΠ΅ пластины, Π° ΠΎΡΡ‚Π°Π»ΡŒΠ½Ρ‹Π΅ пластины Π·Π°Ρ€ΡΠΆΠ°ΡŽΡ‚ΡΡ Ρ‡Π΅Ρ€Π΅Π· влияниС элСктричСского поля. ΠŸΡ€ΠΈ этом заряд пластины 2 Π±ΡƒΠ΄Π΅Ρ‚ Ρ€Π°Π²Π΅Π½ ΠΏΠΎ Π½ΠΎΠΌΠΈΠ½Π°Π»Ρƒ, Π½ΠΎ ΠΏΡ€ΠΎΡ‚ΠΈΠ²ΠΎΠΏΠΎΠ»ΠΎΠΆΠ΅Π½ ΠΏΠΎ Π·Π½Π°ΠΊΡƒ заряду пластины 1, заряд пластины 3 Π±ΡƒΠ΄Π΅Ρ‚ Ρ€Π°Π²Π΅Π½ заряду пластины 2, Π½ΠΎ Ρ‚Π°ΠΊΠΆΠ΅ Π±ΡƒΠ΄Π΅Ρ‚ ΠΏΡ€ΠΎΡ‚ΠΈΠ²ΠΎΠΏΠΎΠ»ΠΎΠΆΠ½ΠΎΠΉ полярности ΠΈ Ρ‚. Π΄.

    Но Ссли Π³ΠΎΠ²ΠΎΡ€ΠΈΡ‚ΡŒ Ρ‚ΠΎΡ‡Π½Π΅Π΅, напряТСния Π½Π° Ρ€Π°Π·Π»ΠΈΡ‡Π½Ρ‹Ρ… Смкостных элСмСнтах Π±ΡƒΠ΄ΡƒΡ‚ ΠΎΡ‚Π»ΠΈΡ‡Π°Ρ‚ΡŒΡΡ, Ρ‚Π°ΠΊ ΠΊΠ°ΠΊ для заряда ΠΎΠ΄Π½ΠΈΠΌ ΠΈ Ρ‚Π΅ΠΌ ΠΆΠ΅ количСством элСктричСства ΠΏΡ€ΠΈ Ρ€Π°Π·Π»ΠΈΡ‡Π½ΠΎΠΉ номинальной Смкости всСгда Π½Π΅ΠΎΠ±Ρ…ΠΎΠ΄ΠΈΠΌΡ‹ Ρ€Π°Π·Π»ΠΈΡ‡Π½Ρ‹Π΅ напряТСния. Π§Π΅ΠΌ Π½ΠΈΠΆΠ΅Π΅ Π΅ΠΌΠΊΠΎΡΡ‚ΡŒ кондСнсатора, Ρ‚Π΅ΠΌ больший ΡƒΡ€ΠΎΠ²Π΅Π½ΡŒ напряТСниС трСбуСтся для Ρ‚ΠΎΠ³ΠΎ, Ρ‡Ρ‚ΠΎΠ±Ρ‹ Π·Π°Ρ€ΡΠ΄ΠΈΡ‚ΡŒ Ρ€Π°Π΄ΠΈΠΎΠΊΠΎΠΌΠΏΠΎΠ½Π΅Π½Ρ‚ Π½Π΅ΠΎΠ±Ρ…ΠΎΠ΄ΠΈΠΌΡ‹ΠΌ количСством элСктричСства, ΠΈ Π½Π°ΠΎΠ±ΠΎΡ€ΠΎΡ‚.

    Π’Π°ΠΊΠΈΠΌ ΠΎΠ±Ρ€Π°Π·ΠΎΠΌ, ΠΏΡ€ΠΈ зарядС Π³Ρ€ΡƒΠΏΠΏΡ‹ СмкостСй, соСдинСнных ΠΏΠΎΡΠ»Π΅Π΄ΠΎΠ²Π°Ρ‚Π΅Π»ΡŒΠ½ΠΎ, Π½Π° кондСнсаторах ΠΌΠ°Π»ΠΎΠΉ Смкости напряТСния Π±ΡƒΠ΄ΡƒΡ‚ Π²Ρ‹ΡˆΠ΅, Π° Π½Π° элСмСнтах большой Смкости — Π½ΠΈΠΆΠ΅.

    Рассмотрим всю Π³Ρ€ΡƒΠΏΠΏΡƒ СмкостСй соСдинСнных ΠΏΠΎΡΠ»Π΅Π΄ΠΎΠ²Π°Ρ‚Π΅Π»ΡŒΠ½ΠΎ, ΠΊΠ°ΠΊ ΠΎΠ΄Π½Ρƒ ΡΠΊΠ²ΠΈΠ²Π°Π»Π΅Π½Ρ‚Π½ΡƒΡŽ Π΅ΠΌΠΊΠΎΡΡ‚ΡŒ, ΠΌΠ΅ΠΆΠ΄Ρƒ пластинами ΠΊΠΎΡ‚ΠΎΡ€ΠΎΠΉ сущСствуСт ΠΊΠ°ΠΊΠΎΠΉ-Ρ‚ΠΎ ΡƒΡ€ΠΎΠ²Π΅Π½ΡŒ напряТСния, Ρ€Π°Π²Π½Ρ‹ΠΉ суммС напряТСний Π½Π° всСх элСмСнтах Π³Ρ€ΡƒΠΏΠΏΡ‹, Π° заряд ΠΊΠΎΡ‚ΠΎΡ€ΠΎΠ³ΠΎ Ρ€Π°Π²Π΅Π½ заряду любого ΠΊΠΎΠΌΠΏΠΎΠ½Π΅Π½Ρ‚Π° ΠΈΠ· Π΄Π°Π½Π½ΠΎΠΉ Π³Ρ€ΡƒΠΏΠΏΡ‹.

    Если Π±ΠΎΠ»Π΅Π΅ ΠΏΡ€ΠΈΡΡ‚Π°Π»ΡŒΠ½ΠΎ Ρ€Π°ΡΡΠΌΠΎΡ‚Ρ€Π΅Ρ‚ΡŒ самый мСньший Π½ΠΎΠΌΠΈΠ½Π°Π» Смкости Π² Π³Ρ€ΡƒΠΏΠΏΠ΅, Ρ‚ΠΎ Π½Π° Π½Π΅ΠΌ Π΄ΠΎΠ»ΠΆΠ½ΠΎ Π±Ρ‹Ρ‚ΡŒ самый высокий ΡƒΡ€ΠΎΠ²Π΅Π½ΡŒ напряТСния. Но фактичСски, ΡƒΡ€ΠΎΠ²Π΅Π½ΡŒ напряТСния Π½Π° Π½Π΅ΠΌ составляСт Ρ‚ΠΎΠ»ΡŒΠΊΠΎ Ρ‡Π°ΡΡ‚ΡŒ ΠΎΠ±Ρ‰Π΅Π³ΠΎ значСния напряТСния, ΠΎΡ‚ ΠΎΠ±Ρ‰Π΅ΠΉ Π³Ρ€ΡƒΠΏΠΏΡ‹. НапряТСниС Π½Π° всСй Π³Ρ€ΡƒΠΏΠΏΠ΅ всСгда Π²Ρ‹ΡˆΠ΅ напряТСния Π½Π° кондСнсаторС, ΠΈΠΌΠ΅ΡŽΡ‰Π΅ΠΌ ΡΠ°ΠΌΡƒΡŽ ΠΌΠ°Π»ΡƒΡŽ Π²Π΅Π»Π΅Ρ‡ΠΈΠ½Ρƒ Смкости. А поэтому ΠΌΠΎΠΆΠ½ΠΎ ΡΠΊΠ°Π·Π°Ρ‚ΡŒ, Ρ‡Ρ‚ΠΎ общая Π΅ΠΌΠΊΠΎΡΡ‚ΡŒ Π³Ρ€ΡƒΠΏΠΏΡ‹ кондСнсаторов, соСдинСнных ΠΏΠΎΡΠ»Π΅Π΄ΠΎΠ²Π°Ρ‚Π΅Π»ΡŒΠ½ΠΎ, мСньшС Смкости самого ΠΌΠ°Π»ΠΎΠ³ΠΎ кондСнсатора Π² Π³Ρ€ΡƒΠΏΠΏΠ΅ .

    Для вычислСния ΠΎΠ±Ρ‰Π΅ΠΉ Смкости Π³Ρ€ΡƒΠΏΠΏΡ‹, Π² Π΄Π°Π½Π½ΠΎΠΌ ΠΏΡ€ΠΈΠΌΠ΅Ρ€Π΅ Π²ΠΎΡΠΏΠΎΠ»ΡŒΠ·ΡƒΠ΅ΠΌΡΡ ΡΠ»Π΅Π΄ΡƒΡŽΡ‰Π΅ΠΉ Ρ„ΠΎΡ€ΠΌΡƒΠ»ΠΎΠΉ:

    1 / C ΠΎΠ±Ρ‰ = 1/C 1 + 1/C 2 + 1/C 3

    Для частного случая ΠΏΡ€ΠΈ Π΄Π²ΡƒΡ… ΠΏΠΎΡΠ»Π΅Π΄ΠΎΠ²Π°Ρ‚Π΅Π»ΡŒΠ½ΠΎ соСдинСнных элСмСнтов Ρ„ΠΎΡ€ΠΌΡƒΠ»Π° ΠΏΡ€ΠΈΠΌΠ΅Ρ‚ Π²ΠΈΠ΄:

    C ΠΎΠ±Ρ‰ = Π‘ 1 Γ— Π‘ 2 /C 1 + C 2

    Если Π³Ρ€ΡƒΠΏΠΏΠ° Смкостных элСмСнтов Π²ΠΊΠ»ΡŽΡ‡Π΅Π½Π° Π² схСму Ρ‚Π°ΠΊΠΈΠΌ ΠΎΠ±Ρ€Π°Π·ΠΎΠΌ, Ρ‡Ρ‚ΠΎ ΠΊ Ρ‚ΠΎΡ‡ΠΊΠ°ΠΌ нСпосрСдствСнного Π²ΠΊΠ»ΡŽΡ‡Π΅Π½ΠΈΡ присоСдинСны пластины всСх ΠΊΠΎΠΌΠΏΠΎΠ½Π΅Π½Ρ‚ΠΎΠ² схСмы, Ρ‚ΠΎ Ρ‚Π°ΠΊΠΎΠ΅ соСдинСниС называСтся ΠΏΠ°Ρ€Π°Π»Π»Π΅Π»ΡŒΠ½Ρ‹ΠΌ соСдинСниСм кондСнсаторов.

    ΠŸΡ€ΠΈ зарядС Π³Ρ€ΡƒΠΏΠΏΡ‹ СмкостСй, Π²ΠΊΠ»ΡŽΡ‡Π΅Π½Π½Ρ‹Ρ… ΠΏΠ°Ρ€Π°Π»Π»Π΅Π»ΡŒΠ½ΠΎ, ΠΌΠ΅ΠΆΠ΄Ρƒ пластинами всСх элСмСнтов Π±ΡƒΠ΄Π΅Ρ‚ ΠΎΠ΄Π½ΠΎ ΠΈ Ρ‚ΠΎΠΆΠ΅ напряТСниС, Ρ‚Π°ΠΊ ΠΊΠ°ΠΊ всС ΠΎΠ½ΠΈ Π·Π°Ρ€ΡΠΆΠ°ΡŽΡ‚ΡΡ ΠΎΡ‚ ΠΎΠ΄Π½ΠΎΠ³ΠΎ источника питания. ΠžΠ±Ρ‰Π΅Π΅ количСство элСктричСства Π½Π° всСх элСмСнтах Π±ΡƒΠ΄Π΅Ρ‚ Ρ€Π°Π²Π½ΠΎ суммС количСств элСктричСства, ΠΏΠΎΠΌΠ΅Ρ‰Π°ΡŽΡ‰ΠΈΡ…ΡΡ Π½Π° ΠΊΠ°ΠΆΠ΄ΠΎΠΉ Смкости Π² ΠΎΡ‚Π΄Π΅Π»ΡŒΠ½ΠΎΡΡ‚ΠΈ, Ρ‚Π°ΠΊ ΠΊΠ°ΠΊ заряд ΠΊΠ°ΠΆΠ΄ΠΎΠΉ ΠΈΠ· Π½ΠΈΡ… осущСствляСтся нСзависимо ΠΎΡ‚ заряда Π΄Ρ€ΡƒΠ³ΠΈΡ… ΠΊΠΎΠΌΠΏΠΎΠ½Π΅Π½Ρ‚ΠΎΠ² Π΄Π°Π½Π½ΠΎΠΉ схСмы. Π˜ΡΡ…ΠΎΠ΄Ρ ΠΈΠ· этого, всю систСму ΠΌΠΎΠΆΠ½ΠΎ Ρ€Π°ΡΡΠΌΠ°Ρ‚Ρ€ΠΈΠ²Π°Ρ‚ΡŒ ΠΊΠ°ΠΊ ΠΎΠ΄ΠΈΠ½ ΠΎΠ±Ρ‰ΠΈΠΉ эквивалСнтный кондСнсатор. Π’ΠΎΠ³Π΄Π° общая Π΅ΠΌΠΊΠΎΡΡ‚ΡŒ ΠΏΡ€ΠΈ ΠΏΠ°Ρ€Π°Π»Π»Π΅Π»ΡŒΠ½ΠΎΠΌ соСдинСнии кондСнсаторов Ρ€Π°Π²Π½Π° суммС СмкостСй всСх соСдинСнных элСмСнтов.

    ΠžΠ±ΠΎΠ·Π½Π°Ρ‡ΠΈΠΌ ΡΡƒΠΌΠΌΠ°Ρ€Π½ΡƒΡŽ Π΅ΠΌΠΊΠΎΡΡ‚ΡŒ соСдинСнных Π² Π±Π°Ρ‚Π°Ρ€Π΅ΡŽ элСмСнтов символом Π‘ ΠΎΠ±Ρ‰ , Ρ‚ΠΎΠ³Π΄Π° ΠΌΠΎΠΆΠ½ΠΎ Π·Π°ΠΏΠΈΡΠ°Ρ‚ΡŒ Ρ„ΠΎΡ€ΠΌΡƒΠ»Ρƒ:

    C ΠΎΠ±Ρ‰ = Π‘ 1 + Π‘ 2 + C 3

    ΠŸΠΎΡΠ»Π΅Π΄ΠΎΠ²Π°Ρ‚Π΅Π»ΡŒΠ½ΠΎ-ΠΏΠ°Ρ€Π°Π»Π»Π΅Π»ΡŒΠ½Ρ‹ΠΌ соСдинСниСм кондСнсаторов называСтся Ρ†Π΅ΠΏΡŒ ΠΈΠ»ΠΈ схСма ΠΈΠΌΠ΅ΡŽΡ‰Π°Ρ Π² своСм составС участки, ΠΊΠ°ΠΊ с ΠΏΠ°Ρ€Π°Π»Π»Π΅Π»ΡŒΠ½Ρ‹ΠΌ, Ρ‚Π°ΠΊ ΠΈ с ΠΏΠΎΡΠ»Π΅Π΄ΠΎΠ²Π°Ρ‚Π΅Π»ΡŒΠ½Ρ‹ΠΌ соСдинСниСм Ρ€Π°Π΄ΠΈΠΎΠΊΠΎΠΌΠΏΠΎΠ½Π΅Π½Ρ‚ΠΎΠ².

    ΠŸΡ€ΠΈ расчСтС ΠΎΠ±Ρ‰Π΅ΠΉ Смкости Ρ‚Π°ΠΊΠΎΠΉ схСмы с ΠΏΠΎΡΠ»Π΅Π΄ΠΎΠ²Π°Ρ‚Π΅Π»ΡŒΠ½ΠΎ-ΠΏΠ°Ρ€Π°Π»Π»Π΅Π»ΡŒΠ½Ρ‹ΠΌ Ρ‚ΠΈΠΏΠΎΠΌ соСдинСния этот участок (ΠΊΠ°ΠΊ ΠΈ Π² случаС с ) Ρ€Π°Π·Π±ΠΈΠ²Π°ΡŽΡ‚ Π½Π° элСмСнтарныС участки, состоящиС ΠΈΠ· простых Π³Ρ€ΡƒΠΏΠΏ с ΠΏΠΎΡΠ»Π΅Π΄ΠΎΠ²Π°Ρ‚Π΅Π»ΡŒΠ½Ρ‹ΠΌ ΠΈΠ»ΠΈ ΠΏΠ°Ρ€Π°Π»Π»Π΅Π»ΡŒΠ½Ρ‹ΠΌ соСдинСниСм СмкостСй. Π”Π°Π»ΡŒΡˆΠ΅ Π°Π»Π³ΠΎΡ€ΠΈΡ‚ΠΌ вычислСний ΠΏΡ€ΠΈΠ½ΠΈΠΌΠ°Π΅Ρ‚ Π²ΠΈΠ΄:

    1. Π’Ρ‹Ρ‡ΠΈΡΠ»ΡΡŽΡ‚ ΡΠΊΠ²ΠΈΠ²Π°Π»Π΅Π½Ρ‚Π½ΡƒΡŽ Π΅ΠΌΠΊΠΎΡΡ‚ΡŒ участков с ΠΏΠΎΡΠ»Π΅Π΄ΠΎΠ²Π°Ρ‚Π΅Π»ΡŒΠ½Ρ‹ΠΌ соСдинСниСм кондСнсаторов
    2. Если эти участки состоят ΠΈΠ· ΠΏΠΎΡΠ»Π΅Π΄ΠΎΠ²Π°Ρ‚Π΅Π»ΡŒΠ½ΠΎ соСдинСнныС кондСнсаторы, Ρ‚ΠΎ сначала Π²Ρ‹Ρ‡ΠΈΡΠ»ΡΡŽΡ‚ ΠΈΡ… Π΅ΠΌΠΊΠΎΡΡ‚ΡŒ.
    3. ПослС расчСта эквивалСнтных СмкостСй ΠΏΠ΅Ρ€Π΅Ρ€ΠΈΡΠΎΠ²Ρ‹Π²Π°ΡŽΡ‚ схСму. ΠžΠ±Ρ‹Ρ‡Π½ΠΎ получаСтся схСма ΠΈΠ· ΠΏΠΎΡΠ»Π΅Π΄ΠΎΠ²Π°Ρ‚Π΅Π»ΡŒΠ½ΠΎ соСдинСнных эквивалСнтных кондСнсаторов.
    4. Π Π°ΡΡΡ‡ΠΈΡ‚Ρ‹Π²Π°ΡŽΡ‚ ΠΎΠ±Ρ‰ΡƒΡŽ Π΅ΠΌΠΊΠΎΡΡ‚ΡŒ ΠΏΠΎΠ»ΡƒΡ‡Π΅Π½Π½ΠΎΠΉ схСмы.

    ΠŸΡ€ΠΈΠΌΠ΅Ρ€ расчСта Смкости ΠΏΡ€ΠΈ смСшанном соСдинСниС кондСнсаторов

    ΠŸΠΎΡΠ»Π΅Π΄ΠΎΠ²Π°Ρ‚Π΅Π»ΡŒΠ½ΠΎΠ΅ соСдинСниС

    Π’ΠΎ ΠΌΠ½ΠΎΠ³ΠΈΡ… случаях для получСния Π½ΡƒΠΆΠ½ΠΎΠΉ элСктроСмкости кондСнсаторы ΠΎΠ±ΡŠΠ΅Π΄ΠΈΠ½ΡΡŽΡ‚ Π² Π³Ρ€ΡƒΠΏΠΏΡƒ, которая называСтся Π±Π°Ρ‚Π°Ρ€Π΅Π΅ΠΉ. Π•ΠΌΠΊΠΎΡΡ‚ΡŒ Π±Π°Ρ‚Π°Ρ€Π΅ΠΈ кондСнсаторов зависит ΠΎΡ‚ схСмы соСдинСния ΡΠΎΡΡ‚Π°Π²Π»ΡΡŽΡ‰ΠΈΡ… Π΅Π΅ кондСнсаторов. Π Π°Π·Π»ΠΈΡ‡Π°ΡŽΡ‚ Π΄Π²Π° Π²ΠΈΠ΄Π° соСдинСния: ΠΏΠΎΡΠ»Π΅Π΄ΠΎΠ²Π°Ρ‚Π΅Π»ΡŒΠ½ΠΎΠ΅ ΠΈ ΠΏΠ°Ρ€Π°Π»Π»Π΅Π»ΡŒΠ½ΠΎΠ΅. Π’ΠΎΠ·ΠΌΠΎΠΆΠ΅Π½ Ρ‚Π°ΠΊΠΆΠ΅ ΠΈ ΡΠΌΠ΅ΡˆΠ°Π½Π½Ρ‹ΠΉ Ρ‚ΠΈΠΏ соСдинСния кондСнсаторов Π² Π±Π°Ρ‚Π°Ρ€Π΅ΡŽ.

    Рис. 2.13. ΠŸΠΎΡΠ»Π΅Π΄ΠΎΠ²Π°Ρ‚Π΅Π»ΡŒΠ½ΠΎΠ΅ соСдинСниС кондСнсаторов

    ΠŸΠΎΡΠ»Π΅Π΄ΠΎΠ²Π°Ρ‚Π΅Π»ΡŒΠ½ΠΎΠ΅ соСдинСниС. ΠŸΡ€ΠΈ зарядкС Π±Π°Ρ‚Π°Ρ€Π΅ΠΈ (рис. 2.13) Ρ€Π°Π·Π½ΠΎΡΡ‚ΡŒ ΠΏΠΎΡ‚Π΅Π½Ρ†ΠΈΠ°Π»ΠΎΠ² распрСдСлится ΠΌΠ΅ΠΆΠ΄Ρƒ ΠΎΡ‚Π΄Π΅Π»ΡŒΠ½Ρ‹ΠΌΠΈ кондСнсаторами ΠΈ Π±ΡƒΠ΄Π΅Ρ‚ Ρ€Π°Π²Π½Π°

    Если ΠΏΠ΅Ρ€Π²ΠΎΠΉ ΠΎΠ±ΠΊΠ»Π°Π΄ΠΊΠ΅ Π±Π°Ρ‚Π°Ρ€Π΅ΠΈ кондСнсаторов сообщаСтся заряд , Ρ‚ΠΎ Π½Π° Π΅Π΅ Π²Ρ‚ΠΎΡ€ΠΎΠΉ ΠΎΠ±ΠΊΠ»Π°Π΄ΠΊΠ΅ появится ΠΈΠ½Π΄ΡƒΡ†ΠΈΡ€ΠΎΠ²Π°Π½Π½Ρ‹ΠΉ заряд . ΠŸΠΎΡΠΊΠΎΠ»ΡŒΠΊΡƒ эта ΠΎΠ±ΠΊΠ»Π°Π΄ΠΊΠ° соСдинСна с ΠΏΠ΅Ρ€Π²ΠΎΠΉ ΠΎΠ±ΠΊΠ»Π°Π΄ΠΊΠΎΠΉ Π²Ρ‚ΠΎΡ€ΠΎΠ³ΠΎ кондСнсатора ΠΈ ΠΏΠΎΡΠΊΠΎΠ»ΡŒΠΊΡƒ дСйствуСт Π·Π°ΠΊΠΎΠ½ сохранСния заряда, Π½Π° послСднСй появится заряд . Π’ свою ΠΎΡ‡Π΅Ρ€Π΅Π΄ΡŒ, это ΠΏΡ€ΠΈΠ²Π΅Π΄Π΅Ρ‚ ΠΊ появлСнию заряда Π½Π° Π΄Ρ€ΡƒΠ³ΠΎΠΉ ΠΎΠ±ΠΊΠ»Π°Π΄ΠΊΠ΅ Π²Ρ‚ΠΎΡ€ΠΎΠ³ΠΎ кондСнсатора ΠΈ Ρ‚. Π΄. Π’ Ρ€Π΅Π·ΡƒΠ»ΡŒΡ‚Π°Ρ‚Π΅ всС ΠΏΠΎΡΠ»Π΅Π΄ΠΎΠ²Π°Ρ‚Π΅Π»ΡŒΠ½ΠΎ соСдинСнныС кондСнсаторы Π±ΡƒΠ΄ΡƒΡ‚ заряТСны ΠΎΠ΄ΠΈΠ½Π°ΠΊΠΎΠ²ΠΎ, ΠΏΡ€ΠΈΡ‡Π΅ΠΌ Π±Π°Ρ‚Π°Ρ€Π΅Π΅ ΠΌΡ‹ сообщили Ρ‚ΠΎΠ»ΡŒΠΊΠΎ заряд .

    Разности ΠΏΠΎΡ‚Π΅Π½Ρ†ΠΈΠ°Π»ΠΎΠ² , ΠΈ Ρ‚. Π΄. ΠΌΠΎΠ³ΡƒΡ‚ Π±Ρ‹Ρ‚ΡŒ Π½Π΅ Ρ€Π°Π²Π½Ρ‹ ΠΌΠ΅ΠΆΠ΄Ρƒ собой, Ρ‚Π°ΠΊ ΠΊΠ°ΠΊ Смкости ΠΎΡ‚Π΄Π΅Π»ΡŒΠ½Ρ‹Ρ… кондСнсаторов, Π²ΠΎΠΎΠ±Ρ‰Π΅ говоря, Π½Π΅ΠΎΠ΄ΠΈΠ½Π°ΠΊΠΎΠ²Ρ‹. ΠŸΠΎΡΡ‚ΠΎΠΌΡƒ Ρ€Π°Π·Π½ΠΎΡΡ‚ΡŒ ΠΏΠΎΡ‚Π΅Π½Ρ†ΠΈΠ°Π»ΠΎΠ² Π½Π° ΠΊΠ»Π΅ΠΌΠΌΠ°Ρ… всСй Π±Π°Ρ‚Π°Ρ€Π΅ΠΈ находится ΠΊΠ°ΠΊ сумма напряТСний Π½Π° ΠΊΠ°ΠΆΠ΄ΠΎΠΌ ΠΈΠ· кондСнсаторов:

    Π‘ Π΄Ρ€ΡƒΠ³ΠΎΠΉ стороны,

    Π³Π΄Π΅ Π΅ΠΌΠΊΠΎΡΡ‚ΡŒ всСй Π±Π°Ρ‚Π°Ρ€Π΅ΠΈ. Π‘Π»Π΅Π΄ΠΎΠ²Π°Ρ‚Π΅Π»ΡŒΠ½ΠΎ, Π΅ΠΌΠΊΠΎΡΡ‚ΡŒ Π±Π°Ρ‚Π°Ρ€Π΅ΠΈ ΠΏΠΎΡΠ»Π΅Π΄ΠΎΠ²Π°Ρ‚Π΅Π»ΡŒΠ½ΠΎ соСдинСнных кондСнсаторов опрСдСляСтся Π²Ρ‹Ρ€Π°ΠΆΠ΅Π½ΠΈΠ΅ΠΌ:

    Для Π±Π°Ρ‚Π°Ρ€Π΅ΠΈ ΠΈΠ· Π΄Π²ΡƒΡ… кондСнсаторов, Π½Π°ΠΏΡ€ΠΈΠΌΠ΅Ρ€, ΠΎΡ‚ΡΡŽΠ΄Π° слСдуСт Π²Ρ‹Ρ€Π°ΠΆΠ΅Π½ΠΈΠ΅ (рис. 2.14)


    Рис. 2.14. ΠŸΠΎΡΠ»Π΅Π΄ΠΎΠ²Π°Ρ‚Π΅Π»ΡŒΠ½ΠΎΠ΅ соСдинСниС Π΄Π²ΡƒΡ… кондСнсаторов

    ΠŸΠ°Ρ€Π°Π»Π»Π΅Π»ΡŒΠ½ΠΎΠ΅ соСдинСниС

    Рис. 2.15. ΠŸΠ°Ρ€Π°Π»Π»Π΅Π»ΡŒΠ½ΠΎΠ΅ соСдинСниС кондСнсаторов

    ΠŸΡ€ΠΈ ΠΏΠ°Ρ€Π°Π»Π»Π΅Π»ΡŒΠ½ΠΎΠΌ соСдинСнии кондСнсаторов (рис. 2.15) Ρ€Π°Π·Π½ΠΎΡΡ‚ΡŒ ΠΏΠΎΡ‚Π΅Π½Ρ†ΠΈΠ°Π»ΠΎΠ² Π±Π°Ρ‚Π°Ρ€Π΅ΠΈ Ρ€Π°Π²Π½Π° разности ΠΏΠΎΡ‚Π΅Π½Ρ†ΠΈΠ°Π»ΠΎΠ² ΠΊΠ°ΠΆΠ΄ΠΎΠ³ΠΎ ΠΎΡ‚Π΄Π΅Π»ΡŒΠ½ΠΎΠ³ΠΎ кондСнсатора:

    ЗаряТая Ρ‚Π°ΠΊΡƒΡŽ Π±Π°Ρ‚Π°Ρ€Π΅ΡŽ, ΠΌΡ‹ сообщаСм Π΅ΠΉ заряд, Ρ‡Π°ΡΡ‚ΡŒ ΠΊΠΎΡ‚ΠΎΡ€ΠΎΠ³ΠΎ ΠΏΠΎΠΏΠ°Π΄Π΅Ρ‚ Π½Π° ΠΎΠ±ΠΊΠ»Π°Π΄ΠΊΠΈ ΠΏΠ΅Ρ€Π²ΠΎΠ³ΠΎ кондСнсатора, Ρ‡Π°ΡΡ‚ΡŒ — Π½Π° ΠΎΠ±ΠΊΠ»Π°Π΄ΠΊΠΈ Π²Ρ‚ΠΎΡ€ΠΎΠ³ΠΎ ΠΈ Ρ‚. Π΄. ВслСдствиС Π·Π°ΠΊΠΎΠ½Π° сохранСния элСктричСского заряда ΠΏΠΎΠ»Π½Ρ‹ΠΉ заряд Π±Π°Ρ‚Π°Ρ€Π΅ΠΈ ΠΏΠ°Ρ€Π°Π»Π»Π΅Π»ΡŒΠ½ΠΎ соСдинСнных кондСнсаторов Π±ΡƒΠ΄Π΅Ρ‚ Ρ€Π°Π²Π΅Π½ суммС зарядов ΠΎΡ‚Π΄Π΅Π»ΡŒΠ½Ρ‹Ρ… кондСнсаторов:

    Для ΠΊΠ°ΠΆΠ΄ΠΎΠ³ΠΎ кондСнсатора ΠΌΠΎΠΆΠ½ΠΎ Π½Π°ΠΏΠΈΡΠ°Ρ‚ΡŒ ΡΠΎΠΎΡ‚Π½ΠΎΡˆΠ΅Π½ΠΈΠ΅

    подставляя ΠΊΠΎΡ‚ΠΎΡ€ΠΎΠ΅ Π² (2.25), ΠΏΠΎΠ»ΡƒΡ‡ΠΈΠΌ:

    Π‘ Π΄Ρ€ΡƒΠ³ΠΎΠΉ стороны,

    Π³Π΄Π΅ Π΅ΠΌΠΊΠΎΡΡ‚ΡŒ всСй Π±Π°Ρ‚Π°Ρ€Π΅ΠΈ. Бравнивая (2.27) ΠΈ (2.28) ΠΎΠΊΠΎΠ½Ρ‡Π°Ρ‚Π΅Π»ΡŒΠ½ΠΎ ΠΏΠΎΠ»ΡƒΡ‡Π°Π΅ΠΌ

    Ρ‚ΠΎ Π΅ΡΡ‚ΡŒ ΠΏΡ€ΠΈ ΠΏΠ°Ρ€Π°Π»Π»Π΅Π»ΡŒΠ½ΠΎΠΌ соСдинСнии кондСнсаторов Π΅ΠΌΠΊΠΎΡΡ‚ΡŒ Π±Π°Ρ‚Π°Ρ€Π΅ΠΈ Ρ€Π°Π²Π½Π° суммС СмкостСй ΠΎΡ‚Π΄Π΅Π»ΡŒΠ½Ρ‹Ρ… кондСнсаторов. Для Π±Π°Ρ‚Π°Ρ€Π΅ΠΈ ΠΈΠ· Π΄Π²ΡƒΡ… кондСнсаторов, Π½Π°ΠΏΡ€ΠΈΠΌΠ΅Ρ€, ΠΎΡ‚ΡΡŽΠ΄Π° слСдуСт Π²Ρ‹Ρ€Π°ΠΆΠ΅Π½ΠΈΠ΅ (рис. 2.16)

    Рис. 2.16. ΠŸΠ°Ρ€Π°Π»Π»Π΅Π»ΡŒΠ½ΠΎΠ΅ соСдинСниС Π΄Π²ΡƒΡ… кондСнсаторов

    Π—Π°Π΄Π°Ρ‡Π°. Π’ сфСричСский кондСнсатор с радиусами см Π²Π½ΡƒΡ‚Ρ€Π΅Π½Π½Π΅ΠΉ сфСры ΠΈ см внСшнСй сфСры помСстили ΡΠΏΠ»ΠΎΡˆΠ½ΡƒΡŽ ΡΡ„Π΅Ρ€ΠΈΡ‡Π΅ΡΠΊΡƒΡŽ ΠΏΡ€ΠΎΠ²ΠΎΠ΄ΡΡ‰ΡƒΡŽ ΠΎΠ±ΠΎΠ»ΠΎΡ‡ΠΊΡƒ с Π²Π½ΡƒΡ‚Ρ€Π΅Π½Π½ΠΈΠΌ см ΠΈ внСшним см радиусами (рис. 2.17). Π‘Ρ€Π°Π²Π½ΠΈΡ‚ΡŒ Смкости ΠΏΡ€Π΅ΠΆΠ½Π΅Π³ΠΎ ΠΈ Π½ΠΎΠ²ΠΎΠ³ΠΎ кондСнсаторов.

    ΠšΠΎΠ½Π΄Π΅Π½ΡΠ°Ρ‚ΠΎΡ€ — ΠΎΡ‡Π΅Π½ΡŒ распространённая Ρ€Π°Π΄ΠΈΠΎΠ΄Π΅Ρ‚Π°Π»ΡŒ, которая встрСчаСтся Π²ΠΎ всСх ΠΏΡ€ΠΈΠ½Ρ†ΠΈΠΏΠΈΠ°Π»ΡŒΠ½Ρ‹Ρ… схСмах. Он прСдставляСт собой Π΄Π²Π° ΠΏΡ€ΠΎΠ²ΠΎΠ΄Π½ΠΈΠΊΠ°, Ρ€Π°Π·Π΄Π΅Π»Ρ‘Π½Π½Ρ‹Ρ… диэлСктриком (Π² зависимости ΠΎΡ‚ Ρ‚ΠΈΠΏΠ° кондСнсаторов ΠΏΡ€ΠΈΠΌΠ΅Π½ΡΡŽΡ‚ΡΡ Ρ€Π°Π·Π»ΠΈΡ‡Π½Ρ‹Π΅ Π΅Π³ΠΎ Ρ‚ΠΈΠΏΡ‹), Ρ‚ΠΎ Π΅ΡΡ‚ΡŒ физичСски это Ρ€Π°Π·Ρ€Ρ‹Π² Ρ†Π΅ΠΏΠΈ, Π½ΠΎ Π² диэлСктрикС ΠΌΠΎΠΆΠ΅Ρ‚ Π½Π°ΠΊΠ°ΠΏΠ»ΠΈΠ²Π°Ρ‚ΡŒΡΡ заряд. Основной характСристикой любого кондСнсатора слуТит ΡΠΏΠΎΡΠΎΠ±Π½ΠΎΡΡ‚ΡŒ Π½Π°ΠΊΠ°ΠΏΠ»ΠΈΠ²Π°Ρ‚ΡŒ заряд — Ρ‘ΠΌΠΊΠΎΡΡ‚ΡŒ, ΠΈ этого заряда.ЭлСктролитичСскиС кондСнсаторы ΠΈΠΌΠ΅ΡŽΡ‚ ΠΏΠΎΠ»ΡΡ€Π½ΠΎΡΡ‚ΡŒ ΠΈ Ρ…Π°Ρ€Π°ΠΊΡ‚Π΅Ρ€ΠΈΠ·ΡƒΡŽΡ‚ΡΡ большой Ρ‘ΠΌΠΊΠΎΡΡ‚ΡŒΡŽ ΠΈ ΡˆΠΈΡ€ΠΎΠΊΠΈΠΌ Π΄ΠΈΠ°ΠΏΠ°Π·ΠΎΠ½ΠΎΠΌ напряТСний, Π±ΡƒΠΌΠ°ΠΆΠ½Ρ‹Π΅ Π²Ρ‹Π΄Π΅Ρ€ΠΆΠΈΠ²Π°ΡŽΡ‚ большоС напряТСниС, Π½ΠΎ ΠΈΠΌΠ΅ΡŽΡ‚ Π½Π΅Π±ΠΎΠ»ΡŒΡˆΡƒΡŽ Ρ‘ΠΌΠΊΠΎΡΡ‚ΡŒ. Π‘ΡƒΡ‰Π΅ΡΡ‚Π²ΡƒΡŽΡ‚ ΠΈ ΠΏΡ€ΠΈΠ±ΠΎΡ€Ρ‹ с ΠΈΠ·ΠΌΠ΅Π½ΡΡŽΡ‰Π΅ΠΉΡΡ Ρ‘ΠΌΠΊΠΎΡΡ‚ΡŒΡŽ, Π½ΠΎ ΠΊΠ°ΠΆΠ΄ΠΎΠΌΡƒ Ρ‚ΠΈΠΏΡƒ — своё ΠΏΡ€ΠΈΠΌΠ΅Π½Π΅Π½ΠΈΠ΅.

    Часто Ρ€Π°Π΄ΠΈΠΎΠ»ΡŽΠ±ΠΈΡ‚Π΅Π»ΠΈ ΡΡ‚Π°Π»ΠΊΠΈΠ²Π°ΡŽΡ‚ΡΡ с ΠΏΡ€ΠΎΠ±Π»Π΅ΠΌΠΎΠΉ ΠΏΠΎΠ΄Π±ΠΎΡ€Π° кондСнсаторов ΠΏΠΎ ёмкости ΠΈΠ»ΠΈ Π½Π°ΠΏΡ€ΡΠΆΠ΅Π½ΠΈΡŽ. ΠŸΡ€ΠΎΡ„Π΅ΡΡΠΈΠΎΠ½Π°Π»Ρ‹ Π·Π½Π°ΡŽΡ‚: Π² случаС отсутствия Π½ΡƒΠΆΠ½ΠΎΠ³ΠΎ, ΠΌΠΎΠΆΠ½ΠΎ ΡΠΎΠ±Ρ€Π°Ρ‚ΡŒ ΠΊΠΎΠΌΠ±ΠΈΠ½Π°Ρ†ΠΈΡŽ ΠΈΠ· Π½Π΅ΡΠΊΠΎΠ»ΡŒΠΊΠΈΡ… ΠΏΡ€ΠΈΠ±ΠΎΡ€ΠΎΠ², Π±Π°Ρ‚Π°Ρ€Π΅ΡŽ ΠΈΠ· Π½ΠΈΡ…. Π’ батарСях допускаСтся ΠΊΠΎΠΌΠ±ΠΈΠ½ΠΈΡ€ΠΎΠ²Π°Π½Π½ΠΎΠ΅, кондСнсаторов.

    БоСдиняя ΠΏΡ€ΠΈΠ±ΠΎΡ€Ρ‹ ΠΏΠ°Ρ€Π°Π»Π»Π΅Π»ΡŒΠ½ΠΎ, ΠΌΠΎΠΆΠ½ΠΎ Π΄ΠΎΠ±ΠΈΡ‚ΡŒΡΡ ΡƒΠ²Π΅Π»ΠΈΡ‡Π΅Π½ΠΈΠ΅ ёмкости. ΠžΠ±Ρ‰Π°Ρ Π² Ρ‚Π°ΠΊΠΎΠΉ Π±Π°Ρ‚Π°Ρ€Π΅Π΅ Π±ΡƒΠ΄Π΅Ρ‚ Ρ€Π°Π²Π½Π° суммС всСх СмкостСй (Бэкв.=Π‘1+Π‘2+…), напряТСниС Π½Π° ΠΊΠ°ΠΆΠ΄ΠΎΠΌ элСмСнтС Π±ΡƒΠ΄Π΅Ρ‚ Ρ€Π°Π²Π½Ρ‹ΠΌ. Π­Ρ‚ΠΎ ΠΎΠ·Π½Π°Ρ‡Π°Π΅Ρ‚, Ρ‡Ρ‚ΠΎ минимальноС напряТСниС кондСнсатора, ΠΏΡ€ΠΈΠΌΠ΅Π½Ρ‘Π½Π½ΠΎΠ³ΠΎ Π² соСдинСнии, являСтся максимально допустимым для всСй Π±Π°Ρ‚Π°Ρ€Π΅ΠΈ.

    ΠŸΠΎΡΠ»Π΅Π΄ΠΎΠ²Π°Ρ‚Π΅Π»ΡŒΠ½ΠΎΠ΅ соСдинСниС кондСнсаторов примСняСтся Π² случаС, ΠΊΠΎΠ³Π΄Π° Π½Π΅ΠΎΠ±Ρ…ΠΎΠ΄ΠΈΠΌΠΎ ΡƒΠ²Π΅Π»ΠΈΡ‡ΠΈΡ‚ΡŒ напряТСниС, способноС Π²Ρ‹Π΄Π΅Ρ€ΠΆΠ°Ρ‚ΡŒ ΠΏΡ€ΠΈΠ±ΠΎΡ€Ρ‹ ΠΈΠ»ΠΈ ΡΠ½ΠΈΠ·ΠΈΡ‚ΡŒ ΠΈΡ… Ρ‘ΠΌΠΊΠΎΡΡ‚ΡŒ.Π’ Ρ‚Π°ΠΊΠΎΠΌ Π²Π°Ρ€ΠΈΠ°Π½Ρ‚Π΅ элСмСнты ΡΠΎΠ΅Π΄ΠΈΠ½ΡΡŽΡ‚ΡΡ ΠΏΠΎ ΡΠ»Π΅Π΄ΡƒΡŽΡ‰Π΅ΠΉ схСмС: Π½Π°Ρ‡Π°Π»ΠΎ ΠΎΠ΄Π½ΠΎΠ³ΠΎ с ΠΊΠΎΠ½Ρ†ΠΎΠΌ Π΄Ρ€ΡƒΠ³ΠΎΠ³ΠΎ, Ρ‚ΠΎ Π΅ΡΡ‚ΡŒ «ΠΏΠ»ΡŽΡ» ΠΎΠ΄Π½ΠΎΠ³ΠΎ с «ΠΌΠΈΠ½ΡƒΡΠΎΠΌ» Π΄Ρ€ΡƒΠ³ΠΎΠ³ΠΎ. ΠΠΌΠΊΠΎΡΡ‚ΡŒ эквивалСнтного кондСнсатора Π² этом случаС вычисляСтся ΠΏΠΎ Ρ‚Π°ΠΊΠΎΠΉ Ρ„ΠΎΡ€ΠΌΡƒΠ»Π΅: 1/Бэкв.=1/Π‘1+1/Π‘2+… Из этго слСдуСт, Ρ‡Ρ‚ΠΎ для Π΄Π²ΡƒΡ… кондСнсаторов Бэкв=Π‘1*Π‘2/(Π‘1+Π‘2), Π° Π·Π½Π°Ρ‡ΠΈΡ‚, Ρ‘ΠΌΠΊΠΎΡΡ‚ΡŒ Π±Π°Ρ‚Π°Ρ€Π΅ΠΈ Π±ΡƒΠ΄Π΅Ρ‚ мСньшС минимальной ёмкости, использованной Π² Π½Π΅ΠΉ.

    БатарСя кондСнсаторов часто прСдусматриваСт ΠΊΠΎΠΌΠ±ΠΈΠ½ΠΈΡ€ΠΎΠ²Π°Π½Π½ΠΎΠ΅ (смСшанноС)
    соСдинСниС. Для расчёта ёмкости Ρ‚Π°ΠΊΠΎΠ³ΠΎ устройства, Π² ΠΊΠΎΡ‚ΠΎΡ€ΠΎΠΌ ΠΏΡ€ΠΈΠΌΠ΅Π½Π΅Π½ΠΎ ΠΏΠ°Ρ€Π°Π»Π»Π΅Π»ΡŒΠ½ΠΎΠ΅ ΠΈ ΠΏΠΎΡΠ»Π΅Π΄ΠΎΠ²Π°Ρ‚Π΅Π»ΡŒΠ½ΠΎΠ΅ соСдинСниС кондСнсаторов, схСму Ρ€Π°Π·Π±ΠΈΠ²Π°ΡŽΡ‚ Π½Π° участки, Π·Π°Ρ‚Π΅ΠΌ ΠΏΠΎΠΎΡ‡Π΅Ρ€Ρ‘Π΄Π½ΠΎ Π²Ρ‹Ρ‡ΠΈΡΠ»ΡΡŽΡ‚ Ρ‘ΠΌΠΊΠΎΡΡ‚ΡŒ ΠΊΠ°ΠΆΠ΄ΠΎΠ³ΠΎ ΠΈΠ· Π½ΠΈΡ…. Π’Π°ΠΊ, вычисляСтся Ρ‘ΠΌΠΊΠΎΡΡ‚ΡŒ Π‘12=Π‘1+Π‘2, Π° Π·Π°Ρ‚Π΅ΠΌ Бэкв=Π‘12*Π‘3/(Π‘12+Π‘3).


    Благодаря созданию кондСнсаторных Π±Π°Ρ‚Π°Ρ€Π΅ΠΉ с Ρ€Π°Π·Π»ΠΈΡ‡Π½ΠΎΠΉ ΠΊΠΎΠ½Ρ„ΠΈΠ³ΡƒΡ€Π°Ρ†ΠΈΠ΅ΠΉ ΠΈ схСмой
    соСдинСния, ΠΌΠΎΠΆΠ½ΠΎ ΠΏΠΎΠ΄ΠΎΠ±Ρ€Π°Ρ‚ΡŒ Π»ΡŽΠ±ΡƒΡŽ Ρ‘ΠΌΠΊΠΎΡΡ‚ΡŒ Π½Π° любоС ΠΈΠ½Ρ‚Π΅Ρ€Π΅ΡΡƒΡŽΡ‰Π΅Π΅ напряТСниС. кондСнсаторов, ΠΊΠ°ΠΊ ΠΈ ΠΊΠΎΠΌΠ±ΠΈΠ½ΠΈΡ€ΠΎΠ²Π°Π½Π½ΠΎΠ΅, примСняСтся Π²ΠΎ ΠΌΠ½ΠΎΠ³ΠΈΡ… Π³ΠΎΡ‚ΠΎΠ²Ρ‹Ρ… Ρ€Π°Π΄ΠΈΠΎΠ»ΡŽΠ±ΠΈΡ‚Π΅Π»ΡŒΡΠΊΠΈΡ… схСмах. ΠŸΡ€ΠΈ этом ΠΎΠ±ΡΠ·Π°Ρ‚Π΅Π»ΡŒΠ½ΠΎ учитываСтся Ρ‚ΠΎ, Ρ‡Ρ‚ΠΎ ΠΊΠ°ΠΆΠ΄Ρ‹ΠΉ кондСнсатор ΠΈΠΌΠ΅Π΅Ρ‚ ΠΎΡ‡Π΅Π½ΡŒ Π²Π°ΠΆΠ½Ρ‹ΠΉ ΠΈΠ½Π΄ΠΈΠ²ΠΈΠ΄ΡƒΠ°Π»ΡŒΠ½Ρ‹ΠΉ ΠΏΠ°Ρ€Π°ΠΌΠ΅Ρ‚Ρ€ — Ρ‚ΠΎΠΊ ΡƒΡ‚Π΅Ρ‡ΠΊΠΈ, ΠΎΠ½ ΠΌΠΎΠΆΠ΅Ρ‚ Ρ€Π°Π·Π±Π°Π»Π°Π½ΡΠΈΡ€ΠΎΠ²Π°Ρ‚ΡŒ напряТСниС ΠΏΡ€ΠΈ ΠΏΠ°Ρ€Π°Π»Π»Π΅Π»ΡŒΠ½ΠΎΠΌ соСдинСнии ΠΈ Ρ‘ΠΌΠΊΠΎΡΡ‚ΡŒ ΠΏΡ€ΠΈ ΠΏΠΎΡΠ»Π΅Π΄ΠΎΠ²Π°Ρ‚Π΅Π»ΡŒΠ½ΠΎΠΌ. ΠžΡ‡Π΅Π½ΡŒ Π²Π°ΠΆΠ½ΠΎ ΠΏΠΎΠ΄Π±ΠΈΡ€Π°Ρ‚ΡŒ Π½Π΅ΠΎΠ±Ρ…ΠΎΠ΄ΠΈΠΌΠΎΠ΅ сопротивлСниС ΡˆΡƒΠ½Ρ‚Π°.

    ΠŸΡ€ΠΈ Ρ€Π°Π±ΠΎΡ‚Π΅ с кондСнсаторами ΠΈ элСктроникой, Π½Π΅ Π·Π°Π±Ρ‹Π²Π°ΠΉΡ‚Π΅ ΠΎ ΠΏΡ€Π°Π²ΠΈΠ»Π°Ρ… Π»ΠΈΡ‡Π½ΠΎΠΉ бСзопасности ΠΈ ΡƒΠ³Ρ€ΠΎΠ·Π΅ пораТСния Ρ‚ΠΎΠΊΠΎΠΌ.

    Π—Π°Π΄Π°Ρ‡ΠΈ

    Π—Π°Π΄Π°Ρ‡ΠΈ ΠΊ ΡƒΡ€ΠΎΠΊΡƒ 50/14

    1.Β Β Β Β Β  ΠšΠΎΡΠΌΠΈΡ‡Π΅ΡΠΊΠ°Ρ Ρ€Π°ΠΊΠ΅Ρ‚Π° ΠΏΡ€ΠΈ стартС с Π—Π΅ΠΌΠ»ΠΈ двиТСтся Π²Π΅Ρ€Ρ‚ΠΈΠΊΠ°Π»ΡŒΠ½ΠΎ Π²Π²Π΅Ρ€Ρ… с ускорСниСм a = 25 ΠΌ/с2. ΠžΠΏΡ€Π΅Π΄Π΅Π»ΠΈΡ‚Π΅ вСс космонавта массой m = 100 ΠΊΠ³. УскорСниС свободного падСния ΡΡ‡ΠΈΡ‚Π°Ρ‚ΡŒ Ρ€Π°Π²Π½Ρ‹ΠΌ 10 ΠΌ/с2.

    2.Β Β Β Β Β  ΠŸΠ°Ρ€Π°ΡˆΡŽΡ‚ΠΈΡΡ‚, достигнув Π² затяТном ΠΏΡ€Ρ‹ΠΆΠΊΠ΅ скорости Ο…1 = 60 ΠΌ/с, раскрыл ΠΏΠ°Ρ€Π°ΡˆΡŽΡ‚, послС Ρ‡Π΅Π³ΠΎ Π΅Π³ΠΎ ΡΠΊΠΎΡ€ΠΎΡΡ‚ΡŒ Π·Π° t = 2 с ΡƒΠΌΠ΅Π½ΡŒΡˆΠΈΠ»Π°ΡΡŒ Π΄ΠΎ Ο…2 = 10 ΠΌ/с. Π§Π΅ΠΌΡƒ Ρ€Π°Π²Π΅Π½ вСс ΠΏΠ°Ρ€Π°ΡˆΡŽΡ‚ΠΈΡΡ‚Π° массой m = 70 ΠΊΠ³ Π²ΠΎ врСмя тормоТСния? УскорСниС свободного падСния ΡΡ‡ΠΈΡ‚Π°Ρ‚ΡŒ Ρ€Π°Π²Π½Ρ‹ΠΌ 10 ΠΌ/с2.

    3.Β Β Β Β Β  Π‘Π°ΠΌΠΎΠ»Π΅Ρ‚, двигаясь с постоянной ΡΠΊΠΎΡ€ΠΎΡΡ‚ΡŒΡŽ 720 ΠΊΠΌ/Ρ‡, ΡΠΎΠ²Π΅Ρ€ΡˆΠ°Π΅Ρ‚ Ρ„ΠΈΠ³ΡƒΡ€Ρƒ Π²Ρ‹ΡΡˆΠ΅Π³ΠΎ ΠΏΠΈΠ»ΠΎΡ‚Π°ΠΆΠ° – Β«ΠΌΠ΅Ρ€Ρ‚Π²ΡƒΡŽ ΠΏΠ΅Ρ‚Π»ΡŽΒ» – радиусом 1000 ΠΌ. Π§Π΅ΠΌΡƒ Ρ€Π°Π²Π½Π° ΠΏΠ΅Ρ€Π΅Π³Ρ€ΡƒΠ·ΠΊΠ° Π»Π΅Ρ‚Ρ‡ΠΈΠΊΠ° Π² Π²Π΅Ρ€Ρ…Π½Π΅ΠΉ Ρ‚ΠΎΡ‡ΠΊΠ΅ ΠΏΠ΅Ρ‚Π»ΠΈ? (g = 10 ΠΌ/с2).

    Β 

    Π—Π°Π΄Π°Ρ‡ΠΈ Π΄/Π· ΠΊ ΡƒΡ€ΠΎΠΊΡƒ 48/12

    1.Β Β Β Β Β Β Β Β  Π’ΠΎ сколько Ρ€Π°Π· измСнится сила ВсСмирного тяготСния, Ссли массу ΠΎΠ΄Π½ΠΎΠ³ΠΎ Ρ‚Π΅Π»Π° ΡƒΠ²Π΅Π»ΠΈΡ‡ΠΈΡ‚ΡŒ Π² 3 Ρ€Π°Π·Π°, Π° Π΄Ρ€ΡƒΠ³ΠΎΠ³ΠΎ ΡƒΠΌΠ΅Π½ΡŒΡˆΠΈΡ‚ΡŒ Π² 9 Ρ€Π°Π·?

    2.Β Β Β Β Β Β Β Β  Π’ΠΎ сколько Ρ€Π°Π· измСнится сила ВсСмирного тяготСния, Ссли расстояниС ΠΌΠ΅ΠΆΠ΄Ρƒ Ρ‚Π΅Π»Π°ΠΌΠΈ ΡƒΠΌΠ΅Π½ΡŒΡˆΠΈΡ‚ΡŒ Π² 5 Ρ€Π°Π·?

    3.Β Β Β Β Β Β Β Β  Π‘ ΠΊΠ°ΠΊΠΈΠΌ ускорСниСм всплываСт Ρ‚Π΅Π»ΠΎ массой 25 ΠΊΠ³, Ссли Π½Π° Π½Π΅Π³ΠΎ дСйствуСт сила АрхимСда 300 Н?

    Π—Π°Π΄Π°Ρ‡ΠΈ Π΄/Π· ΠΊ ΡƒΡ€ΠΎΠΊΡƒ 60 Β 

    1. ΠŸΠΎΡ‡Π΅ΠΌΡƒ Π½Π΅Π²ΠΎΠ·ΠΌΠΎΠΆΠ½ΠΎ, ΠΈΠ· полоТСния сидя прямо Π½Π° стулС, Π²ΡΡ‚Π°Ρ‚ΡŒ Π½Π° Π½ΠΎΠ³ΠΈ, Π½Π΅ наклонившись ΠΏΡ€Π΅Π΄Π²Π°Ρ€ΠΈΡ‚Π΅Π»ΡŒΠ½ΠΎ Π²ΠΏΠ΅Ρ€Π΅Π΄?

    2. ΠŸΠΎΡ‡Π΅ΠΌΡƒ ΠΎΠ΄Π½ΠΎΡ€ΠΎΠ΄Π½Ρ‹ΠΉ ΠΏΡ€ΡΠΌΠΎΡƒΠ³ΠΎΠ»ΡŒΠ½Ρ‹ΠΉ ΠΊΠΈΡ€ΠΏΠΈΡ‡ ΠΌΠΎΠΆΠ½ΠΎ ΠΏΠΎΠ»ΠΎΠΆΠΈΡ‚ΡŒ Π½Π° ΠΊΡ€Π°ΠΉ стола, Ρ‚ΠΎΠ»ΡŒΠΊΠΎ Ссли с края стола свисаСт Π½Π΅ Π±ΠΎΠ»Π΅Π΅ ΠΏΠΎΠ»ΠΎΠ²ΠΈΠ½Ρ‹ Π΄Π»ΠΈΠ½Ρ‹ ΠΊΠΈΡ€ΠΏΠΈΡ‡Π°?

    3. ΠŸΠΎΡ‡Π΅ΠΌΡƒ Π²Ρ‹ Π²Ρ‹Π½ΡƒΠΆΠ΄Π΅Π½Ρ‹ ΠΎΡ‚ΠΊΠ»ΠΎΠ½ΡΡ‚ΡŒΡΡ Π½Π°Π·Π°Π΄, ΠΊΠΎΠ³Π΄Π° нСсСтС Π² Ρ€ΡƒΠΊΠ°Ρ… тяТСлый Π³Ρ€ΡƒΠ·?

    Π—Π°Π΄Π°Ρ‡ΠΈ Π΄/Π· ΠΊ ΡƒΡ€ΠΎΠΊΡƒ 58/7Β 

    1. Какова срСдняя сила давлСния F Π½Π° ΠΏΠ»Π΅Ρ‡ΠΎ ΠΏΡ€ΠΈ ΡΡ‚Ρ€Π΅Π»ΡŒΠ±Π΅ ΠΈΠ· Π°Π²Ρ‚ΠΎΠΌΠ°Ρ‚Π°, Ссли масса ΠΏΡƒΠ»ΠΈ m = 10 Π³, Π° ΡΠΊΠΎΡ€ΠΎΡΡ‚ΡŒ ΠΏΡƒΠ»ΠΈ ΠΏΡ€ΠΈ Π²Ρ‹Π»Π΅Ρ‚Π΅ ΠΈΠ· ΠΊΠ°Π½Π°Π»Π° ствола v = 300 ΠΌ/с? Автомат Π΄Π΅Π»Π°Π΅Ρ‚ 300 выстрСлов Π² ΠΌΠΈΠ½ΡƒΡ‚Ρƒ.

    2. Для провСдСния ΠΎΠ³Π½Π΅Π²Ρ‹Ρ… испытаний Тидкостный Ρ€Π°ΠΊΠ΅Ρ‚Π½Ρ‹ΠΉ Π΄Π²ΠΈΠ³Π°Ρ‚Π΅Π»ΡŒ Π·Π°ΠΊΡ€Π΅ΠΏΠΈΠ»ΠΈ Π½Π° стСндС. Π‘ ΠΊΠ°ΠΊΠΎΠΉ силой ΠΎΠ½ дСйствуСт Π½Π° стСнд, Ссли ΡΠΊΠΎΡ€ΠΎΡΡ‚ΡŒ истСчСния ΠΏΡ€ΠΎΠ΄ΡƒΠΊΡ‚ΠΎΠ² сгорания ΠΈΠ· сопла 150 ΠΌ/с, Π° расход Ρ‚ΠΎΠΏΠ»ΠΈΠ²Π° Π·Π° 5 сСкунд составил 30 ΠΊΠ³?

    3. Π Π°ΠΊΠ΅Ρ‚Π° массой 1000 ΠΊΠ³ Π½Π΅ΠΏΠΎΠ΄Π²ΠΈΠΆΠ½ΠΎ зависла Π½Π°Π΄ ΠΏΠΎΠ²Π΅Ρ€Ρ…Π½ΠΎΡΡ‚ΡŒΡŽ Π·Π΅ΠΌΠ»ΠΈ. Бколько Ρ‚ΠΎΠΏΠ»ΠΈΠ²Π° Π² Π΅Π΄ΠΈΠ½ΠΈΡ†Ρƒ Π²Ρ€Π΅ΠΌΠ΅Π½ΠΈ сТигаСт Ρ€Π°ΠΊΠ΅Ρ‚Π°, Ссли ΡΠΊΠΎΡ€ΠΎΡΡ‚ΡŒ истСчСния ΠΏΡ€ΠΎΠ΄ΡƒΠΊΡ‚ΠΎΠ² сгорания ΠΈΠ· Ρ€Π°ΠΊΠ΅Ρ‚Ρ‹ Ρ€Π°Π²Π½Π° 2 ΠΊΠΌ/с?

    Π•ΠΌΠΊΠΎΡΡ‚ΡŒ кондСнсатора Formula

    Π•ΠΌΠΊΠΎΡΡ‚ΡŒ кондСнсатора — это ΡΠΏΠΎΡΠΎΠ±Π½ΠΎΡΡ‚ΡŒ кондСнсатора Π½Π°ΠΊΠ°ΠΏΠ»ΠΈΠ²Π°Ρ‚ΡŒ элСктричСский заряд Π½Π° Π΅Π΄ΠΈΠ½ΠΈΡ†Ρƒ напряТСния Π½Π° своих пластинах кондСнсатора. Π•ΠΌΠΊΠΎΡΡ‚ΡŒ опрСдСляСтся Π΄Π΅Π»Π΅Π½ΠΈΠ΅ΠΌ элСктричСского заряда Π½Π° напряТСниС ΠΏΠΎ Ρ„ΠΎΡ€ΠΌΡƒΠ»Π΅ C = Q / V. Π•Π³ΠΎ Π΅Π΄ΠΈΠ½ΠΈΡ†Π° — Π€Π°Ρ€Π°Π΄.

    Π€ΠΎΡ€ΠΌΡƒΠ»Π°

    Π•Π³ΠΎ Ρ„ΠΎΡ€ΠΌΡƒΠ»Π° выглядит Ρ‚Π°ΠΊ:

    C = Q / V

    Π“Π΄Π΅ C — Π΅ΠΌΠΊΠΎΡΡ‚ΡŒ, Q — напряТСниС, Π° V — напряТСниС. ΠœΡ‹ Ρ‚Π°ΠΊΠΆΠ΅ ΠΌΠΎΠΆΠ΅ΠΌ Π½Π°ΠΉΡ‚ΠΈ заряд Q ΠΈ напряТСниС V, пСрСписав ΠΏΡ€ΠΈΠ²Π΅Π΄Π΅Π½Π½ΡƒΡŽ Π²Ρ‹ΡˆΠ΅ Ρ„ΠΎΡ€ΠΌΡƒΠ»Ρƒ ΠΊΠ°ΠΊ:

    Q =

    CV

    Π’ = Q / C

    Π€Π°Ρ€Π°Π΄ — Π΅Π΄ΠΈΠ½ΠΈΡ†Π° измСрСния Смкости.Один Ρ„Π°Ρ€Π°Π΄ — это Π²Π΅Π»ΠΈΡ‡ΠΈΠ½Π° Смкости, ΠΊΠΎΠ³Π΄Π° ΠΎΠ΄ΠΈΠ½ ΠΊΡƒΠ»ΠΎΠ½ заряда хранится с ΠΎΠ΄Π½ΠΈΠΌ Π²ΠΎΠ»ΡŒΡ‚ Π½Π° пластинах.

    Π‘ΠΎΠ»ΡŒΡˆΠΈΠ½ΡΡ‚Π²ΠΎ кондСнсаторов, ΠΊΠΎΡ‚ΠΎΡ€Ρ‹Π΅ ΠΈΡΠΏΠΎΠ»ΡŒΠ·ΡƒΡŽΡ‚ΡΡ Π² элСктроникС, ΠΈΠΌΠ΅ΡŽΡ‚ значСния Смкости, ΡƒΠΊΠ°Π·Π°Π½Π½Ρ‹Π΅ Π² ΠΌΠΈΠΊΡ€ΠΎΡ„Π°Ρ€Π°Π΄Π°Ρ… (ΠΌΠΊΠ€) ΠΈ ΠΏΠΈΠΊΠΎΡ„Π°Ρ€Π°Π΄Π°Ρ… (ΠΏΠ€). ΠœΠΈΠΊΡ€ΠΎΡ„Π°Ρ€Π°Π΄ — это ΠΎΠ΄Π½Π° миллионная Ρ„Π°Ρ€Π°Π΄Π°, Π° ΠΏΠΈΠΊΠΎΡ„Π°Ρ€Π°Π΄ — ΠΎΠ΄Π½Π° триллионная Ρ„Π°Ρ€Π°Π΄Π°.

    КакиС Ρ„Π°ΠΊΡ‚ΠΎΡ€Ρ‹ Π²Π»ΠΈΡΡŽΡ‚ Π½Π° Π΅ΠΌΠΊΠΎΡΡ‚ΡŒ кондСнсатора?

    Зависит ΠΎΡ‚ ΡΠ»Π΅Π΄ΡƒΡŽΡ‰ΠΈΡ… Ρ„Π°ΠΊΡ‚ΠΎΡ€ΠΎΠ²:

    ΠŸΠ»ΠΎΡ‰Π°Π΄ΡŒ ΠΏΠ»ΠΈΡ‚

    Π•ΠΌΠΊΠΎΡΡ‚ΡŒ прямо ΠΏΡ€ΠΎΠΏΠΎΡ€Ρ†ΠΈΠΎΠ½Π°Π»ΡŒΠ½Π° физичСскому Ρ€Π°Π·ΠΌΠ΅Ρ€Ρƒ пластин, опрСдСляСмому ΠΏΠ»ΠΎΡ‰Π°Π΄ΡŒΡŽ пластины A.Π‘ΠΎΠ»ΡŒΡˆΠ°Ρ ΠΏΠ»ΠΎΡ‰Π°Π΄ΡŒ пластины Π΄Π°Π΅Ρ‚ Π±ΠΎΠ»ΡŒΡˆΡƒΡŽ Π΅ΠΌΠΊΠΎΡΡ‚ΡŒ ΠΈ ΠΌΠ΅Π½ΡŒΡˆΡƒΡŽ Π΅ΠΌΠΊΠΎΡΡ‚ΡŒ. На рисункС (Π°) ΠΏΠΎΠΊΠ°Π·Π°Π½ΠΎ, Ρ‡Ρ‚ΠΎ ΠΏΠ»ΠΎΡ‰Π°Π΄ΡŒ пластины кондСнсатора с ΠΏΠ°Ρ€Π°Π»Π»Π΅Π»ΡŒΠ½Ρ‹ΠΌΠΈ пластинами Ρ€Π°Π²Π½Π° ΠΏΠ»ΠΎΡ‰Π°Π΄ΠΈ ΠΎΠ΄Π½ΠΎΠΉ ΠΈΠ· пластин. Если пластины ΠΏΠ΅Ρ€Π΅ΠΌΠ΅Ρ‰Π°ΡŽΡ‚ΡΡ ΠΎΡ‚Π½ΠΎΡΠΈΡ‚Π΅Π»ΡŒΠ½ΠΎ Π΄Ρ€ΡƒΠ³ Π΄Ρ€ΡƒΠ³Π°, ΠΊΠ°ΠΊ ΠΏΠΎΠΊΠ°Π·Π°Π½ΠΎ Π½Π° рис (b), ΠΏΠ»ΠΎΡ‰Π°Π΄ΡŒ пСрСкрытия опрСдСляСт ΡΡ„Ρ„Π΅ΠΊΡ‚ΠΈΠ²Π½ΡƒΡŽ ΠΏΠ»ΠΎΡ‰Π°Π΄ΡŒ пластины. Π­Ρ‚ΠΎ ΠΈΠ·ΠΌΠ΅Π½Π΅Π½ΠΈΠ΅ эффСктивной ΠΏΠ»ΠΎΡ‰Π°Π΄ΠΈ пластины являСтся основным для ΠΎΠΏΡ€Π΅Π΄Π΅Π»Π΅Π½Π½ΠΎΠ³ΠΎ Ρ‚ΠΈΠΏΠ° ΠΏΠ΅Ρ€Π΅ΠΌΠ΅Π½Π½ΠΎΠ³ΠΎ кондСнсатора.

    Π’Π°Ρ€Π΅Π»ΠΊΠΈ Ρ€Π°Π·Π΄Π΅Π»ΠΈΡ‚Π΅Π»ΡŒΠ½Ρ‹Π΅

    `Π•ΠΌΠΊΠΎΡΡ‚ΡŒ ΠΎΠ±Ρ€Π°Ρ‚Π½ΠΎ ΠΏΡ€ΠΎΠΏΠΎΡ€Ρ†ΠΈΠΎΠ½Π°Π»ΡŒΠ½Π° Ρ€Π°ΡΡΡ‚ΠΎΡΠ½ΠΈΡŽ ΠΌΠ΅ΠΆΠ΄Ρƒ пластинами.Π Π°Π·Π΄Π΅Π»Π΅Π½ΠΈΠ΅ пластин ΠΎΠ±ΠΎΠ·Π½Π°Ρ‡Π΅Π½ΠΎ Π±ΡƒΠΊΠ²ΠΎΠΉ d, ΠΊΠ°ΠΊ ΠΏΠΎΠΊΠ°Π·Π°Π½ΠΎ Π½Π° рис. (А). Π§Π΅ΠΌ большС Ρ€Π°Π·Π΄Π΅Π»Π΅Π½ΠΈΠ΅ пластин, Ρ‚Π΅ΠΌ мСньшС Π΅ΠΌΠΊΠΎΡΡ‚ΡŒ, ΠΊΠ°ΠΊ ΠΏΠΎΠΊΠ°Π·Π°Π½ΠΎ Π½Π° рис. (B). Как ΠΎΠ±ΡΡƒΠΆΠ΄Π°Π»ΠΎΡΡŒ Ρ€Π°Π½Π΅Π΅, напряТСниС пробоя прямо ΠΏΡ€ΠΎΠΏΠΎΡ€Ρ†ΠΈΠΎΠ½Π°Π»ΡŒΠ½ΠΎ Ρ€Π°ΡΡΡ‚ΠΎΡΠ½ΠΈΡŽ ΠΌΠ΅ΠΆΠ΄Ρƒ пластинами. Π§Π΅ΠΌ дальшС Ρ€Π°Π·Π΄Π΅Π»Π΅Π½Ρ‹ пластины, Ρ‚Π΅ΠΌ большС напряТСниС пробоя .

    ДиэлСктричСская ΠΏΡ€ΠΎΠ½ΠΈΡ†Π°Π΅ΠΌΠΎΡΡ‚ΡŒ ΠΌΠ°Ρ‚Π΅Ρ€ΠΈΠ°Π»Π°

    Как извСстно, изоляционный ΠΌΠ°Ρ‚Π΅Ρ€ΠΈΠ°Π» ΠΌΠ΅ΠΆΠ΄Ρƒ пластинами кондСнсатора называСтся диэлСктриком. ДиэлСктричСскиС ΠΌΠ°Ρ‚Π΅Ρ€ΠΈΠ°Π»Ρ‹ ΠΈΠΌΠ΅ΡŽΡ‚ Ρ‚Π΅Π½Π΄Π΅Π½Ρ†ΠΈΡŽ ΡƒΠΌΠ΅Π½ΡŒΡˆΠ°Ρ‚ΡŒ напряТСниС ΠΌΠ΅ΠΆΠ΄Ρƒ пластинами ΠΏΡ€ΠΈ Π·Π°Π΄Π°Π½Π½ΠΎΠΌ зарядС ΠΈ, Ρ‚Π°ΠΊΠΈΠΌ ΠΎΠ±Ρ€Π°Π·ΠΎΠΌ, ΡƒΠ²Π΅Π»ΠΈΡ‡ΠΈΠ²Π°Ρ‚ΡŒ Π΅ΠΌΠΊΠΎΡΡ‚ΡŒ.Если напряТСниС фиксировано, ΠΈΠ·-Π·Π° наличия диэлСктрика ΠΌΠΎΠΆΠ΅Ρ‚ Ρ…Ρ€Π°Π½ΠΈΡ‚ΡŒΡΡ большС заряда, Ρ‡Π΅ΠΌ ΠΌΠΎΠΆΠ΅Ρ‚ Ρ…Ρ€Π°Π½ΠΈΡ‚ΡŒΡΡ Π±Π΅Π· диэлСктрика. ΠœΠ΅Ρ€Π° способности ΠΌΠ°Ρ‚Π΅Ρ€ΠΈΠ°Π»Π° ΡΠΎΠ·Π΄Π°Π²Π°Ρ‚ΡŒ элСктричСскоС ΠΏΠΎΠ»Π΅ называСтся диэлСктричСской постоянной ΠΈΠ»ΠΈ ΠΎΡ‚Π½ΠΎΡΠΈΡ‚Π΅Π»ΡŒΠ½ΠΎΠΉ диэлСктричСской ΠΏΡ€ΠΎΠ½ΠΈΡ†Π°Π΅ΠΌΠΎΡΡ‚ΡŒΡŽ ΠΈ обозначаСтся ΠΊΠ°ΠΊ? r .

    Π•ΠΌΠΊΠΎΡΡ‚ΡŒ прямо ΠΏΡ€ΠΎΠΏΠΎΡ€Ρ†ΠΈΠΎΠ½Π°Π»ΡŒΠ½Π° диэлСктричСской проницаСмости. ДиэлСктричСская ΠΏΡ€ΠΎΠ½ΠΈΡ†Π°Π΅ΠΌΠΎΡΡ‚ΡŒ Π²Π°ΠΊΡƒΡƒΠΌΠ° опрСдСляСтся ΠΊΠ°ΠΊ 1, Π° диэлСктричСская ΠΏΡ€ΠΎΠ½ΠΈΡ†Π°Π΅ΠΌΠΎΡΡ‚ΡŒ Π²ΠΎΠ·Π΄ΡƒΡ…Π° ΠΎΡ‡Π΅Π½ΡŒ Π±Π»ΠΈΠ·ΠΊΠ° ΠΊ 1. Π­Ρ‚ΠΈ значСния ΠΈΡΠΏΠΎΠ»ΡŒΠ·ΡƒΡŽΡ‚ΡΡ Π² качСствС справочных, Π° для всСх Π΄Ρ€ΡƒΠ³ΠΈΡ… ΠΌΠ°Ρ‚Π΅Ρ€ΠΈΠ°Π»ΠΎΠ² значСния ∈r ΡƒΠΊΠ°Π·Π°Π½Ρ‹ ΠΏΠΎ ΠΎΡ‚Π½ΠΎΡˆΠ΅Π½ΠΈΡŽ ΠΊ Ρ‚Π°ΠΊΠΎΠ²Ρ‹ΠΌ для Π²Π°ΠΊΡƒΡƒΠΌΠ° ΠΈΠ»ΠΈ Π²ΠΎΠ·Π΄ΡƒΡ…Π°.НапримСр, ΠΌΠ°Ρ‚Π΅Ρ€ΠΈΠ°Π» с Ξ΅r = 8 ΠΌΠΎΠΆΠ΅Ρ‚ ΠΈΠΌΠ΅Ρ‚ΡŒ Π΅ΠΌΠΊΠΎΡΡ‚ΡŒ Π² восСмь Ρ€Π°Π· Π±ΠΎΠ»ΡŒΡˆΡƒΡŽ, Ρ‡Π΅ΠΌ Ρƒ Π²ΠΎΠ·Π΄ΡƒΡ…Π°, ΠΏΡ€ΠΈ ΠΏΡ€ΠΎΡ‡ΠΈΡ… Ρ€Π°Π²Π½Ρ‹Ρ… условиях.

    ДиэлСктричСская ΠΏΡ€ΠΎΠ½ΠΈΡ†Π°Π΅ΠΌΠΎΡΡ‚ΡŒ ∈r Π±Π΅Π·Ρ€Π°Π·ΠΌΠ΅Ρ€Π½Π°, ΠΏΠΎΡΠΊΠΎΠ»ΡŒΠΊΡƒ являСтся ΠΎΡ‚Π½ΠΎΡΠΈΡ‚Π΅Π»ΡŒΠ½ΠΎΠΉ ΠΌΠ΅Ρ€ΠΎΠΉ. Π­Ρ‚ΠΎ ΠΎΡ‚Π½ΠΎΡˆΠ΅Π½ΠΈΠ΅ Π°Π±ΡΠΎΠ»ΡŽΡ‚Π½ΠΎΠΉ диэлСктричСской проницаСмости ΠΌΠ°Ρ‚Π΅Ρ€ΠΈΠ°Π»Π°, ∈r, ΠΊ Π°Π±ΡΠΎΠ»ΡŽΡ‚Π½ΠΎΠΉ диэлСктричСской проницаСмости Π²Π°ΠΊΡƒΡƒΠΌΠ°, ∈ 0 , ΠΊΠΎΡ‚ΠΎΡ€ΠΎΠ΅ выраТаСтся ΡΠ»Π΅Π΄ΡƒΡŽΡ‰Π΅ΠΉ Ρ„ΠΎΡ€ΠΌΡƒΠ»ΠΎΠΉ:

    ∈ r = ∈ / ∈ 0

    НиТС ΠΏΡ€ΠΈΠ²Π΅Π΄Π΅Π½Ρ‹ Π½Π΅ΠΊΠΎΡ‚ΠΎΡ€Ρ‹Π΅ ΠΎΠ±Ρ‰ΠΈΠ΅ диэлСктричСскиС ΠΌΠ°Ρ‚Π΅Ρ€ΠΈΠ°Π»Ρ‹ ΠΈ Ρ‚ΠΈΠΏΠΈΡ‡Π½Ρ‹Π΅ диэлСктричСскиС постоянныС для ΠΊΠ°ΠΆΠ΄ΠΎΠ³ΠΎ ΠΈΠ· Π½ΠΈΡ….ЗначСния ΠΌΠΎΠ³ΡƒΡ‚ Π²Π°Ρ€ΡŒΠΈΡ€ΠΎΠ²Π°Ρ‚ΡŒΡΡ, ΠΏΠΎΡΠΊΠΎΠ»ΡŒΠΊΡƒ зависят ΠΎΡ‚ ΠΊΠΎΠ½ΠΊΡ€Π΅Ρ‚Π½ΠΎΠ³ΠΎ состава ΠΌΠ°Ρ‚Π΅Ρ€ΠΈΠ°Π»Π°.

    ΠœΠ°Ρ‚Π΅Ρ€ΠΈΠ°Π» Π‘Ρ‚Π°Π½Π΄Π°Ρ€Ρ‚Π½Ρ‹Π΅ значСния ∈r

    • Π’ΠΎΠ·Π΄ΡƒΡ… 1.0
    • Ρ‚Π΅Ρ„Π»ΠΎΠ½ 2,0
    • Π‘ΡƒΠΌΠ°Π³Π° 2.5
    • Масло 4.0
    • Блюда 5,0
    • Π‘Ρ‚Π΅ΠΊΠ»ΠΎ 7,5
    • ΠšΠ΅Ρ€Π°ΠΌΠΈΠΊΠ° 1200

    ДиэлСктричСская ΠΏΡ€ΠΎΠ½ΠΈΡ†Π°Π΅ΠΌΠΎΡΡ‚ΡŒ ∈r Π±Π΅Π·Ρ€Π°Π·ΠΌΠ΅Ρ€Π½Π°, ΠΏΠΎΡΠΊΠΎΠ»ΡŒΠΊΡƒ являСтся ΠΎΡ‚Π½ΠΎΡΠΈΡ‚Π΅Π»ΡŒΠ½ΠΎΠΉ ΠΌΠ΅Ρ€ΠΎΠΉ.Π­Ρ‚ΠΎ ΠΎΡ‚Π½ΠΎΡˆΠ΅Π½ΠΈΠ΅ Π°Π±ΡΠΎΠ»ΡŽΡ‚Π½ΠΎΠΉ диэлСктричСской проницаСмости ΠΌΠ°Ρ‚Π΅Ρ€ΠΈΠ°Π»Π°, ∈r, ΠΊ Π°Π±ΡΠΎΠ»ΡŽΡ‚Π½ΠΎΠΉ диэлСктричСской проницаСмости Π²Π°ΠΊΡƒΡƒΠΌΠ°, ∈0, ΠΊΠΎΡ‚ΠΎΡ€ΠΎΠ΅ выраТаСтся ΡΠ»Π΅Π΄ΡƒΡŽΡ‰Π΅ΠΉ Ρ„ΠΎΡ€ΠΌΡƒΠ»ΠΎΠΉ:

    ∈r = ∈ / ∈0

    Π—Π½Π°Ρ‡Π΅Π½ΠΈΠ΅ ∈0 составляСт 8,85 Γ— 10-12 Π€ / ΠΌ.

    Π€ΠΎΡ€ΠΌΡƒΠ»Π° Смкости ΠΏΠΎ физичСским ΠΏΠ°Ρ€Π°ΠΌΠ΅Ρ‚Ρ€Π°ΠΌ

    Π’Ρ‹ Π²ΠΈΠ΄Π΅Π»ΠΈ, ΠΊΠ°ΠΊ Π΅ΠΌΠΊΠΎΡΡ‚ΡŒ Π½Π°ΠΏΡ€ΡΠΌΡƒΡŽ связана с ΠΏΠ»ΠΎΡ‰Π°Π΄ΡŒΡŽ пластины, A, ΠΈ диэлСктричСской ΠΏΡ€ΠΎΠ½ΠΈΡ†Π°Π΅ΠΌΠΎΡΡ‚ΡŒΡŽ, ∈r, ΠΈ ΠΎΠ±Ρ€Π°Ρ‚Π½ΠΎ ΠΏΡ€ΠΎΠΏΠΎΡ€Ρ†ΠΈΠΎΠ½Π°Π»ΡŒΠ½Π° Ρ€Π°ΡΡΡ‚ΠΎΡΠ½ΠΈΡŽ ΠΌΠ΅ΠΆΠ΄Ρƒ пластинами, d. Вочная Ρ„ΠΎΡ€ΠΌΡƒΠ»Π° для расчСта Смкости ΠΏΠΎ этим Ρ‚Ρ€Π΅ΠΌ Π²Π΅Π»ΠΈΡ‡ΠΈΠ½Π°ΠΌ:

    C = A ∈ r ∈ / d

    Π³Π΄Π΅ ∈ = ∈ r ∈ 0 = ∈r (8.85 Γ— 10-12Π€ / ΠΌ)

    Π•ΠΌΠΊΠΎΡΡ‚ΡŒ ΠΏΠ°Ρ€Π°Π»Π»Π΅Π»ΡŒΠ½ΠΎΠ³ΠΎ Π²Ρ‹Π²ΠΎΠ΄Π° кондСнсатора

    Рассмотрим кондСнсатор с ΠΏΠ°Ρ€Π°Π»Π»Π΅Π»ΡŒΠ½Ρ‹ΠΌΠΈ пластинами. Π Π°Π·ΠΌΠ΅Ρ€ пластины большой, Π° расстояниС ΠΌΠ΅ΠΆΠ΄Ρƒ пластинами ΠΎΡ‡Π΅Π½ΡŒ малСнькоС, поэтому элСктричСскоС ΠΏΠΎΠ»Π΅ ΠΌΠ΅ΠΆΠ΄Ρƒ пластинами ΠΎΠ΄Π½ΠΎΡ€ΠΎΠ΄Π½ΠΎ.

    ЭлСктричСскоС ΠΏΠΎΠ»Π΅ Β«EΒ» ΠΌΠ΅ΠΆΠ΄Ρƒ кондСнсаторами с ΠΏΠ°Ρ€Π°Π»Π»Π΅Π»ΡŒΠ½Ρ‹ΠΌΠΈ пластинами составляСт:

    Π•ΠΌΠΊΠΎΡΡ‚ΡŒ цилиндричСских кондСнсаторов Ρ„ΠΈΠ·ΠΈΠΊΠ°

    Рассмотрим цилиндричСский кондСнсатор Π΄Π»ΠΈΠ½ΠΎΠΉ L, ΠΎΠ±Ρ€Π°Π·ΠΎΠ²Π°Π½Π½Ρ‹ΠΉ двумя ΠΊΠΎΠ°ΠΊΡΠΈΠ°Π»ΡŒΠ½Ρ‹ΠΌΠΈ Ρ†ΠΈΠ»ΠΈΠ½Π΄Ρ€Π°ΠΌΠΈ радиусами Β«aΒ» ΠΈ Β«bΒ».ΠŸΡ€Π΅Π΄ΠΏΠΎΠ»ΠΎΠΆΠΈΠΌ, Ρ‡Ρ‚ΠΎ L >> b, Ρ‚Π°ΠΊΠΎΠ΅, Ρ‡Ρ‚ΠΎ Π½Π° ΠΊΠΎΠ½Ρ†Π°Ρ… Ρ†ΠΈΠ»ΠΈΠ½Π΄Ρ€ΠΎΠ² Π½Π΅Ρ‚ ΠΎΠΊΠ°ΠΉΠΌΠ»ΡΡŽΡ‰Π΅Π³ΠΎ поля.

    ΠŸΡƒΡΡ‚ΡŒ Β«qΒ» — это заряд кондСнсатора, Π° Β«VΒ» — это Ρ€Π°Π·Π½ΠΎΡΡ‚ΡŒ ΠΏΠΎΡ‚Π΅Π½Ρ†ΠΈΠ°Π»ΠΎΠ² ΠΌΠ΅ΠΆΠ΄Ρƒ пластинами. Π’Π½ΡƒΡ‚Ρ€Π΅Π½Π½ΠΈΠΉ Ρ†ΠΈΠ»ΠΈΠ½Π΄Ρ€ заряТСн ΠΏΠΎΠ»ΠΎΠΆΠΈΡ‚Π΅Π»ΡŒΠ½ΠΎ, Π° внСшний Ρ†ΠΈΠ»ΠΈΠ½Π΄Ρ€ — ΠΎΡ‚Ρ€ΠΈΡ†Π°Ρ‚Π΅Π»ΡŒΠ½ΠΎ. ΠœΡ‹ Ρ…ΠΎΡ‚ΠΈΠΌ ΡƒΠ·Π½Π°Ρ‚ΡŒ Π²Ρ‹Ρ€Π°ΠΆΠ΅Π½ΠΈΠ΅ Смкости для цилиндричСского кондСнсатора. Для этого ΠΌΡ‹ рассматриваСм Ρ†ΠΈΠ»ΠΈΠ½Π΄Ρ€ΠΈΡ‡Π΅ΡΠΊΡƒΡŽ Π³Π°ΡƒΡΡΠΎΠ²ΡΠΊΡƒΡŽ ΠΏΠΎΠ²Π΅Ρ€Ρ…Π½ΠΎΡΡ‚ΡŒ радиуса Β«rΒ», Ρ‚Π°ΠΊΡƒΡŽ ​​что a << b.

    Если Β«EΒ» — Π½Π°ΠΏΡ€ΡΠΆΠ΅Π½Π½ΠΎΡΡ‚ΡŒ элСктричСского поля Π² любой Ρ‚ΠΎΡ‡ΠΊΠ΅ цилиндричСской гауссовой повСрхности, Ρ‚ΠΎ ΠΏΠΎ Π·Π°ΠΊΠΎΠ½Ρƒ Гаусса:

    Если Β«VΒ» — Ρ€Π°Π·Π½ΠΎΡΡ‚ΡŒ ΠΏΠΎΡ‚Π΅Π½Ρ†ΠΈΠ°Π»ΠΎΠ² ΠΌΠ΅ΠΆΠ΄Ρƒ пластинами, Ρ‚ΠΎΠ³Π΄Π°

    Π­Ρ‚ΠΎ ΡΠΎΠΎΡ‚Π½ΠΎΡˆΠ΅Π½ΠΈΠ΅ для Смкости цилиндричСского кондСнсатора.

    Π•ΠΌΠΊΠΎΡΡ‚ΡŒ сфСричСского кондСнсатора

    Π•ΠΌΠΊΠΎΡΡ‚ΡŒ ΠΈΠ·ΠΎΠ»ΠΈΡ€ΠΎΠ²Π°Π½Π½ΠΎΠ³ΠΎ сфСричСского кондСнсатора

    Π’Π½Π΅ΡˆΠ½ΠΈΠΉ источник
    https://en.wikipedia.org/wiki/Capacitance

    Π‘Ρ‚Ρ€Π°Π½ΠΈΡ†Π° Π½Π΅ Π½Π°ΠΉΠ΄Π΅Π½Π° | MIT

    ΠŸΠ΅Ρ€Π΅ΠΉΡ‚ΠΈ ΠΊ ΡΠΎΠ΄Π΅Ρ€ΠΆΠ°Π½ΠΈΡŽ ↓
    • ΠžΠ±Ρ€Π°Π·ΠΎΠ²Π°Π½ΠΈΠ΅
    • Π˜ΡΡΠ»Π΅Π΄ΠΎΠ²Π°Ρ‚Π΅Π»ΡŒΡΠΊΠ°Ρ Ρ€Π°Π±ΠΎΡ‚Π°
    • Π˜Π½Π½ΠΎΠ²Π°Ρ†ΠΈΠΈ
    • ΠŸΡ€ΠΈΠ΅ΠΌ + ΠΏΠΎΠΌΠΎΡ‰ΡŒ
    • БтудСнчСская Тизнь
    • Новости
    • Выпускников
    • О ΠœΠ°ΡΡΠ°Ρ‡ΡƒΡΠ΅Ρ‚ΡΠΊΠΎΠΌ тСхнологичСском институтС
    • ΠŸΠΎΠ΄Ρ€ΠΎΠ±Π½Π΅Π΅ ↓
      • ΠŸΡ€ΠΈΠ΅ΠΌ + ΠΏΠΎΠΌΠΎΡ‰ΡŒ
      • БтудСнчСская Тизнь
      • Новости
      • Выпускников
      • О ΠœΠ°ΡΡΠ°Ρ‡ΡƒΡΠ΅Ρ‚ΡΠΊΠΎΠΌ тСхнологичСском институтС
    МСню ↓ Поиск МСню Ой, ΠΏΠΎΡ…ΠΎΠΆΠ΅, ΠΌΡ‹ Π½Π΅ смогли Π½Π°ΠΉΡ‚ΠΈ Ρ‚ΠΎ, Ρ‡Ρ‚ΠΎ Π²Ρ‹ искали!
    ΠŸΠΎΠΏΡ€ΠΎΠ±ΡƒΠΉΡ‚Π΅ ΠΏΠΎΠΈΡΠΊΠ°Ρ‚ΡŒ Ρ‡Ρ‚ΠΎ-Π½ΠΈΠ±ΡƒΠ΄ΡŒ Π΅Ρ‰Π΅! Π§Ρ‚ΠΎ Π²Ρ‹ ΠΈΡ‰Π΅Ρ‚Π΅? Π£Π²ΠΈΠ΄Π΅Ρ‚ΡŒ большС Ρ€Π΅Π·ΡƒΠ»ΡŒΡ‚Π°Ρ‚ΠΎΠ²

    ΠŸΡ€Π΅Π΄Π»ΠΎΠΆΠ΅Π½ΠΈΡ ΠΈΠ»ΠΈ ΠΎΡ‚Π·Ρ‹Π²Ρ‹?

    Ρ„Π°ΠΊΡ‚ΠΎΡ€ΠΎΠ², Π²Π»ΠΈΡΡŽΡ‰ΠΈΡ… Π½Π° Π΅ΠΌΠΊΠΎΡΡ‚ΡŒ | ΠšΠΎΠ½Π΄Π΅Π½ΡΠ°Ρ‚ΠΎΡ€Ρ‹

    БущСствуСт Ρ‚Ρ€ΠΈ основных Ρ„Π°ΠΊΡ‚ΠΎΡ€Π° конструкции кондСнсатора, ΠΎΠΏΡ€Π΅Π΄Π΅Π»ΡΡŽΡ‰ΠΈΡ… Π²Π΅Π»ΠΈΡ‡ΠΈΠ½Ρƒ создаваСмой Смкости.ВсС эти Ρ„Π°ΠΊΡ‚ΠΎΡ€Ρ‹ ΠΎΠΏΡ€Π΅Π΄Π΅Π»ΡΡŽΡ‚ Π΅ΠΌΠΊΠΎΡΡ‚ΡŒ, влияя Π½Π° Ρ‚ΠΎ, ΠΊΠ°ΠΊΠΎΠΉ ΠΏΠΎΡ‚ΠΎΠΊ элСктричСского поля (ΠΎΡ‚Π½ΠΎΡΠΈΡ‚Π΅Π»ΡŒΠ½Π°Ρ Ρ€Π°Π·Π½ΠΈΡ†Π° элСктронов ΠΌΠ΅ΠΆΠ΄Ρƒ пластинами) Π±ΡƒΠ΄Π΅Ρ‚ Ρ€Π°Π·Π²ΠΈΠ²Π°Ρ‚ΡŒΡΡ для Π΄Π°Π½Π½ΠΎΠΉ Π²Π΅Π»ΠΈΡ‡ΠΈΠ½Ρ‹ силы элСктричСского поля (напряТСния ΠΌΠ΅ΠΆΠ΄Ρƒ двумя пластинами):

    ΠŸΠ›ΠžΠ©ΠΠ”Π¬ ΠŸΠ›ΠΠ‘Π’Π˜ΠΠ« : ΠŸΡ€ΠΈ ΠΏΡ€ΠΎΡ‡ΠΈΡ… Ρ€Π°Π²Π½Ρ‹Ρ… условиях большая ΠΏΠ»ΠΎΡ‰Π°Π΄ΡŒ пластины Π΄Π°Π΅Ρ‚ Π±ΠΎΠ»ΡŒΡˆΡƒΡŽ Π΅ΠΌΠΊΠΎΡΡ‚ΡŒ; мСньшая ΠΏΠ»ΠΎΡ‰Π°Π΄ΡŒ пластины Π΄Π°Π΅Ρ‚ ΠΌΠ΅Π½ΡŒΡˆΡƒΡŽ Π΅ΠΌΠΊΠΎΡΡ‚ΡŒ.

    ПояснСниС: Π§Π΅ΠΌ большС ΠΏΠ»ΠΎΡ‰Π°Π΄ΡŒ пластины, Ρ‚Π΅ΠΌ большС ΠΌΠ°Π³Π½ΠΈΡ‚Π½Ρ‹ΠΉ ΠΏΠΎΡ‚ΠΎΠΊ (заряд, собранный Π½Π° пластинах) для Π΄Π°Π½Π½ΠΎΠΉ силы поля (напряТСниС Π½Π° пластинах).

    Π ΠΠ‘Π‘Π’ΠžΠ―ΠΠ˜Π• ΠŸΠ›ΠΠ‘Π’Π˜Π : ΠŸΡ€ΠΈ ΠΏΡ€ΠΎΡ‡ΠΈΡ… Ρ€Π°Π²Π½Ρ‹Ρ… условиях большСС расстояниС ΠΌΠ΅ΠΆΠ΄Ρƒ пластинами Π΄Π°Π΅Ρ‚ ΠΌΠ΅Π½ΡŒΡˆΡƒΡŽ Π΅ΠΌΠΊΠΎΡΡ‚ΡŒ; мСньшСС расстояниС ΠΌΠ΅ΠΆΠ΄Ρƒ пластинами Π΄Π°Π΅Ρ‚ Π±ΠΎΠ»ΡŒΡˆΡƒΡŽ Π΅ΠΌΠΊΠΎΡΡ‚ΡŒ.

    ПояснСниС: Π‘ΠΎΠ»Π΅Π΅ Π±Π»ΠΈΠ·ΠΊΠΎΠ΅ расстояниС ΠΏΡ€ΠΈΠ²ΠΎΠ΄ΠΈΡ‚ ΠΊ большСй силС поля (напряТСниС Π½Π° кондСнсаторС, Π΄Π΅Π»Π΅Π½Π½ΠΎΠ΅ Π½Π° расстояниС ΠΌΠ΅ΠΆΠ΄Ρƒ пластинами), Ρ‡Ρ‚ΠΎ ΠΏΡ€ΠΈΠ²ΠΎΠ΄ΠΈΡ‚ ΠΊ Π±ΠΎΠ»ΡŒΡˆΠ΅ΠΌΡƒ ΠΏΠΎΡ‚ΠΎΠΊΡƒ поля (заряд, Π½Π°ΠΊΠΎΠΏΠ»Π΅Π½Π½Ρ‹ΠΉ Π½Π° пластинах) для любого Π·Π°Π΄Π°Π½Π½ΠΎΠ³ΠΎ напряТСния, ΠΏΡ€ΠΈΠ»ΠΎΠΆΠ΅Π½Π½ΠΎΠ³ΠΎ ΠΊ пластинам.

    Π”Π˜Π­Π›Π•ΠšΠ’Π Π˜Π§Π•Π‘ΠšΠ˜Π™ ΠœΠΠ’Π•Π Π˜ΠΠ› : ΠŸΡ€ΠΈ ΠΏΡ€ΠΎΡ‡ΠΈΡ… Ρ€Π°Π²Π½Ρ‹Ρ… условиях большая диэлСктричСская ΠΏΡ€ΠΎΠ½ΠΈΡ†Π°Π΅ΠΌΠΎΡΡ‚ΡŒ диэлСктрика Π΄Π°Π΅Ρ‚ Π±ΠΎΠ»ΡŒΡˆΡƒΡŽ Π΅ΠΌΠΊΠΎΡΡ‚ΡŒ; мСньшая диэлСктричСская ΠΏΡ€ΠΎΠ½ΠΈΡ†Π°Π΅ΠΌΠΎΡΡ‚ΡŒ диэлСктрика Π΄Π°Π΅Ρ‚ ΠΌΠ΅Π½ΡŒΡˆΡƒΡŽ Π΅ΠΌΠΊΠΎΡΡ‚ΡŒ.

    ПояснСниС: Π₯отя это слоТно ΠΎΠ±ΡŠΡΡΠ½ΠΈΡ‚ΡŒ, Π½Π΅ΠΊΠΎΡ‚ΠΎΡ€Ρ‹Π΅ ΠΌΠ°Ρ‚Π΅Ρ€ΠΈΠ°Π»Ρ‹ ΠΏΡ€Π΅Π΄Π»Π°Π³Π°ΡŽΡ‚ мСньшСС сопротивлСниС ΠΏΠΎΡ‚ΠΎΠΊΡƒ поля для Π΄Π°Π½Π½ΠΎΠΉ Π²Π΅Π»ΠΈΡ‡ΠΈΠ½Ρ‹ силы поля. ΠœΠ°Ρ‚Π΅Ρ€ΠΈΠ°Π»Ρ‹ с большСй диэлСктричСской ΠΏΡ€ΠΎΠ½ΠΈΡ†Π°Π΅ΠΌΠΎΡΡ‚ΡŒΡŽ Π΄ΠΎΠΏΡƒΡΠΊΠ°ΡŽΡ‚ больший ΠΏΠΎΡ‚ΠΎΠΊ поля (ΠΏΡ€Π΅Π΄Π»Π°Π³Π°ΡŽΡ‚ мСньшСС сопротивлСниС) ΠΈ, ΡΠ»Π΅Π΄ΠΎΠ²Π°Ρ‚Π΅Π»ΡŒΠ½ΠΎ, больший Π½Π°ΠΊΠΎΠΏΠ»Π΅Π½Π½Ρ‹ΠΉ заряд для любой Π·Π°Π΄Π°Π½Π½ΠΎΠΉ Π²Π΅Π»ΠΈΡ‡ΠΈΠ½Ρ‹ силы поля (ΠΏΡ€ΠΈΠ»ΠΎΠΆΠ΅Π½Π½ΠΎΠ³ΠΎ напряТСния).

    Β«ΠžΡ‚Π½ΠΎΡΠΈΡ‚Π΅Π»ΡŒΠ½Π°ΡΒ» диэлСктричСская ΠΏΡ€ΠΎΠ½ΠΈΡ†Π°Π΅ΠΌΠΎΡΡ‚ΡŒ ΠΎΠ·Π½Π°Ρ‡Π°Π΅Ρ‚ Π΄ΠΈΡΠ»Π΅ΠΊΡ‚Ρ€ΠΈΡ‡Π΅ΡΠΊΡƒΡŽ ΠΏΡ€ΠΎΠ½ΠΈΡ†Π°Π΅ΠΌΠΎΡΡ‚ΡŒ ΠΌΠ°Ρ‚Π΅Ρ€ΠΈΠ°Π»Π° ΠΎΡ‚Π½ΠΎΡΠΈΡ‚Π΅Π»ΡŒΠ½ΠΎ диэлСктричСской проницаСмости чистого Π²Π°ΠΊΡƒΡƒΠΌΠ°. Π§Π΅ΠΌ большС число, Ρ‚Π΅ΠΌ большС диэлСктричСская ΠΏΡ€ΠΎΠ½ΠΈΡ†Π°Π΅ΠΌΠΎΡΡ‚ΡŒ ΠΌΠ°Ρ‚Π΅Ρ€ΠΈΠ°Π»Π°.Π‘Ρ‚Π΅ΠΊΠ»ΠΎ, Π½Π°ΠΏΡ€ΠΈΠΌΠ΅Ρ€, с ΠΎΡ‚Π½ΠΎΡΠΈΡ‚Π΅Π»ΡŒΠ½ΠΎΠΉ диэлСктричСской ΠΏΡ€ΠΎΠ½ΠΈΡ†Π°Π΅ΠΌΠΎΡΡ‚ΡŒΡŽ 7, ΠΈΠΌΠ΅Π΅Ρ‚ Π² сСмь Ρ€Π°Π· Π±ΠΎΠ»ΡŒΡˆΡƒΡŽ Π΄ΠΈΡΠ»Π΅ΠΊΡ‚Ρ€ΠΈΡ‡Π΅ΡΠΊΡƒΡŽ ΠΏΡ€ΠΎΠ½ΠΈΡ†Π°Π΅ΠΌΠΎΡΡ‚ΡŒ чистого Π²Π°ΠΊΡƒΡƒΠΌΠ° ΠΈ, ΡΠ»Π΅Π΄ΠΎΠ²Π°Ρ‚Π΅Π»ΡŒΠ½ΠΎ, позволяСт ΡΠΎΠ·Π΄Π°Ρ‚ΡŒ ΠΏΠΎΡ‚ΠΎΠΊ элСктричСского поля, Π² сСмь Ρ€Π°Π· Π±ΠΎΠ»Π΅Π΅ ΡΠΈΠ»ΡŒΠ½Ρ‹ΠΉ, Ρ‡Π΅ΠΌ Ρƒ Π²Π°ΠΊΡƒΡƒΠΌΠ°, ΠΏΡ€ΠΈ ΠΏΡ€ΠΎΡ‡ΠΈΡ… Ρ€Π°Π²Π½Ρ‹Ρ… условиях. Π’ ΡΠ»Π΅Π΄ΡƒΡŽΡ‰Π΅ΠΉ Ρ‚Π°Π±Π»ΠΈΡ†Π΅ пСрСчислСны ΠΎΡ‚Π½ΠΎΡΠΈΡ‚Π΅Π»ΡŒΠ½Ρ‹Π΅ диэлСктричСскиС проницаСмости (Ρ‚Π°ΠΊΠΆΠ΅ извСстныС ΠΊΠ°ΠΊ «диэлСктричСская ΠΏΡ€ΠΎΠ½ΠΈΡ†Π°Π΅ΠΌΠΎΡΡ‚ΡŒΒ») Ρ€Π°Π·Π»ΠΈΡ‡Π½Ρ‹Ρ… распространСнных вСщСств:

    ΠœΠ°Ρ‚Π΅Ρ€ΠΈΠ°Π»

    ΠžΡ‚Π½ΠΎΡΠΈΡ‚Π΅Π»ΡŒΠ½Π°Ρ диэлСктричСская ΠΏΡ€ΠΎΠ½ΠΈΡ†Π°Π΅ΠΌΠΎΡΡ‚ΡŒ (диэлСктричСская ΠΏΡ€ΠΎΠ½ΠΈΡ†Π°Π΅ΠΌΠΎΡΡ‚ΡŒ)
    Π’Π°ΠΊΡƒΡƒΠΌ 1.0000
    Π’ΠΎΠ·Π΄ΡƒΡ… 1.0006
    PTFE, FEP (Β«Ρ‚Π΅Ρ„Π»ΠΎΠ½Β») 2,0
    ΠŸΠΎΠ»ΠΈΠΏΡ€ΠΎΠΏΠΈΠ»Π΅Π½ ΠΎΡ‚ 2,20 Π΄ΠΎ 2,28
    Π‘ΠΌΠΎΠ»Π° АББ ΠΎΡ‚ 2,4 Π΄ΠΎ 3,2
    ΠŸΠΎΠ»ΠΈΡΡ‚ΠΈΡ€ΠΎΠ» ΠΎΡ‚ 2,45 Π΄ΠΎ 4,0
    ВощСная Π±ΡƒΠΌΠ°Π³Π° 2,5
    Масло трансформаторноС ΠΎΡ‚ 2,5 Π΄ΠΎ 4
    ВвСрдая Ρ€Π΅Π·ΠΈΠ½Π° 2.ΠžΡ‚ 5 Π΄ΠΎ 4,80
    Π”Π΅Ρ€Π΅Π²ΠΎ (Π”ΡƒΠ±) 3,3
    Π‘ΠΈΠ»ΠΈΠΊΠΎΠ½Ρ‹ ΠΎΡ‚ 3,4 Π΄ΠΎ 4,3
    Π‘Π°ΠΊΠ΅Π»ΠΈΡ‚ ΠΎΡ‚ 3,5 Π΄ΠΎ 6,0
    ΠšΠ²Π°Ρ€Ρ† ΠΏΠ»Π°Π²Π»Π΅Π½Ρ‹ΠΉ 3,8
    Π”Π΅Ρ€Π΅Π²ΠΎ (ΠΊΠ»Π΅Π½) 4,4
    Π‘Ρ‚Π΅ΠΊΠ»ΠΎ ΠΎΡ‚ 4,9 Π΄ΠΎ 7,5
    ΠšΠ°ΡΡ‚ΠΎΡ€ΠΎΠ²ΠΎΠ΅ масло 5,0
    Π”Π΅Ρ€Π΅Π²ΠΎ (Π±Π΅Ρ€Π΅Π·Π°) 5,2
    Блюда, мусковит 5.ΠžΡ‚ 0 Π΄ΠΎ 8,7
    Блюда Π½Π° стСкловолокнС ΠΎΡ‚ 6,3 Π΄ΠΎ 9,3
    Π€Π°Ρ€Ρ„ΠΎΡ€, стСатит 6,5
    Π“Π»ΠΈΠ½ΠΎΠ·Π΅ΠΌ ΠΎΡ‚ 8,0 Π΄ΠΎ 10,0
    Π’ΠΎΠ΄Π° дистиллированная 80,0
    Π‘Π°Ρ€ΠΈΠΉ-стронций-Ρ‚ΠΈΡ‚Π°Π½ΠΈΡ‚ 7500

    ΠŸΡ€ΠΈΠ±Π»ΠΈΠ·ΠΈΡ‚Π΅Π»ΡŒΠ½ΡƒΡŽ Π΅ΠΌΠΊΠΎΡΡ‚ΡŒ для любой ΠΏΠ°Ρ€Ρ‹ Ρ€Π°Π·Π΄Π΅Π»Π΅Π½Π½Ρ‹Ρ… ΠΏΡ€ΠΎΠ²ΠΎΠ΄ΠΎΠ² ΠΌΠΎΠΆΠ½ΠΎ Π½Π°ΠΉΡ‚ΠΈ ΠΏΠΎ ΡΠ»Π΅Π΄ΡƒΡŽΡ‰Π΅ΠΉ Ρ„ΠΎΡ€ΠΌΡƒΠ»Π΅:

    ΠšΠΎΠ½Π΄Π΅Π½ΡΠ°Ρ‚ΠΎΡ€ ΠΌΠΎΠΆΠ½ΠΎ ΡΠ΄Π΅Π»Π°Ρ‚ΡŒ ΠΏΠ΅Ρ€Π΅ΠΌΠ΅Π½Π½Ρ‹ΠΌ, Π° Π½Π΅ фиксированным, ΠΏΡƒΡ‚Π΅ΠΌ измСнСния любого ΠΈΠ· физичСских Ρ„Π°ΠΊΡ‚ΠΎΡ€ΠΎΠ², ΠΎΠΏΡ€Π΅Π΄Π΅Π»ΡΡŽΡ‰ΠΈΡ… Π΅ΠΌΠΊΠΎΡΡ‚ΡŒ.Один ΠΎΡ‚Π½ΠΎΡΠΈΡ‚Π΅Π»ΡŒΠ½ΠΎ простой Ρ„Π°ΠΊΡ‚ΠΎΡ€, ΠΊΠΎΡ‚ΠΎΡ€Ρ‹ΠΉ ΠΌΠΎΠΆΠ½ΠΎ ΠΈΠ·ΠΌΠ΅Π½ΠΈΡ‚ΡŒ Π² конструкции кондСнсатора, — это ΠΏΠ»ΠΎΡ‰Π°Π΄ΡŒ пластины ΠΈΠ»ΠΈ, Ρ‚ΠΎΡ‡Π½Π΅Π΅, Π²Π΅Π»ΠΈΡ‡ΠΈΠ½Π° пСрСкрытия пластин.

    На ΡΠ»Π΅Π΄ΡƒΡŽΡ‰Π΅ΠΉ Ρ„ΠΎΡ‚ΠΎΠ³Ρ€Π°Ρ„ΠΈΠΈ ΠΏΠΎΠΊΠ°Π·Π°Π½ ΠΏΡ€ΠΈΠΌΠ΅Ρ€ ΠΏΠ΅Ρ€Π΅ΠΌΠ΅Π½Π½ΠΎΠ³ΠΎ кондСнсатора, ΠΈΡΠΏΠΎΠ»ΡŒΠ·ΡƒΡŽΡ‰Π΅Π³ΠΎ Π½Π°Π±ΠΎΡ€ Ρ‡Π΅Ρ€Π΅Π΄ΡƒΡŽΡ‰ΠΈΡ…ΡΡ мСталличСских пластин ΠΈ Π²ΠΎΠ·Π΄ΡƒΡˆΠ½Ρ‹ΠΉ Π·Π°Π·ΠΎΡ€ Π² качСствС диэлСктричСского ΠΌΠ°Ρ‚Π΅Ρ€ΠΈΠ°Π»Π°:

    ΠŸΡ€ΠΈ Π²Ρ€Π°Ρ‰Π΅Π½ΠΈΠΈ Π²Π°Π»Π° ΡΡ‚Π΅ΠΏΠ΅Π½ΡŒ пСрСкрытия Π½Π°Π±ΠΎΡ€ΠΎΠ² пластин Π±ΡƒΠ΄Π΅Ρ‚ ΠΈΠ·ΠΌΠ΅Π½ΡΡ‚ΡŒΡΡ, измСняя ΡΡ„Ρ„Π΅ΠΊΡ‚ΠΈΠ²Π½ΡƒΡŽ ΠΏΠ»ΠΎΡ‰Π°Π΄ΡŒ пластин, ΠΌΠ΅ΠΆΠ΄Ρƒ ΠΊΠΎΡ‚ΠΎΡ€Ρ‹ΠΌΠΈ ΠΌΠΎΠΆΠ΅Ρ‚ Π±Ρ‹Ρ‚ΡŒ установлСно ΠΊΠΎΠ½Ρ†Π΅Π½Ρ‚Ρ€ΠΈΡ€ΠΎΠ²Π°Π½Π½ΠΎΠ΅ элСктричСскоС ΠΏΠΎΠ»Π΅.Π­Ρ‚ΠΎΡ‚ ΠΊΠΎΠ½ΠΊΡ€Π΅Ρ‚Π½Ρ‹ΠΉ кондСнсатор ΠΈΠΌΠ΅Π΅Ρ‚ Π΅ΠΌΠΊΠΎΡΡ‚ΡŒ Π² ΠΏΠΈΠΊΠΎΡ„Π°Ρ€Π°Π΄Π½ΠΎΠΌ Π΄ΠΈΠ°ΠΏΠ°Π·ΠΎΠ½Π΅ ΠΈ Π½Π°Ρ…ΠΎΠ΄ΠΈΡ‚ ΠΏΡ€ΠΈΠΌΠ΅Π½Π΅Π½ΠΈΠ΅ Π² радиосхСмах.

    БВЯЗАННЫЕ Π ΠΠ‘ΠžΠ§Π˜Π• Π›Π˜Π‘Π’Π«:

    кондСнсаторов ΠΏΠΎΡΠ»Π΅Π΄ΠΎΠ²Π°Ρ‚Π΅Π»ΡŒΠ½ΠΎ ΠΈ ΠΏΠ°Ρ€Π°Π»Π»Π΅Π»ΡŒΠ½ΠΎ

    кондСнсаторов ΠΏΠΎΡΠ»Π΅Π΄ΠΎΠ²Π°Ρ‚Π΅Π»ΡŒΠ½ΠΎ ΠΈ ΠΏΠ°Ρ€Π°Π»Π»Π΅Π»ΡŒΠ½ΠΎ
    Π”Π°Π»Π΅Π΅: ЭнСргия Π² кондСнсаторах Up: Π•ΠΌΠΊΠΎΡΡ‚ΡŒ ΠŸΡ€Π΅Π΄Ρ‹Π΄ΡƒΡ‰Π°Ρ: ДиэлСктрики ΠšΠΎΠ½Π΄Π΅Π½ΡΠ°Ρ‚ΠΎΡ€Ρ‹ — ΠΎΠ΄ΠΈΠ½ ΠΈΠ· стандартных ΠΊΠΎΠΌΠΏΠΎΠ½Π΅Π½Ρ‚ΠΎΠ² элСктронных схСм. ΠšΡ€ΠΎΠΌΠ΅ Ρ‚ΠΎΠ³ΠΎ, часто Π²ΡΡ‚Ρ€Π΅Ρ‡Π°ΡŽΡ‚ΡΡ слоТныС ΠΊΠΎΠΌΠ±ΠΈΠ½Π°Ρ†ΠΈΠΈ кондСнсаторов. Π² практичСских схСмах.Π­Ρ‚ΠΎ, поэтому ΠΏΠΎΠ»Π΅Π·Π½ΠΎ ΠΈΠΌΠ΅Ρ‚ΡŒ Π½Π°Π±ΠΎΡ€ ΠΏΡ€Π°Π²ΠΈΠ» для опрСдСлСния эквивалСнтной Смкости Π½Π΅ΠΊΠΎΡ‚ΠΎΡ€ΠΎΠ³ΠΎ ΠΎΠ±Ρ‰Π΅Π³ΠΎ располоТСния кондСнсаторов. ΠžΠΊΠ°Π·Ρ‹Π²Π°Π΅Ρ‚ΡΡ, всСгда ΠΌΠΎΠΆΠ½ΠΎ Π½Π°ΠΉΡ‚ΠΈ эквивалСнтная Π΅ΠΌΠΊΠΎΡΡ‚ΡŒ ΠΏΡ€ΠΈ ΠΏΠΎΠ²Ρ‚ΠΎΡ€Π½ΠΎΠΌ ΠΏΡ€ΠΈΠΌΠ΅Π½Π΅Π½ΠΈΠ΅ Π΄Π²ΡƒΡ… простых ΠΏΡ€Π°Π²ΠΈΠ» . Π­Ρ‚ΠΈ ΠΏΡ€Π°Π²ΠΈΠ»Π° относятся ΠΊ ΠΏΠΎΠ΄ΠΊΠ»ΡŽΡ‡Π΅Π½Π½Ρ‹ΠΌ кондСнсаторам. ΠΏΠΎΡΠ»Π΅Π΄ΠΎΠ²Π°Ρ‚Π΅Π»ΡŒΠ½ΠΎ ΠΈ ΠΏΠ°Ρ€Π°Π»Π»Π΅Π»ΡŒΠ½ΠΎ.
    Рисунок 15: Π”Π²Π° кондСнсатора ΠΏΠΎΠ΄ΠΊΠ»ΡŽΡ‡Π΅Π½Ρ‹ ΠΏΠ°Ρ€Π°Π»Π»Π΅Π»ΡŒΠ½ΠΎ.
    Рассмотрим Π΄Π²Π° кондСнсатора, соСдинСнных ΠΏΠ°Ρ€Π°Π»Π»Π΅Π»ΡŒΠ½ΠΎ : i.Π΅. , с ΠΏΠΎΠ»ΠΎΠΆΠΈΡ‚Π΅Π»ΡŒΠ½ΠΎ заряТСнныС пластины ΠΏΠΎΠ΄ΠΊΠ»ΡŽΡ‡Π΅Π½Ρ‹ ΠΊ ΠΎΠ±Ρ‰Π΅ΠΌΡƒ « Π²Ρ…ΠΎΠ΄Π½ΠΎΠΌΡƒ » ΠΏΡ€ΠΎΠ²ΠΎΠ΄Ρƒ, Π° ΠΎΡ‚Ρ€ΠΈΡ†Π°Ρ‚Π΅Π»ΡŒΠ½ΠΎ заряТСнныС пластины присоСдинСны ΠΊ ΠΎΠ±Ρ‰Π΅ΠΌΡƒ « Π²Ρ‹Ρ…ΠΎΠ΄Π½ΠΎΠΌΡƒ » ΠΏΡ€ΠΎΠ²ΠΎΠ΄Ρƒ — см. рис. 15. Какая эквивалСнтная Π΅ΠΌΠΊΠΎΡΡ‚ΡŒ ΠΌΠ΅ΠΆΠ΄Ρƒ Π²Ρ…ΠΎΠ΄Π½Ρ‹ΠΌ ΠΈ Π²Ρ‹Ρ…ΠΎΠ΄Π½Ρ‹ΠΌ ΠΏΡ€ΠΎΠ²ΠΎΠ΄Π°ΠΌΠΈ? Π’ этом случаС ΠΏΠΎΡ‚Π΅Π½Ρ†ΠΈΠ°Π» Ρ€Π°Π·Π½ΠΈΡ†Π° ΠΌΠ΅ΠΆΠ΄Ρƒ двумя кондСнсаторами ΠΎΠ΄ΠΈΠ½Π°ΠΊΠΎΠ²Π° ΠΈ Ρ€Π°Π²Π½Π° Ρ€Π°Π·Π½ΠΎΡΡ‚ΡŒ ΠΏΠΎΡ‚Π΅Π½Ρ†ΠΈΠ°Π»ΠΎΠ² ΠΌΠ΅ΠΆΠ΄Ρƒ Π²Ρ…ΠΎΠ΄Π½Ρ‹ΠΌ ΠΈ Π²Ρ‹Ρ…ΠΎΠ΄Π½Ρ‹ΠΌ ΠΏΡ€ΠΎΠ²ΠΎΠ΄Π°ΠΌΠΈ. ΠžΠ±Ρ‰ΠΈΠΉ заряд ΠΎΠ΄Π½Π°ΠΊΠΎ, хранящиСся Π² Π΄Π²ΡƒΡ… кондСнсаторах дСлятся ΠΌΠ΅ΠΆΠ΄Ρƒ кондСнсаторы, Ρ‚Π°ΠΊ ΠΊΠ°ΠΊ ΠΎΠ½ Π΄ΠΎΠ»ΠΆΠ΅Π½ Ρ€Π°ΡΠΏΡ€Π΅Π΄Π΅Π»ΡΡ‚ΡŒΡΡ Ρ‚Π°ΠΊ, Ρ‡Ρ‚ΠΎΠ±Ρ‹ напряТСниС Π½Π° Π΄Π²Π° Ρ‚ΠΎ ΠΆΠ΅ самоС.ΠŸΠΎΡΠΊΠΎΠ»ΡŒΠΊΡƒ кондСнсаторы ΠΌΠΎΠ³ΡƒΡ‚ ΠΈΠΌΠ΅Ρ‚ΡŒ Ρ€Π°Π·Π½ΡƒΡŽ Π΅ΠΌΠΊΠΎΡΡ‚ΡŒ, ΠΈ, ΠΈ сборы Ρ‚ΠΎΠΆΠ΅ ΠΌΠΎΠ³ΡƒΡ‚ Π±Ρ‹Ρ‚ΡŒ Ρ€Π°Π·Π½Ρ‹ΠΌΠΈ. ЭквивалСнтная Π΅ΠΌΠΊΠΎΡΡ‚ΡŒ ΠΏΠ°Ρ€Ρ‹ кондСнсаторов — это просто ΡΠΎΠΎΡ‚Π½ΠΎΡˆΠ΅Π½ΠΈΠ΅, Π³Π΄Π΅ — ΠΎΠ±Ρ‰ΠΈΠΉ Π½Π°ΠΊΠΎΠΏΠ»Π΅Π½Π½Ρ‹ΠΉ заряд. Π­Ρ‚ΠΎ слСдуСт ΠΈΠ· Ρ‚ΠΎΠ³ΠΎ
    (113)

    давая
    (114)

    Π—Π΄Π΅ΡΡŒ ΠΌΡ‹ воспользовались Ρ‚Π΅ΠΌ Ρ„Π°ΠΊΡ‚ΠΎΠΌ, Ρ‡Ρ‚ΠΎ напряТСниС являСтся ΠΎΠ±Ρ‰ΠΈΠΌ для всСх Ρ‚Ρ€Π΅Ρ… кондСнсаторы.Π’Π°ΠΊΠΈΠΌ ΠΎΠ±Ρ€Π°Π·ΠΎΠΌ, ΠΏΡ€Π°Π²ΠΈΠ»ΠΎ Ρ‚Π°ΠΊΠΎΠ²ΠΎ:
    ЭквивалСнтная Π΅ΠΌΠΊΠΎΡΡ‚ΡŒ Π΄Π²ΡƒΡ… кондСнсаторов, соСдинСнных ΠΏΠ°Ρ€Π°Π»Π»Π΅Π»ΡŒΠ½ΠΎ прСдставляСт собой сумму ΠΎΡ‚Π΄Π΅Π»ΡŒΠ½Ρ‹Ρ… СмкостСй.
    Для кондСнсаторов, соСдинСнных ΠΏΠ°Ρ€Π°Π»Π»Π΅Π»ΡŒΠ½ΠΎ, ΡƒΡ€Π°Π²Π½Π΅Π½ΠΈΠ΅. (114) ΠΎΠ±ΠΎΠ±Ρ‰Π°Π΅Ρ‚ Π½Π° .
    Рисунок 16: Π”Π²Π° кондСнсатора, соСдинСнных ΠΏΠΎΡΠ»Π΅Π΄ΠΎΠ²Π°Ρ‚Π΅Π»ΡŒΠ½ΠΎ.
    Рассмотрим Π΄Π²Π° кондСнсатора, ΠΏΠΎΠ΄ΠΊΠ»ΡŽΡ‡Π΅Π½Π½Ρ‹Ρ… Π² ΡΠ΅Ρ€ΠΈΡŽ : , Ρ‚. Π•. , Π² линию Ρ‚Π°ΠΊ, Ρ‡Ρ‚ΠΎ ΠΏΠΎΠ»ΠΎΠΆΠΈΡ‚Π΅Π»ΡŒΠ½Π°Ρ пластина ΠΎΠ΄Π½ΠΎΠ³ΠΎ ΠΏΡ€ΠΈΠΊΡ€Π΅ΠΏΠ»Π΅Π½Π° ΠΊ ΠΎΡ‚Ρ€ΠΈΡ†Π°Ρ‚Π΅Π»ΡŒΠ½ΠΎΠΉ пластинС Π΄Ρ€ΡƒΠ³ΠΎΠ³ΠΎ — см. Π˜Π½ΠΆΠΈΡ€.16. ЀактичСски, ΠΏΡ€Π΅Π΄ΠΏΠΎΠ»ΠΎΠΆΠΈΠΌ, Ρ‡Ρ‚ΠΎ ΠΏΠΎΠ»ΠΎΠΆΠΈΡ‚Π΅Π»ΡŒΠ½Π°Ρ ΠΎΠ±ΠΊΠ»Π°Π΄ΠΊΠ° кондСнсатора 1 ΠΏΠΎΠ΄ΠΊΠ»ΡŽΡ‡Π΅Π½Π° ΠΊ ΠΏΡ€ΠΎΠ²ΠΎΠ΄Ρƒ « Π²Ρ…ΠΎΠ΄ » ΠΎΡ‚Ρ€ΠΈΡ†Π°Ρ‚Π΅Π»ΡŒΠ½Π°Ρ ΠΎΠ±ΠΊΠ»Π°Π΄ΠΊΠ° кондСнсатора 1 ΠΏΠΎΠ΄ΠΊΠ»ΡŽΡ‡Π°Π΅Ρ‚ΡΡ ΠΊ ΠΏΠΎΠ»ΠΎΠΆΠΈΡ‚Π΅Π»ΡŒΠ½Π°Ρ пластина кондСнсатора 2 ΠΈ ΠΎΡ‚Ρ€ΠΈΡ†Π°Ρ‚Π΅Π»ΡŒΠ½Π°Ρ пластина кондСнсатора 2 ΠΏΠΎΠ΄ΠΊΠ»ΡŽΡ‡Π°Π΅Ρ‚ΡΡ ΠΊ ΠΏΡ€ΠΎΠ²ΠΎΠ΄Ρƒ « Π²Ρ‹Ρ…ΠΎΠ΄ ». Какая эквивалСнтная Π΅ΠΌΠΊΠΎΡΡ‚ΡŒ ΠΌΠ΅ΠΆΠ΄Ρƒ Π²Ρ…ΠΎΠ΄Π½Ρ‹ΠΌΠΈ ΠΈ Π²Ρ‹Ρ…ΠΎΠ΄Π½Ρ‹ΠΌΠΈ ΠΏΡ€ΠΎΠ²ΠΎΠ΄Π°ΠΌΠΈ? Π’ этом случаС Π²Π°ΠΆΠ½ΠΎ ΠΏΠΎΠ½ΠΈΠΌΠ°Ρ‚ΡŒ, Ρ‡Ρ‚ΠΎ заряд, хранящийся Π² Π΄Π²Π° кондСнсатора ΠΎΠ΄ΠΈΠ½Π°ΠΊΠΎΠ²Ρ‹Π΅. Π­Ρ‚ΠΎ Π»Π΅Π³Ρ‡Π΅ всСго ΡƒΠ²ΠΈΠ΄Π΅Ρ‚ΡŒ, Ссли Ρ€Π°ΡΡΠΌΠΎΡ‚Ρ€Π΅Ρ‚ΡŒ « Π²Π½ΡƒΡ‚Ρ€Π΅Π½Π½ΠΈΠ΅ » пластины: i.Π΅. , ΠΎΡ‚Ρ€ΠΈΡ†Π°Ρ‚Π΅Π»ΡŒΠ½Π°Ρ пластина кондСнсатора 1, ΠΈ ΠΏΠΎΠ»ΠΎΠΆΠΈΡ‚Π΅Π»ΡŒΠ½Π°Ρ пластина кондСнсатора 2. Π­Ρ‚ΠΈ пластины физичСски ΠΎΡ‚ΠΊΠ»ΡŽΡ‡Π΅Π½Ρ‹. ΠΎΡ‚ ΠΎΡΡ‚Π°Π»ΡŒΠ½ΠΎΠΉ части схСмы, поэтому ΠΎΠ±Ρ‰ΠΈΠΉ заряд Π½Π° Π½ΠΈΡ… Π΄ΠΎΠ»ΠΆΠ΅Π½ ΠžΡΡ‚Π°ΡŽΡ‚ΡΡ Π½Π΅ΠΈΠ·ΠΌΠ΅Π½Π½Ρ‹ΠΌΠΈ. Если ΠΏΡ€Π΅Π΄ΠΏΠΎΠ»ΠΎΠΆΠΈΡ‚ΡŒ, Ρ‡Ρ‚ΠΎ каТСтся Ρ€Π°Π·ΡƒΠΌΠ½Ρ‹ΠΌ, Ρ‡Ρ‚ΠΎ эти пластины нСсут Π½ΡƒΠ»Π΅Π²ΠΎΠΉ заряд ΠΊΠΎΠ³Π΄Π° ΠΊ Π΄Π²ΡƒΠΌ кондСнсаторам ΠΏΡ€ΠΈΠ»ΠΎΠΆΠ΅Π½Π° нулСвая Ρ€Π°Π·Π½ΠΎΡΡ‚ΡŒ ΠΏΠΎΡ‚Π΅Π½Ρ†ΠΈΠ°Π»ΠΎΠ², слСдуСт Ρ‡Ρ‚ΠΎ ΠΏΡ€ΠΈ Π½Π°Π»ΠΈΡ‡ΠΈΠΈ Π½Π΅Π½ΡƒΠ»Π΅Π²ΠΎΠΉ разности ΠΏΠΎΡ‚Π΅Π½Ρ†ΠΈΠ°Π»ΠΎΠ² заряд Π½Π° ΠΏΠΎΠ»ΠΎΠΆΠΈΡ‚Π΅Π»ΡŒΠ½ΠΎΠΌ пластина кондСнсатора 2 Π΄ΠΎΠ»ΠΆΠ½Π° Π±Ρ‹Ρ‚ΡŒ ΡƒΡ€Π°Π²Π½ΠΎΠ²Π΅ΡˆΠ΅Π½Π° Ρ€Π°Π²Π½Ρ‹ΠΌ ΠΈ ΠΏΡ€ΠΎΡ‚ΠΈΠ²ΠΎΠΏΠΎΠ»ΠΎΠΆΠ½Ρ‹ΠΌ зарядом Π½Π° ΠΎΡ‚Ρ€ΠΈΡ†Π°Ρ‚Π΅Π»ΡŒΠ½ΠΎΠΉ пластинС кондСнсатора 1.ΠŸΠΎΡΠΊΠΎΠ»ΡŒΠΊΡƒ ΠΎΡ‚Ρ€ΠΈΡ†Π°Ρ‚Π΅Π»ΡŒΠ½Π°Ρ пластина ΠšΠΎΠ½Π΄Π΅Π½ΡΠ°Ρ‚ΠΎΡ€ 1 нСсСт заряд, ΠΏΠΎΠ»ΠΎΠΆΠΈΡ‚Π΅Π»ΡŒΠ½Π°Ρ пластина Π΄ΠΎΠ»ΠΆΠ½Π° нСсти заряд. Аналогичным ΠΎΠ±Ρ€Π°Π·ΠΎΠΌ, ΠΏΠΎΡΠΊΠΎΠ»ΡŒΠΊΡƒ ΠΏΠΎΠ»ΠΎΠΆΠΈΡ‚Π΅Π»ΡŒΠ½Π°Ρ пластина кондСнсатора 2 нСсСт заряд, ΠΎΡ‚Ρ€ΠΈΡ†Π°Ρ‚Π΅Π»ΡŒΠ½Π°Ρ пластина Π΄ΠΎΠ»ΠΆΠ½Π° нСсти заряд. Π’ ΠΈΡ‚ΠΎΠ³Π΅ ΠΎΠ±Π° кондСнсатора ΠΎΠ±Π»Π°Π΄Π°ΡŽΡ‚ Ρ‚Π°ΠΊΠΈΠΌ ΠΆΠ΅ Π½Π°ΠΊΠΎΠΏΠ»Π΅Π½Π½Ρ‹ΠΌ зарядом. ΠŸΠΎΡ‚Π΅Π½Ρ†ΠΈΠ°Π» ΠΏΠ°Π΄Π°Π΅Ρ‚, ΠΈ Π΄Π²Π° кондСнсатора, ΠΊΠ°ΠΊ ΠΏΡ€Π°Π²ΠΈΠ»ΠΎ, Ρ€Π°Π·Π½Ρ‹Π΅. Однако сумма этих ΠΏΠ°Π΄Π΅Π½ΠΈΠ΅ равняСтся ΠΎΠ±Ρ‰Π΅ΠΌΡƒ падСнию ΠΏΠΎΡ‚Π΅Π½Ρ†ΠΈΠ°Π»Π°, ΠΏΡ€ΠΈΠ»ΠΎΠΆΠ΅Π½Π½ΠΎΠΌΡƒ Π½Π° Π²Ρ…ΠΎΠ΄Π΅ ΠΈ Π²Ρ‹Ρ…ΠΎΠ΄Π΅ ΠΏΡ€ΠΎΠ²ΠΎΠ΄Π°: Ρ‚.Π΅. ,. ЭквивалСнтная Π΅ΠΌΠΊΠΎΡΡ‚ΡŒ ΠΏΠ°Ρ€Ρ‹ кондСнсаторы снова .Π’Π°ΠΊΠΈΠΌ ΠΎΠ±Ρ€Π°Π·ΠΎΠΌ,
    (115)

    давая
    (116)

    Π—Π΄Π΅ΡΡŒ ΠΌΡ‹ воспользовались Ρ‚Π΅ΠΌ Ρ„Π°ΠΊΡ‚ΠΎΠΌ, Ρ‡Ρ‚ΠΎ заряд являСтся ΠΎΠ±Ρ‰ΠΈΠΌ для всСх Ρ‚Ρ€Π΅Ρ… кондСнсаторы. Π‘Π»Π΅Π΄ΠΎΠ²Π°Ρ‚Π΅Π»ΡŒΠ½ΠΎ, ΠΏΡ€Π°Π²ΠΈΠ»ΠΎ Ρ‚Π°ΠΊΠΎΠ²ΠΎ:
    Π’Π΅Π»ΠΈΡ‡ΠΈΠ½Π°, обратная эквивалСнтной Смкости Π΄Π²ΡƒΡ… кондСнсаторов, ΠΏΠΎΠ΄ΠΊΠ»ΡŽΡ‡Π΅Π½Π½Ρ‹Ρ… Π² сСрия — это сумма ΠΎΠ±Ρ€Π°Ρ‚Π½Ρ‹Ρ… Π²Π΅Π»ΠΈΡ‡ΠΈΠ½ ΠΎΡ‚Π΄Π΅Π»ΡŒΠ½Ρ‹Ρ… СмкостСй.
    Для кондСнсаторов, соСдинСнных ΠΏΠΎΡΠ»Π΅Π΄ΠΎΠ²Π°Ρ‚Π΅Π»ΡŒΠ½ΠΎ, ΡƒΡ€Π°Π²Π½Π΅Π½ΠΈΠ΅.(116) ΠΎΠ±ΠΎΠ±Ρ‰Π°Π΅Ρ‚ Π½Π°

    Π”Π°Π»Π΅Π΅: ЭнСргия Π² кондСнсаторах Up: Π•ΠΌΠΊΠΎΡΡ‚ΡŒ ΠŸΡ€Π΅Π΄Ρ‹Π΄ΡƒΡ‰Π°Ρ: ДиэлСктрики
    Π ΠΈΡ‡Π°Ρ€Π΄ Π€ΠΈΡ†ΠΏΠ°Ρ‚Ρ€ΠΈΠΊ 2007-07-14

    ΠšΠžΠΠ”Π•ΠΠ‘ΠΠ’ΠžΠ Π« И Π”Π˜Π­Π›Π•ΠšΠ’Π Π˜ΠšΠ˜

    ΠšΠžΠΠ”Π•ΠΠ‘ΠΠ’ΠžΠ Π« И Π”Π˜Π­Π›Π•ΠšΠ’Π Π˜ΠšΠ˜

    ΠšΠΎΠ½Π΄Π΅Π½ΡΠ°Ρ‚ΠΎΡ€ — это Π½Π°Π±ΠΎΡ€ ΠΏΡ€ΠΎΠ²ΠΎΠ΄Π½ΠΈΠΊΠΎΠ², ΠΊΠΎΡ‚ΠΎΡ€Ρ‹ΠΉ ΠΈΡΠΏΠΎΠ»ΡŒΠ·ΡƒΠ΅Ρ‚ΡΡ для хранСния элСктричСского Π·Π°Ρ€ΡΠΆΠ°Ρ‚ΡŒ. ΠžΡ‡Π΅Π½ΡŒ простой кондСнсатор прСдставляСт собой ΠΈΠ·ΠΎΠ»ΠΈΡ€ΠΎΠ²Π°Π½Π½Ρ‹ΠΉ мСталличСский ΡˆΠ°Ρ€.ΠŸΠΎΡ‚Π΅Π½Ρ†ΠΈΠ°Π» ΡˆΠ°Ρ€Π° радиусом R ΠΈ зарядом Q Ρ€Π°Π²Π½ΠΎ

    (27,1)

    Π£Ρ€Π°Π²Π½Π΅Π½ΠΈΠ΅ (27.1) ΠΏΠΎΠΊΠ°Π·Ρ‹Π²Π°Π΅Ρ‚, Ρ‡Ρ‚ΠΎ ΠΏΠΎΡ‚Π΅Π½Ρ†ΠΈΠ°Π» сфСры ΠΏΡ€ΠΎΠΏΠΎΡ€Ρ†ΠΈΠΎΠ½Π°Π»Π΅Π½ Π·Π°Ρ€ΡΠ΄ΠΈΡ‚ΡŒ Q Π½Π° ΠΏΡ€ΠΎΠ²ΠΎΠ΄Π½ΠΈΠΊΠ΅. Π’ Ρ†Π΅Π»ΠΎΠΌ это Π²Π΅Ρ€Π½ΠΎ для любой ΠΊΠΎΠ½Ρ„ΠΈΠ³ΡƒΡ€Π°Ρ†ΠΈΠΈ ΠΏΡ€ΠΎΠ²ΠΎΠ΄Π½ΠΈΠΊΠΈ. Π­Ρ‚ΠΎ ΠΎΡ‚Π½ΠΎΡˆΠ΅Π½ΠΈΠ΅ ΠΌΠΎΠΆΠ½ΠΎ Π·Π°ΠΏΠΈΡΠ°Ρ‚ΡŒ ΠΊΠ°ΠΊ

    . (27,2)

    Π³Π΄Π΅ C называСтся Π΅ΠΌΠΊΠΎΡΡ‚ΡŒΡŽ систСмы ΠΏΡ€ΠΎΠ²ΠΎΠ΄Π½ΠΈΠΊΠΎΠ². Π•Π΄ΠΈΠ½ΠΈΡ†Π° измСрСния Смкости — Ρ„Π°Ρ€Π°Π΄ (Π€).Π•ΠΌΠΊΠΎΡΡ‚ΡŒ мСталличСский ΡˆΠ°Ρ€ Ρ€Π°Π²Π΅Π½

    (27,3)

    Π”Ρ€ΡƒΠ³ΠΎΠΉ ΠΏΡ€ΠΈΠΌΠ΅Ρ€ кондСнсатора — систСма, состоящая ΠΈΠ· Π΄Π²ΡƒΡ… ΠΏΠ°Ρ€Π°Π»Π»Π΅Π»ΡŒΠ½Ρ‹Ρ… мСталличСских Ρ‚Π°Ρ€Π΅Π»ΠΊΠΈ. Π’ Π³Π»Π°Π²Π΅ 26 Π±Ρ‹Π»ΠΎ ΠΏΠΎΠΊΠ°Π·Π°Π½ΠΎ, Ρ‡Ρ‚ΠΎ Ρ€Π°Π·Π½ΠΎΡΡ‚ΡŒ ΠΏΠΎΡ‚Π΅Π½Ρ†ΠΈΠ°Π»ΠΎΠ² ΠΌΠ΅ΠΆΠ΄Ρƒ двумя пластины области A, расстояниС раздСлСния d, ΠΈ с зарядами + Q ΠΈ -Q, задаСтся ΠΏΠΎ

    (27,4)

    Π˜ΡΠΏΠΎΠ»ΡŒΠ·ΡƒΡ ΠΎΠΏΡ€Π΅Π΄Π΅Π»Π΅Π½ΠΈΠ΅ Смкости (ΡƒΡ€.(27.2)) Π΅ΠΌΠΊΠΎΡΡ‚ΡŒ этого Π² систСмС ΠΌΠΎΠΆΠ½ΠΎ Ρ€Π°ΡΡΡ‡ΠΈΡ‚Π°Ρ‚ΡŒ:

    (27,5)

    Π£Ρ€Π°Π²Π½Π΅Π½ΠΈΠ΅ (27.2) ΠΏΠΎΠΊΠ°Π·Ρ‹Π²Π°Π΅Ρ‚, Ρ‡Ρ‚ΠΎ заряд кондСнсатора ΠΏΡ€ΠΎΠΏΠΎΡ€Ρ†ΠΈΠΎΠ½Π°Π»Π΅Π½ Смкости C ΠΈ ΠΏΠΎΡ‚Π΅Π½Ρ†ΠΈΠ°Π»Ρƒ V. Для увСличСния количСства Ρ…Ρ€Π°Π½ΠΈΠΌΠΎΠ³ΠΎ заряда Π½Π° кондСнсаторС, сохраняя постоянный ΠΏΠΎΡ‚Π΅Π½Ρ†ΠΈΠ°Π» (напряТСниС), Π΅ΠΌΠΊΠΎΡΡ‚ΡŒ кондСнсатор Π½ΡƒΠΆΠ½ΠΎ Π±ΡƒΠ΄Π΅Ρ‚ ΡƒΠ²Π΅Π»ΠΈΡ‡ΠΈΡ‚ΡŒ. ΠŸΠΎΡΠΊΠΎΠ»ΡŒΠΊΡƒ Π΅ΠΌΠΊΠΎΡΡ‚ΡŒ ΠΏΠ°Ρ€Π°Π»Π»Π΅Π»ΡŒΠ½ΠΎΠ³ΠΎ пластинчатый кондСнсатор ΠΏΡ€ΠΎΠΏΠΎΡ€Ρ†ΠΈΠΎΠ½Π°Π»Π΅Π½ ΠΏΠ»ΠΎΡ‰Π°Π΄ΠΈ пластины А ΠΈ ΠΎΠ±Ρ€Π°Ρ‚Π½ΠΎ ΠΏΡ€ΠΎΠΏΠΎΡ€Ρ†ΠΈΠΎΠ½Π°Π»Π΅Π½ Π½Π° расстояниС d ΠΌΠ΅ΠΆΠ΄Ρƒ пластинами, этого ΠΌΠΎΠΆΠ½ΠΎ Π΄ΠΎΠ±ΠΈΡ‚ΡŒΡΡ, ΡƒΠ²Π΅Π»ΠΈΡ‡ΠΈΠ² ΠΏΠ»ΠΎΡ‰Π°Π΄ΡŒ повСрхности A ΠΈ / ΠΈΠ»ΠΈ ΡƒΠΌΠ΅Π½ΡŒΡˆΠ΅Π½ΠΈΠ΅ Ρ€Π°Π·Π΄Π΅Π»ΠΈΡ‚Π΅Π»ΡŒΠ½ΠΎΠ³ΠΎ расстояния d.Π­Ρ‚ΠΈ большиС кондСнсаторы ΠΎΠ±Ρ‹Ρ‡Π½ΠΎ ΠΈΠ·Π³ΠΎΡ‚Π°Π²Π»ΠΈΠ²Π°ΡŽΡ‚ΡΡ ΠΈΠ· Π΄Π²ΡƒΡ… ΠΏΠ°Ρ€Π°Π»Π»Π΅Π»ΡŒΠ½Ρ‹Ρ… листов Π°Π»ΡŽΠΌΠΈΠ½ΠΈΠ·ΠΈΡ€ΠΎΠ²Π°Π½Π½ΠΎΠΉ Ρ„ΠΎΠ»ΡŒΠ³ΠΈ, нСсколько дюймов Π² ΡˆΠΈΡ€ΠΈΠ½Ρƒ ΠΈ нСсколько ΠΌΠ΅Ρ‚Ρ€ΠΎΠ² Π² Π΄Π»ΠΈΠ½Ρƒ. Листы Ρ€Π°ΡΠΏΠΎΠ»Π°Π³Π°ΡŽΡ‚ΡΡ ΠΎΡ‡Π΅Π½ΡŒ Π±Π»ΠΈΠ·ΠΊΠΎ вмСстС, Π½ΠΎ ΡƒΠ΄Π΅Ρ€ΠΆΠΈΠ²Π°Π΅ΠΌΡ‹ΠΉ ΠΎΡ‚ соприкосновСния Ρ‚ΠΎΠ½ΠΊΠΈΠΌ листом пластика, Π·Π°ΠΆΠ°Ρ‚Ρ‹ΠΌ ΠΌΠ΅ΠΆΠ΄Ρƒ ΠΈΡ…. Π’Π΅ΡΡŒ Π±ΡƒΡ‚Π΅Ρ€Π±Ρ€ΠΎΠ΄ Π½Π°ΠΊΡ€Ρ‹Π²Π°ΡŽΡ‚ Π΅Ρ‰Π΅ ΠΎΠ΄Π½ΠΈΠΌ листом пластика ΠΈ ΡΠΊΡ€ΡƒΡ‡ΠΈΠ²Π°ΡŽΡ‚. Π²Π²Π΅Ρ€Ρ…, ΠΊΠ°ΠΊ Ρ€ΡƒΠ»ΠΎΠ½ Ρ‚ΡƒΠ°Π»Π΅Ρ‚Π½ΠΎΠΉ Π±ΡƒΠΌΠ°Π³ΠΈ.

    ΠŸΡ€ΠΈΠΌΠ΅Ρ€: Π·Π°Π΄Π°Ρ‡Π° 27.7

    Π’Ρ€ΡƒΠ±ΠΊΠ° счСтчика Π“Π΅ΠΉΠ³Π΅Ρ€Π° состоит ΠΈΠ· Ρ‚ΠΎΠ½ΠΊΠΎΠΉ прямой ΠΏΡ€ΠΎΠ²ΠΎΠ»ΠΎΠΊΠΈ. ΠΎΠΊΡ€ΡƒΠΆΠ΅Π½ коаксиальной проводящСй ΠΎΠ±ΠΎΠ»ΠΎΡ‡ΠΊΠΎΠΉ.Π”ΠΈΠ°ΠΌΠ΅Ρ‚Ρ€ ΠΏΡ€ΠΎΠ²ΠΎΠ»ΠΎΠΊΠΈ 0,0025 см, Π° Ρ€Π°ΠΊΠΎΠ²ΠΈΠ½Ρ‹ — 2,5 см. Π”Π»ΠΈΠ½Π° Ρ‚Ρ€ΡƒΠ±ΠΊΠΈ 10 см. Π§Ρ‚ΠΎ Ρ‚Π°ΠΊΠΎΠ΅ Π΅ΠΌΠΊΠΎΡΡ‚ΡŒ Ρ‚Ρ€ΡƒΠ±ΠΊΠΈ счСтчика Π“Π΅ΠΉΠ³Π΅Ρ€Π°?

    Рисунок 27.1. Π‘Ρ…Π΅ΠΌΠ° счСтчика Π“Π΅ΠΉΠ³Π΅Ρ€Π°.

    Π—Π°Π΄Π°Ρ‡Π° Π±ΡƒΠ΄Π΅Ρ‚ Ρ€Π΅ΡˆΠ΅Π½Π° Π² ΠΏΡ€Π΅Π΄ΠΏΠΎΠ»ΠΎΠΆΠ΅Π½ΠΈΠΈ, Ρ‡Ρ‚ΠΎ элСктричСскоС ΠΏΠΎΠ»Π΅ гСнСрируСтся бСсконСчно Π΄Π»ΠΈΠ½Π½ΠΎΠΉ Π»ΠΈΠ½ΠΈΠ΅ΠΉ заряда. БхСматичСский Π²ΠΈΠ΄ сбоку Ρ‚Ρ€ΡƒΠ±Ρ‹ ΠΏΠΎΠΊΠ°Π·Π°Π½ Π½Π° рисункС 27.1. Радиус ΠΏΡ€ΠΎΠ²ΠΎΠ΄Π° r w , радиус Ρ†ΠΈΠ»ΠΈΠ½Π΄Ρ€Π° r c , Π΄Π»ΠΈΠ½Π° счСтчика L, Π° заряд Π½Π° ΠΏΡ€ΠΎΠ²ΠΎΠ΄Π΅ + Q.ЭлСктричСскоС ΠΏΠΎΠ»Π΅ Π² области ΠΌΠ΅ΠΆΠ΄Ρƒ ΠΏΡ€ΠΎΠ²ΠΎΠ»ΠΎΠΊΡƒ ΠΈ Ρ†ΠΈΠ»ΠΈΠ½Π΄Ρ€ ΠΌΠΎΠΆΠ½ΠΎ Ρ€Π°ΡΡΡ‡ΠΈΡ‚Π°Ρ‚ΡŒ ΠΏΠΎ Π·Π°ΠΊΠΎΠ½Ρƒ Гаусса. ЭлСктричСскоС ΠΏΠΎΠ»Π΅ Π² этой области Π±ΡƒΠ΄Π΅Ρ‚ ΠΈΠΌΠ΅Ρ‚ΡŒ Ρ€Π°Π΄ΠΈΠ°Π»ΡŒΠ½ΠΎΠ΅ Π½Π°ΠΏΡ€Π°Π²Π»Π΅Π½ΠΈΠ΅ ΠΈ Π΅Π³ΠΎ Π²Π΅Π»ΠΈΡ‡ΠΈΠ½Π° Π±ΡƒΠ΄Π΅Ρ‚ Π·Π°Π²ΠΈΡΠ΅Ρ‚ΡŒ Ρ‚ΠΎΠ»ΡŒΠΊΠΎ ΠΎΡ‚ Π½Π° Ρ€Π°Π΄ΠΈΠ°Π»ΡŒΠ½ΠΎΠΌ расстоянии r. Рассмотрим Ρ†ΠΈΠ»ΠΈΠ½Π΄Ρ€ Π΄Π»ΠΈΠ½ΠΎΠΉ L ΠΈ радиусом r ΠΏΠΎΠΊΠ°Π·Π°Π½ΠΎ Π½Π° рисункС 27.1. ЭлСктричСский ΠΏΠΎΡ‚ΠΎΠΊ [Phi] Ρ‡Π΅Ρ€Π΅Π· ΠΏΠΎΠ²Π΅Ρ€Ρ…Π½ΠΎΡΡ‚ΡŒ этого Ρ†ΠΈΠ»ΠΈΠ½Π΄Ρ€ Ρ€Π°Π²Π΅Π½

    (27,6)

    Богласно Π·Π°ΠΊΠΎΠ½Ρƒ Гаусса, ΠΏΠΎΡ‚ΠΎΠΊ [Phi] Ρ€Π°Π²Π΅Π½ Π²Π»ΠΎΠΆΠ΅Π½Π½ΠΎΠΌΡƒ заряду, Ρ€Π°Π·Π΄Π΅Π»Π΅Π½Π½ΠΎΠΌΡƒ Автор [epsilon] 0 .Π‘Π»Π΅Π΄ΠΎΠ²Π°Ρ‚Π΅Π»ΡŒΠ½ΠΎ,

    (27,7)

    ЭлСктричСскоС ΠΏΠΎΠ»Π΅ E (r) ΠΌΠΎΠΆΠ½ΠΎ ΠΏΠΎΠ»ΡƒΡ‡ΠΈΡ‚ΡŒ с ΠΏΠΎΠΌΠΎΡ‰ΡŒΡŽ уравнСния (27.7):

    (27,8)

    Π Π°Π·Π½ΠΎΡΡ‚ΡŒ ΠΏΠΎΡ‚Π΅Π½Ρ†ΠΈΠ°Π»ΠΎΠ² ΠΌΠ΅ΠΆΠ΄Ρƒ ΠΏΡ€ΠΎΠ²ΠΎΠ΄ΠΎΠΌ ΠΈ Ρ†ΠΈΠ»ΠΈΠ½Π΄Ρ€ΠΎΠΌ ΠΌΠΎΠΆΠ΅Ρ‚ Π±Ρ‹Ρ‚ΡŒ ΠΏΠΎΠ»ΡƒΡ‡Π΅Π½Π° ΡΠ»Π΅Π΄ΡƒΡŽΡ‰ΠΈΠΌ ΠΎΠ±Ρ€Π°Π·ΠΎΠΌ: интСгрируя элСктричСскоС ΠΏΠΎΠ»Π΅ E (r):

    (27,9)

    Π˜ΡΠΏΠΎΠ»ΡŒΠ·ΡƒΡ ΡƒΡ€Π°Π²Π½Π΅Π½ΠΈΠ΅ (27.2), ΠΌΠΎΠΆΠ½ΠΎ Ρ€Π°ΡΡΡ‡ΠΈΡ‚Π°Ρ‚ΡŒ Π΅ΠΌΠΊΠΎΡΡ‚ΡŒ Ρ‚Ρ€ΡƒΠ±ΠΊΠΈ Π“Π΅ΠΉΠ³Π΅Ρ€Π°:

    (27.10)

    ΠŸΠΎΠ΄ΡΡ‚Π°Π²Π»ΡΡ значСния для r w , r c ΠΈ L Π² ΡƒΡ€.(27.10) ΠΏΠΎΠ»ΡƒΡ‡Π°Π΅ΠΌ

    (27.11)

    Π‘ΠΈΠΌΠ²ΠΎΠ» кондСнсатора ΠΏΠΎΠΊΠ°Π·Π°Π½ Π½Π° рисункС 27.2. ΠšΠΎΠ½Π΄Π΅Π½ΡΠ°Ρ‚ΠΎΡ€Ρ‹ ΠΌΠΎΠ³ΡƒΡ‚ Π±Ρ‹Ρ‚ΡŒ соСдинСны вмСстС; ΠΎΠ½ΠΈ ΠΌΠΎΠ³ΡƒΡ‚ Π±Ρ‹Ρ‚ΡŒ ΠΏΠΎΠ΄ΠΊΠ»ΡŽΡ‡Π΅Π½Ρ‹ ΠΏΠΎΡΠ»Π΅Π΄ΠΎΠ²Π°Ρ‚Π΅Π»ΡŒΠ½ΠΎ ΠΈΠ»ΠΈ ΠΏΠ°Ρ€Π°Π»Π»Π΅Π»ΡŒΠ½ΠΎ. Π€ΠΈΠ³ΡƒΡ€Π° 27.3 ΠΏΠΎΠΊΠ°Π·Π°Π½Ρ‹ Π΄Π²Π° кондСнсатора Π΅ΠΌΠΊΠΎΡΡ‚ΡŒΡŽ C 1 ΠΈ C 2 , ΠΏΠΎΠ΄ΠΊΠ»ΡŽΡ‡Π΅Π½Ρ‹ ΠΏΠ°Ρ€Π°Π»Π»Π΅Π»ΡŒΠ½ΠΎ. Π Π°Π·Π½ΠΎΡΡ‚ΡŒ ΠΏΠΎΡ‚Π΅Π½Ρ†ΠΈΠ°Π»ΠΎΠ² Π½Π° ΠΎΠ±ΠΎΠΈΡ… кондСнсаторах Π΄ΠΎΠ»ΠΆΠ½Π° Π±Ρ‹Ρ‚ΡŒ Ρ€Π°Π²Π½ΠΎ ΠΈ, ΡΠ»Π΅Π΄ΠΎΠ²Π°Ρ‚Π΅Π»ΡŒΠ½ΠΎ,

    (27.12)

    Рисунок 27.2. Π‘ΠΈΠΌΠ²ΠΎΠ» кондСнсатора. Рисунок 27.3. Π”Π²Π° кондСнсатора ΠΏΠΎΠ΄ΠΊΠ»ΡŽΡ‡Π΅Π½Ρ‹ ΠΏΠ°Ρ€Π°Π»Π»Π΅Π»ΡŒΠ½ΠΎ.

    Π˜ΡΠΏΠΎΠ»ΡŒΠ·ΡƒΡ ΡƒΡ€Π°Π²Π½Π΅Π½ΠΈΠ΅ (27.12), ΠΌΠΎΠΆΠ½ΠΎ Ρ€Π°ΡΡΡ‡ΠΈΡ‚Π°Ρ‚ΡŒ ΠΎΠ±Ρ‰ΠΈΠΉ заряд ΠΎΠ±ΠΎΠΈΡ… кондСнсаторов

    (27,13)

    Π£Ρ€Π°Π²Π½Π΅Π½ΠΈΠ΅ (27.13) ΠΏΠΎΠΊΠ°Π·Ρ‹Π²Π°Π΅Ρ‚, Ρ‡Ρ‚ΠΎ ΠΏΠΎΠ»Π½Ρ‹ΠΉ заряд кондСнсаторной систСмы, ΠΏΠΎΠΊΠ°Π·Π°Π½Π½ΠΎΠΉ Π½Π° Рисунок 27.3 ΠΏΡ€ΠΎΠΏΠΎΡ€Ρ†ΠΈΠΎΠ½Π°Π»Π΅Π½ разности ΠΏΠΎΡ‚Π΅Π½Ρ†ΠΈΠ°Π»ΠΎΠ² Π² систСмС. Π’ Π΄Π²Π° кондСнсатора Π½Π° рисункС 27.3 ΠΌΠΎΠΆΠ½ΠΎ Ρ€Π°ΡΡΠΌΠ°Ρ‚Ρ€ΠΈΠ²Π°Ρ‚ΡŒ ΠΊΠ°ΠΊ ΠΎΠ΄ΠΈΠ½ кондСнсатор с Π΅ΠΌΠΊΠΎΡΡ‚ΡŒ C, Π³Π΄Π΅ C относится ΠΊ C 1 ΠΈ C 2 Π² ΡΠ»Π΅Π΄ΡƒΡŽΡ‰ΠΈΠΌ ΠΎΠ±Ρ€Π°Π·ΠΎΠΌ

    (27.14)

    На рисункС 27.4 ΠΏΠΎΠΊΠ°Π·Π°Π½Ρ‹ Π΄Π²Π° кондСнсатора Π΅ΠΌΠΊΠΎΡΡ‚ΡŒΡŽ C 1 ΠΈ C 2 , соСдинСны ΠΏΠΎΡΠ»Π΅Π΄ΠΎΠ²Π°Ρ‚Π΅Π»ΡŒΠ½ΠΎ. ΠŸΡ€Π΅Π΄ΠΏΠΎΠ»ΠΎΠΆΠΈΠΌ, Ρ‡Ρ‚ΠΎ Ρ€Π°Π·Π½ΠΎΡΡ‚ΡŒ ΠΏΠΎΡ‚Π΅Π½Ρ†ΠΈΠ°Π»ΠΎΠ² Π½Π° C 1 составляСт [Delta] V 1 , Π° Ρ€Π°Π·Π½ΠΎΡΡ‚ΡŒ ΠΏΠΎΡ‚Π΅Π½Ρ†ΠΈΠ°Π»ΠΎΠ² Π½Π° C 2 прСдставляСт собой [Delta] V 2 . Заряд Q Π½Π° Π²Π΅Ρ€Ρ…Π½Π΅ΠΉ пластинС Π²Ρ‹Π·ΠΎΠ²Π΅Ρ‚ заряд -Q Π½Π° Π½ΠΈΠΆΠ½Π΅ΠΉ пластинС C 1 . ΠŸΠΎΡΠΊΠΎΠ»ΡŒΠΊΡƒ элСктричСский заряд сохраняСтся, заряд Π½Π° Π²Π΅Ρ€Ρ…Π½Π΅ΠΉ пластинС C 2 Π΄ΠΎΠ»ΠΆΠ΅Π½ Π±Ρ‹Ρ‚ΡŒ Ρ€Π°Π²Π΅Π½ Q.Π’Π°ΠΊΠΈΠΌ ΠΎΠ±Ρ€Π°Π·ΠΎΠΌ, заряд Π½Π° Π½ΠΈΠΆΠ½Π΅ΠΉ пластинС C 2 Ρ€Π°Π²Π΅Π½ -Q. Π’ Ρ€Π°Π·Π½ΠΎΡΡ‚ΡŒ напряТСний Π½Π° C 1 опрСдСляСтся ΠΊΠ°ΠΊ

    (27.15)

    Π° Ρ€Π°Π·Π½ΠΈΡ†Π° напряТСний Π½Π° C 2 Ρ€Π°Π²Π½Π°

    (27.16)

    Рисунок 27.4. Π”Π²Π° кондСнсатора соСдинСны ΠΏΠΎΡΠ»Π΅Π΄ΠΎΠ²Π°Ρ‚Π΅Π»ΡŒΠ½ΠΎ. Π”Π°Π½Π° общая Ρ€Π°Π·Π½ΠΈΡ†Π° напряТСний Π½Π° Π΄Π²ΡƒΡ… кондСнсаторах. ΠΏΠΎ

    (27,17)

    Π£Ρ€Π°Π²Π½Π΅Π½ΠΈΠ΅ (27.17) снова ΠΏΠΎΠΊΠ°Π·Ρ‹Π²Π°Π΅Ρ‚, Ρ‡Ρ‚ΠΎ напряТСниС Π½Π° Π΄Π²ΡƒΡ… кондСнсаторах, соСдинСны ΠΏΠΎΡΠ»Π΅Π΄ΠΎΠ²Π°Ρ‚Π΅Π»ΡŒΠ½ΠΎ, ΠΏΡ€ΠΎΠΏΠΎΡ€Ρ†ΠΈΠΎΠ½Π°Π»ΡŒΠ½Π° заряду Q. БистСма дСйствуСт ΠΊΠ°ΠΊ ΠΎΠ΄ΠΈΠ½ΠΎΡ‡Π½Ρ‹ΠΉ кондСнсатор C, Π΅ΠΌΠΊΠΎΡΡ‚ΡŒ ΠΊΠΎΡ‚ΠΎΡ€ΠΎΠ³ΠΎ ΠΌΠΎΠΆΠ΅Ρ‚ Π±Ρ‹Ρ‚ΡŒ ΠΏΠΎΠ»ΡƒΡ‡Π΅Π½Π° ΠΈΠ· ΡΠ»Π΅Π΄ΡƒΡŽΡ‰ΠΈΡ… Ρ„ΠΎΡ€ΠΌΡƒΠ»Π°

    (27.18)

    ΠŸΡ€ΠΈΠΌΠ΅Ρ€: Π·Π°Π΄Π°Ρ‡Π° 27.10

    ΠœΠ½ΠΎΠ³ΠΎΠΏΠ»Π°ΡΡ‚ΠΈΠ½Ρ‡Π°Ρ‚Ρ‹ΠΉ кондСнсатор, ΠΈΡΠΏΠΎΠ»ΡŒΠ·ΡƒΠ΅ΠΌΡ‹ΠΉ, Π½Π°ΠΏΡ€ΠΈΠΌΠ΅Ρ€, Π² Ρ€Π°Π΄ΠΈΠΎΠΏΡ€ΠΈΠ΅ΠΌΠ½ΠΈΠΊΠ°Ρ…, состоит ΠΈΠ· Ρ‡Π΅Ρ‚Ρ‹Ρ€Π΅Ρ… ΠΏΠ°Ρ€Π°Π»Π»Π΅Π»ΡŒΠ½Ρ‹Π΅ пластины, располоТСнныС ΠΎΠ΄Π½Π° Π½Π°Π΄ Π΄Ρ€ΡƒΠ³ΠΎΠΉ, ΠΊΠ°ΠΊ ΠΏΠΎΠΊΠ°Π·Π°Π½ΠΎ Π½Π° рисункС 27.5. ΠŸΠ»ΠΎΡ‰Π°Π΄ΡŒ ΠΊΠ°ΠΆΠ΄ΠΎΠΉ пластины — А, Π° расстояниС ΠΌΠ΅ΠΆΠ΄Ρƒ сосСдними пластинами — d.Π§Ρ‚ΠΎ Π΅ΠΌΠΊΠΎΡΡ‚ΡŒ этого устройства?

    Рисунок 27.5. ΠœΠ½ΠΎΠ³ΠΎΠΏΠ»Π°ΡΡ‚ΠΈΠ½Ρ‡Π°Ρ‚Ρ‹ΠΉ кондСнсатор.

    ΠœΠ½ΠΎΠ³ΠΎΠΊΠ°Π½Π°Π»ΡŒΠ½Ρ‹ΠΉ кондСнсатор, ΠΏΠΎΠΊΠ°Π·Π°Π½Π½Ρ‹ΠΉ Π½Π° рисункС 27.5, эквивалСнтСн Ρ‚Ρ€Π΅ΠΌ ΠΈΠ΄Π΅Π½Ρ‚ΠΈΡ‡Π½Ρ‹ΠΌ кондСнсаторы, ΠΏΠΎΠ΄ΠΊΠ»ΡŽΡ‡Π΅Π½Π½Ρ‹Π΅ ΠΏΠ°Ρ€Π°Π»Π»Π΅Π»ΡŒΠ½ΠΎ (см. рисунок 27.6). Π•ΠΌΠΊΠΎΡΡ‚ΡŒ ΠΊΠ°ΠΆΠ΄ΠΎΠ³ΠΎ ΠΈΠ· Ρ‚Ρ€ΠΈ кондСнсатора Ρ€Π°Π²Π½Ρ‹ ΠΈ Ρ€Π°Π²Π½Ρ‹

    (27,19)

    Полная Π΅ΠΌΠΊΠΎΡΡ‚ΡŒ многопластинчатого кондСнсатора ΠΌΠΎΠΆΠ΅Ρ‚ Π±Ρ‹Ρ‚ΡŒ рассчитана с ΠΏΠΎΠΌΠΎΡ‰ΡŒΡŽ ΡƒΡ€Π°Π²Π½Π΅Π½ΠΈΠ΅ (27.14):

    (27.20)

    Рисунок 27.6. Π‘Ρ…Π΅ΠΌΠ° многопластинчатого кондСнсатора, ΠΏΠΎΠΊΠ°Π·Π°Π½Π½ΠΎΠ³ΠΎ Π½Π° рисункС 27.5.

    Π’Ρ€ΠΈ кондСнсатора, Π΅ΠΌΠΊΠΎΡΡ‚ΡŒΡŽ C 1 = 2,0 ΠΌΠΊΠ€, C 2 = 5,0 ΠΌΠΊΠ€ ΠΈ C 3 = 7,0 ΠΌΠΊΠ€, ΠΏΠ΅Ρ€Π²ΠΎΠ½Π°Ρ‡Π°Π»ΡŒΠ½ΠΎ Π·Π°Ρ€ΡΠΆΠ°ΡŽΡ‚ΡΡ Π΄ΠΎ 36 Π’ ΠΏΠΎΠ΄ΠΊΠ»ΡŽΡ‡ΠΈΠ² ΠΊΠ°ΠΆΠ΄ΡƒΡŽ Π½Π° нСсколько ΠΌΠ³Π½ΠΎΠ²Π΅Π½ΠΈΠΉ ΠΊ Π±Π°Ρ‚Π°Ρ€Π΅Π΅ Π½Π° 36 Π’. БатарСя Ρ‚ΠΎΠ³Π΄Π° сняты ΠΈ заряТСнныС кондСнсаторы Π²ΠΊΠ»ΡŽΡ‡Π΅Π½Ρ‹ Π² Π·Π°ΠΌΠΊΠ½ΡƒΡ‚ΡƒΡŽ ΠΏΠΎΡΠ»Π΅Π΄ΠΎΠ²Π°Ρ‚Π΅Π»ΡŒΠ½ΡƒΡŽ Ρ†Π΅ΠΏΡŒ, с соСдинСниСм ΠΏΠΎΠ»ΠΎΠΆΠΈΡ‚Π΅Π»ΡŒΠ½ΠΎΠΉ ΠΈ ΠΎΡ‚Ρ€ΠΈΡ†Π°Ρ‚Π΅Π»ΡŒΠ½ΠΎΠΉ ΠΊΠ»Π΅ΠΌΠΌ, ΠΊΠ°ΠΊ ΠΏΠΎΠΊΠ°Π·Π°Π½ΠΎ Π½Π° РисункС 27.7. Π§Ρ‚ΠΎ Π±ΡƒΠ΄Π΅Ρ‚ ΠΎΠΊΠΎΠ½Ρ‡Π°Ρ‚Π΅Π»ΡŒΠ½Ρ‹ΠΉ заряд Π½Π° ΠΊΠ°ΠΆΠ΄ΠΎΠΌ кондСнсаторС? КакоС Π±ΡƒΠ΄Π΅Ρ‚ напряТСниС Π½Π° Ρ‚ΠΎΡ‡ΠΊΠΈ PP ‘?

    Рисунок 27.7. ΠŸΡ€ΠΎΠ±Π»Π΅ΠΌΠ° 27.13.

    ΠΠ°Ρ‡Π°Π»ΡŒΠ½Ρ‹Π΅ заряды Π½Π° ΠΊΠ°ΠΆΠ΄ΠΎΠΌ ΠΈΠ· Ρ‚Ρ€Π΅Ρ… кондСнсаторов, q 1 , q 2 , q 3 , Ρ€Π°Π²Π½Ρ‹

    (27.21)

    ПослС ΠΏΠΎΠ΄ΠΊΠ»ΡŽΡ‡Π΅Π½ΠΈΡ Ρ‚Ρ€Π΅Ρ… кондСнсаторов заряд пСрСраспрСдСлится. Заряды Π½Π° Ρ‚Ρ€Π΅Ρ… кондСнсаторах послС Ρ‚ΠΎΠ³ΠΎ, ΠΊΠ°ΠΊ систСма успокоится, Ρ€Π°Π²Π½Ρ‹ Q 1 , Q 2 ΠΈ Q 3 .ΠŸΠΎΡΠΊΠΎΠ»ΡŒΠΊΡƒ заряд сохраняСтся количСство, сущСствуСт связь ΠΌΠ΅ΠΆΠ΄Ρƒ q 1 , q 2 ΠΈ q 3 ΠΈ Q 1 , Q 2 ΠΈ Q 3 :

    (27.22)

    НапряТСниС ΠΌΠ΅ΠΆΠ΄Ρƒ P ΠΈ P ‘ΠΌΠΎΠΆΠ½ΠΎ Π²Ρ‹Ρ€Π°Π·ΠΈΡ‚ΡŒ Ρ‡Π΅Ρ€Π΅Π· C 3 ΠΈ Q 3 , ΠΈΠ»ΠΈ Π² Ρ‚Π΅Ρ€ΠΌΠΈΠ½Π°Ρ… C 1 , C 2 , Q 1 ΠΈ Π’ 2 :

    (27.23)

    ΠΈ

    (27.24)

    Π˜ΡΠΏΠΎΠ»ΡŒΠ·ΡƒΡ ΡƒΡ€Π°Π²Π½Π΅Π½ΠΈΠ΅ (27.22), ΡΠ»Π΅Π΄ΡƒΡŽΡ‰ΠΈΠ΅ выраТСния для Q 1 ΠΈ Q 2 ΠΌΠΎΠΆΠ½ΠΎ ΠΏΠΎΠ»ΡƒΡ‡ΠΈΡ‚ΡŒ:

    (27,25)

    (27.26)

    ΠŸΠΎΠ΄ΡΡ‚Π°Π²Π»ΡΡ ΡƒΡ€Π°Π²Π½Π΅Π½ΠΈΠ΅ (27.25) ΠΈ ΡƒΡ€Π°Π²Π½Π΅Π½ΠΈΠ΅ (27.26) Π² ΡƒΡ€Π°Π²Π½Π΅Π½ΠΈΠ΅ (27.24), ΠΏΠΎΠ»ΡƒΡ‡Π°Π΅ΠΌ

    (27.27)

    ΠšΠΎΠΌΠ±ΠΈΠ½ΠΈΡ€ΡƒΡ ΡƒΡ€Π°Π²Π½Π΅Π½ΠΈΠ΅ (27.27) ΠΈ ΡƒΡ€Π°Π²Π½Π΅Π½ΠΈΠ΅ (27.23), Q 3 ΠΌΠΎΠΆΠ½ΠΎ Π²Ρ‹Ρ€Π°Π·ΠΈΡ‚ΡŒ Ρ‡Π΅Ρ€Π΅Π· извСстныС ΠΏΠ΅Ρ€Π΅ΠΌΠ΅Π½Π½Ρ‹Π΅:

    (27.28)

    ΠŸΠΎΠ΄ΡΡ‚Π°Π²Π»ΡΡ извСстныС значСния Смкости ΠΈ Π½Π°Ρ‡Π°Π»ΡŒΠ½Ρ‹Ρ… зарядов, ΠΌΡ‹ ΠΏΠΎΠ»ΡƒΡ‡ΠΈΡ‚ΡŒ

    (27.29)

    НапряТСниС Π½Π° P ΠΈ P ‘ΠΌΠΎΠΆΠ½ΠΎ Π½Π°ΠΉΡ‚ΠΈ, объСдинив уравнСния (27.29) ΠΈ ΡƒΡ€Π°Π²Π½Π΅Π½ΠΈΠ΅ (27.23):

    (27.30)

    Заряды кондСнсатора 1 ΠΈ кондСнсатора 2 Ρ€Π°Π²Π½Ρ‹

    . (27.31)

    (27.32)

    Если пространство ΠΌΠ΅ΠΆΠ΄Ρƒ пластинами кондСнсатора Π·Π°ΠΏΠΎΠ»Π½Π΅Π½ΠΎ изолятором, Π΅ΠΌΠΊΠΎΡΡ‚ΡŒ кондСнсатора Π±ΡƒΠ΄Π΅Ρ‚ случайной ΠΏΠΎ ΡΡ€Π°Π²Π½Π΅Π½ΠΈΡŽ с ситуациСй, Π² ΠΊΠΎΡ‚ΠΎΡ€ΠΎΠΉ ΠΌΠ΅ΠΆΠ΄Ρƒ пластинами Π΅ΡΡ‚ΡŒ Π²Π°ΠΊΡƒΡƒΠΌ.ИзмСнСниС Смкости Π²Ρ‹Π·Π²Π°Π½ΠΎ ΠΈΠ·ΠΌΠ΅Π½Π΅Π½ΠΈΠ΅ элСктричСского поля ΠΌΠ΅ΠΆΠ΄Ρƒ пластинами. ЭлСктричСскоС ΠΏΠΎΠ»Π΅ ΠΌΠ΅ΠΆΠ΄Ρƒ пластины кондСнсатора Π±ΡƒΠ΄ΡƒΡ‚ ΡΠΎΠ·Π΄Π°Π²Π°Ρ‚ΡŒ Π΄ΠΈΠΏΠΎΠ»ΡŒΠ½Ρ‹Π΅ ΠΌΠΎΠΌΠ΅Π½Ρ‚Ρ‹ Π² ΠΌΠ°Ρ‚Π΅Ρ€ΠΈΠ°Π»Π΅ ΠΌΠ΅ΠΆΠ΄Ρƒ Ρ‚Π°Ρ€Π΅Π»ΠΊΠΈ. Π­Ρ‚ΠΈ Π½Π°Π²Π΅Π΄Π΅Π½Π½Ρ‹Π΅ Π΄ΠΈΠΏΠΎΠ»ΡŒΠ½Ρ‹Π΅ ΠΌΠΎΠΌΠ΅Π½Ρ‚Ρ‹ ΡƒΠΌΠ΅Π½ΡŒΡˆΠ°Ρ‚ элСктричСскоС ΠΏΠΎΠ»Π΅ Π² ΠΎΠ±Π»Π°ΡΡ‚ΡŒ ΠΌΠ΅ΠΆΠ΄Ρƒ пластинами. ΠœΠ°Ρ‚Π΅Ρ€ΠΈΠ°Π», Π² ΠΊΠΎΡ‚ΠΎΡ€ΠΎΠΌ Π½Π°Π²Π΅Π΄Π΅Π½Π½Ρ‹ΠΉ Π΄ΠΈΠΏΠΎΠ»ΡŒΠ½Ρ‹ΠΉ ΠΌΠΎΠΌΠ΅Π½Ρ‚ Ρ€Π°Π²Π΅Π½ Π»ΠΈΠ½Π΅ΠΉΠ½ΠΎ ΠΏΡ€ΠΎΠΏΠΎΡ€Ρ†ΠΈΠΎΠ½Π°Π»ΡŒΠ½Ρ‹ΠΉ ΠΏΡ€ΠΈΠ»ΠΎΠΆΠ΅Π½Π½ΠΎΠΌΡƒ элСктричСскому полю называСтся Π»ΠΈΠ½Π΅ΠΉΠ½Ρ‹ΠΌ диэлСктрик . Π’ ΠΌΠ°Ρ‚Π΅Ρ€ΠΈΠ°Π»Π°Ρ… этого Ρ‚ΠΈΠΏΠ° ΠΏΠΎΠ»Π½ΠΎΠ΅ элСктричСскоС ΠΏΠΎΠ»Π΅ ΠΌΠ΅ΠΆΠ΄Ρƒ ΠΎΠ±ΠΊΠ»Π°Π΄ΠΊΠΈ кондСнсатора E связаны с элСктричСским ΠΏΠΎΠ»Π΅ΠΌ E free , ΠΊΠΎΡ‚ΠΎΡ€ΠΎΠ΅ сущСствовал Π±Ρ‹ Π±Π΅Π· диэлСктрика:

    (27.33)

    Π³Π΄Π΅ ΠΊΠ°ΠΏΠΏΠ° называСтся диэлСктричСской ΠΏΡ€ΠΎΠ½ΠΈΡ†Π°Π΅ΠΌΠΎΡΡ‚ΡŒΡŽ. ΠŸΠΎΡΠΊΠΎΠ»ΡŒΠΊΡƒ Ρ„ΠΈΠ½Π°Π»ΡŒΠ½Ρ‹ΠΉ элСктричСский ΠΏΠΎΠ»Π΅ E Π½ΠΈΠΊΠΎΠ³Π΄Π° Π½Π΅ ΠΌΠΎΠΆΠ΅Ρ‚ ΠΏΡ€Π΅Π²Ρ‹ΡˆΠ°Ρ‚ΡŒ свободноС элСктричСскоС ΠΏΠΎΠ»Π΅ E free , диэлСктричСская ΠΏΡ€ΠΎΠ½ΠΈΡ†Π°Π΅ΠΌΠΎΡΡ‚ΡŒ [ΠΊΠ°ΠΏΠΏΠ°] Π΄ΠΎΠ»ΠΆΠ½Π° Π±Ρ‹Ρ‚ΡŒ большС 1.

    Π Π°Π·Π½ΠΎΡΡ‚ΡŒ ΠΏΠΎΡ‚Π΅Π½Ρ†ΠΈΠ°Π»ΠΎΠ² Π½Π° кондСнсаторС ΠΏΡ€ΠΎΠΏΠΎΡ€Ρ†ΠΈΠΎΠ½Π°Π»ΡŒΠ½Π° элСктричСскому ΠΏΠΎΠ»Π΅ ΠΌΠ΅ΠΆΠ΄Ρƒ пластинами. ΠŸΠΎΡΠΊΠΎΠ»ΡŒΠΊΡƒ Π½Π°Π»ΠΈΡ‡ΠΈΠ΅ диэлСктрика сниТаСт напряТСнности элСктричСского поля, это Ρ‚Π°ΠΊΠΆΠ΅ ΡƒΠΌΠ΅Π½ΡŒΡˆΠΈΡ‚ Ρ€Π°Π·Π½ΠΎΡΡ‚ΡŒ ΠΏΠΎΡ‚Π΅Π½Ρ†ΠΈΠ°Π»ΠΎΠ² ΠΌΠ΅ΠΆΠ΄Ρƒ ΠΎΠ±ΠΊΠ»Π°Π΄ΠΊΠ°ΠΌΠΈ кондСнсатора (Ссли ΠΎΠ±Ρ‰ΠΈΠΉ заряд Π½Π° ΠΎΠ±ΠΊΠ»Π°Π΄ΠΊΠ°Ρ… сохраняСтся константа):

    (27.34)

    Π•ΠΌΠΊΠΎΡΡ‚ΡŒ C систСмы с диэлСктриком ΠΎΠ±Ρ€Π°Ρ‚Π½ΠΎ ΠΏΡ€ΠΎΠΏΠΎΡ€Ρ†ΠΈΠΎΠ½Π°Π»ΡŒΠ½Π° Ρ€Π°Π·Π½ΠΎΡΡ‚ΡŒ ΠΏΠΎΡ‚Π΅Π½Ρ†ΠΈΠ°Π»ΠΎΠ² ΠΌΠ΅ΠΆΠ΄Ρƒ пластинами ΠΈ связана с Π΅ΠΌΠΊΠΎΡΡ‚ΡŒΡŽ C свободный кондСнсатора Π±Π΅Π· диэлСктрика ΡΠ»Π΅Π΄ΡƒΡŽΡ‰ΠΈΠΌ ΠΎΠ±Ρ€Π°Π·ΠΎΠΌ

    (27.35)

    ΠŸΠΎΡΠΊΠΎΠ»ΡŒΠΊΡƒ [ΠΊΠ°ΠΏΠΏΠ°] большС 1, Π΅ΠΌΠΊΠΎΡΡ‚ΡŒ кондСнсатора ΠΌΠΎΠΆΠ΅Ρ‚ Π±Ρ‹Ρ‚ΡŒ Π·Π½Π°Ρ‡ΠΈΡ‚Π΅Π»ΡŒΠ½ΠΎ увСличиваСтся Π·Π° счСт заполнСния пространства ΠΌΠ΅ΠΆΠ΄Ρƒ ΠΎΠ±ΠΊΠ»Π°Π΄ΠΊΠ°ΠΌΠΈ кондСнсатора диэлСктрик с большой ΠΊΠ°ΠΏΠΏΠ°.

    ЭлСктричСскоС ΠΏΠΎΠ»Π΅ ΠΌΠ΅ΠΆΠ΄Ρƒ двумя ΠΎΠ±ΠΊΠ»Π°Π΄ΠΊΠ°ΠΌΠΈ кондСнсатора прСдставляСт собой Π²Π΅ΠΊΡ‚ΠΎΡ€Π½ΡƒΡŽ сумму поля, создаваСмыС зарядами Π½Π° кондСнсаторС, ΠΈ ΠΏΠΎΠ»Π΅, создаваСмоС повСрхностныС заряды Π½Π° повСрхности диэлСктрика. Π‘ΠΎΠ·Π΄Π°Π²Π°Π΅ΠΌΠΎΠ΅ элСктричСскоС ΠΏΠΎΠ»Π΅ зарядами Π½Π° ΠΎΠ±ΠΊΠ»Π°Π΄ΠΊΠ°Ρ… кондСнсатора (ΠΏΠ»ΠΎΡ‚Π½ΠΎΡΡ‚ΡŒ заряда [sigma] бСсплатно ) выдаСтся ΠΏΠΎ Π½ΠΎΠΌΠ΅Ρ€Ρƒ

    (27,36)

    Полагая ΠΏΠ»ΠΎΡ‚Π½ΠΎΡΡ‚ΡŒ заряда Π½Π° повСрхности диэлСктрика Ρ€Π°Π²Π½ΠΎΠΉ [sigma] bound , ΠΏΠΎΠ»Π΅, создаваСмоС этими связанными зарядами, Ρ€Π°Π²Π½ΠΎ Π½Π° Π½ΠΎΠΌΠ΅Ρ€

    (27.37)

    ЭлСктричСскоС ΠΏΠΎΠ»Π΅ ΠΌΠ΅ΠΆΠ΄Ρƒ пластинами Ρ€Π°Π²Π½ΠΎ E free / Каппа ΠΈ Ρ‚Π°ΠΊΠΈΠΌ ΠΎΠ±Ρ€Π°Π·ΠΎΠΌ

    (27.38)

    ΠŸΠΎΠ΄ΡΡ‚Π°Π²Π»ΡΡ ΡƒΡ€Π°Π²Π½Π΅Π½ΠΈΠ΅ (27,36) ΠΈ ΡƒΡ€Π°Π²Π½Π΅Π½ΠΈΠ΅ (27,37) Π² ΡƒΡ€Π°Π²Π½Π΅Π½ΠΈΠ΅ (27,38), ΠΏΠΎΠ»ΡƒΡ‡Π°Π΅ΠΌ

    (27.39)

    ΠΈΠ»ΠΈ

    (27,40)

    ΠŸΡ€ΠΈΠΌΠ΅Ρ€: Π·Π°Π΄Π°Ρ‡Π° 27.19

    ΠšΠΎΠ½Π΄Π΅Π½ΡΠ°Ρ‚ΠΎΡ€ с ΠΏΠ°Ρ€Π°Π»Π»Π΅Π»ΡŒΠ½Ρ‹ΠΌΠΈ пластинами с ΠΏΠ»ΠΎΡ‰Π°Π΄ΡŒΡŽ пластин A ΠΈ Ρ€Π°Π·Π΄Π΅Π»ΠΈΡ‚Π΅Π»ΡŒΠ½Ρ‹ΠΌ расстояниСм d содСрТит пластину диэлСктрика Ρ‚ΠΎΠ»Ρ‰ΠΈΠ½ΠΎΠΉ d / 2 (см. рисунок 27.8) ΠΈ диэлСктричСский постоянная ΠΊΠ°ΠΏΠΏΠ°. Π Π°Π·Π½ΠΎΡΡ‚ΡŒ ΠΏΠΎΡ‚Π΅Π½Ρ†ΠΈΠ°Π»ΠΎΠ² ΠΌΠ΅ΠΆΠ΄Ρƒ пластинами составляСт Ξ”V.

    Π°) НайдитС элСктричСскоС ΠΏΠΎΠ»Π΅ Π² пустом пространствС ΠΏΠΎ Π·Π°Π΄Π°Π½Π½Ρ‹ΠΌ Π²Π΅Π»ΠΈΡ‡ΠΈΠ½Π°ΠΌ. ΠΎΠ±Π»Π°ΡΡ‚ΡŒ пространства ΠΌΠ΅ΠΆΠ΄Ρƒ пластинами.

    Π±) НайдитС элСктричСскоС ΠΏΠΎΠ»Π΅ Π²Π½ΡƒΡ‚Ρ€ΠΈ диэлСктрика.

    Π²) НайдитС ΠΏΠ»ΠΎΡ‚Π½ΠΎΡΡ‚ΡŒ связанных зарядов Π½Π° повСрхности диэлСктрика.

    Рисунок 27.8. ΠŸΡ€ΠΎΠ±Π»Π΅ΠΌΠ° 27.19.

    Π°) ΠŸΡ€Π΅Π΄ΠΏΠΎΠ»ΠΎΠΆΠΈΠΌ, Ρ‡Ρ‚ΠΎ элСктричСскоС ΠΏΠΎΠ»Π΅ Π² кондСнсаторС Π±Π΅Π· диэлСктрика Ρ€Π°Π²Π½ΠΎ ΠΊ E 0 .ЭлСктричСскоС ΠΏΠΎΠ»Π΅ Π² диэлСктрикС E d Ρ€Π°Π²Π½ΠΎ связано со свободным элСктричСским ΠΏΠΎΠ»Π΅ΠΌ Ρ‡Π΅Ρ€Π΅Π· Π΄ΠΈΡΠ»Π΅ΠΊΡ‚Ρ€ΠΈΡ‡Π΅ΡΠΊΡƒΡŽ ΠΏΡ€ΠΎΠ½ΠΈΡ†Π°Π΅ΠΌΠΎΡΡ‚ΡŒ [ΠΊΠ°ΠΏΠΏΠ°]:

    (27.41)

    Π Π°Π·Π½ΠΎΡΡ‚ΡŒ ΠΏΠΎΡ‚Π΅Π½Ρ†ΠΈΠ°Π»ΠΎΠ² ΠΌΠ΅ΠΆΠ΄Ρƒ пластинами ΠΌΠΎΠΆΠ΅Ρ‚ Π±Ρ‹Ρ‚ΡŒ ΠΏΠΎΠ»ΡƒΡ‡Π΅Π½Π° ΠΏΡƒΡ‚Π΅ΠΌ интСгрирования элСктричСскоС ΠΏΠΎΠ»Π΅ ΠΌΠ΅ΠΆΠ΄Ρƒ пластинами:

    (27,42)

    Π’Π°ΠΊΠΈΠΌ ΠΎΠ±Ρ€Π°Π·ΠΎΠΌ, элСктричСскоС ΠΏΠΎΠ»Π΅ Π² пустой области Ρ€Π°Π²Π½ΠΎ

    . (27,43)

    Π±) ЭлСктричСскоС ΠΏΠΎΠ»Π΅ Π² диэлСктрикС ΠΌΠΎΠΆΠ½ΠΎ Π½Π°ΠΉΡ‚ΠΈ, комбинируя ΡƒΡ€.(27,41) ΠΈ (27.43):

    (27,44)

    c) ΠŸΠ»ΠΎΡ‚Π½ΠΎΡΡ‚ΡŒ свободного заряда [сигма] free Ρ€Π°Π²Π½Π°

    (27,45)

    ΠŸΠ»ΠΎΡ‚Π½ΠΎΡΡ‚ΡŒ связанного заряда связана с ΠΏΠ»ΠΎΡ‚Π½ΠΎΡΡ‚ΡŒΡŽ свободного заряда Ρ‡Π΅Ρ€Π΅Π· ΡΠ»Π΅Π΄ΡƒΡŽΡ‰Π΅Π΅ ΠΎΡ‚Π½ΠΎΡˆΠ΅Π½ΠΈΠ΅

    (27,46)

    ΠšΠΎΠΌΠ±ΠΈΠ½ΠΈΡ€ΡƒΡ ΡƒΡ€Π°Π²Π½Π΅Π½ΠΈΠ΅ (27.45) ΠΈ ΡƒΡ€Π°Π²Π½Π΅Π½ΠΈΠ΅ (27.46), ΠΏΠΎΠ»ΡƒΡ‡Π°Π΅ΠΌ

    (27,47)

    ЭлСктричСскоС ΠΏΠΎΠ»Π΅ Π² «пустом» кондСнсаторС ΠΌΠΎΠΆΠ½ΠΎ ΠΏΠΎΠ»ΡƒΡ‡ΠΈΡ‚ΡŒ с ΠΏΠΎΠΌΠΎΡ‰ΡŒΡŽ Π·Π°ΠΊΠΎΠ½Π° Гаусса.Рассмотрим ΠΈΠ΄Π΅Π°Π»ΡŒΠ½Ρ‹ΠΉ кондСнсатор (Π±Π΅Π· ΠΊΡ€Π°Π΅Π²Ρ‹Ρ… ΠΏΠΎΠ»Π΅ΠΉ) ΠΈ ΠΈΠ½Ρ‚Π΅Π³Ρ€ΠΈΡ€ΠΎΠ²Π°Π½ΠΈΠ΅ объСм, ΠΏΠΎΠΊΠ°Π·Π°Π½Π½Ρ‹ΠΉ Π½Π° рисункС 27.9. ΠŸΠ»ΠΎΡ‰Π°Π΄ΡŒ ΠΊΠ°ΠΆΠ΄ΠΎΠΉ пластины кондСнсатора — А, Π° ΠΏΠ»ΠΎΡ‰Π°Π΄ΡŒ заряды Π½Π° пластинах +/- Q. Заряд, Π·Π°ΠΊΠ»ΡŽΡ‡Π΅Π½Π½Ρ‹ΠΉ Π² ΠΎΠ±ΡŠΡ‘ΠΌΠ΅ ΠΈΠ½Ρ‚Π΅Π³Ρ€Π°Ρ†ΠΈΠΈ ΠΏΠΎΠΊΠ°Π·Π°Π½Π½ΠΎΠ΅ Π½Π° рисункС 27.9, Ρ€Π°Π²Π½ΠΎ + Q. Π—Π°ΠΊΠΎΠ½ Гаусса гласит, Ρ‡Ρ‚ΠΎ элСктричСский ΠΏΠΎΡ‚ΠΎΠΊ [Phi] Ρ‡Π΅Ρ€Π΅Π· ΠΏΠΎΠ²Π΅Ρ€Ρ…Π½ΠΎΡΡ‚ΡŒ ΠΎΠ±ΡŠΡ‘ΠΌΠ° интСгрирования относится ΠΊ ΠΏΡ€ΠΈΠ»ΠΎΠΆΠ΅Π½Π½ΠΎΠΌΡƒ ΠΏΠ»Π°Ρ‚Π°:

    (27,48)

    Если ΠΌΠ΅ΠΆΠ΄Ρƒ пластинами вставлСн диэлСктрик, элСктричСскоС ΠΏΠΎΠ»Π΅ ΠΌΠ΅ΠΆΠ΄Ρƒ пластинами пластины Π±ΡƒΠ΄ΡƒΡ‚ ΠΌΠ΅Π½ΡΡ‚ΡŒΡΡ (Π΄Π°ΠΆΠ΅ Ссли заряд Π½Π° пластинах остаСтся постоянным).ΠžΡ‡Π΅Π²ΠΈΠ΄Π½ΠΎ, Ρ‡Ρ‚ΠΎ Π·Π°ΠΊΠΎΠ½ Гаусса, сформулированный Π² ΡƒΡ€Π°Π²Π½Π΅Π½ΠΈΠΈ (27.48), Π² этом случаС Π½Π΅ выполняСтся. ЭлСктричСскоС ΠΏΠΎΠ»Π΅ E ΠΌΠ΅ΠΆΠ΄Ρƒ ΠΎΠ±ΠΊΠ»Π°Π΄ΠΊΠ°ΠΌΠΈ кондСнсатора связано с бСздиэлСктричСскоС ΠΏΠΎΠ»Π΅ E свободноС :

    (27,49)

    Π³Π΄Π΅ [ΠΊΠ°ΠΏΠΏΠ°] — диэлСктричСская ΠΏΡ€ΠΎΠ½ΠΈΡ†Π°Π΅ΠΌΠΎΡΡ‚ΡŒ ΠΌΠ°Ρ‚Π΅Ρ€ΠΈΠ°Π»Π° ΠΌΠ΅ΠΆΠ΄Ρƒ пластинами. Π’Π΅ΠΏΠ΅Ρ€ΡŒ Π·Π°ΠΊΠΎΠ½ Гаусса ΠΌΠΎΠΆΠ½ΠΎ ΠΏΠ΅Ρ€Π΅ΠΏΠΈΡΠ°Ρ‚ΡŒ ΠΊΠ°ΠΊ

    (27,50)

    Π—Π°ΠΊΠΎΠ½ Гаусса Π² Π²Π°ΠΊΡƒΡƒΠΌΠ΅ являСтся частным случаСм уравнСния (27.50) с [ΠΊΠ°ΠΏΠΏΠ°] = 1.

    Рисунок 27.9. Π˜Π΄Π΅Π°Π»ΡŒΠ½Ρ‹ΠΉ кондСнсатор.

    ΠŸΡ€ΠΈΠΌΠ΅Ρ€: Π·Π°Π΄Π°Ρ‡Π° 27.25

    ΠœΠ΅Ρ‚Π°Π»Π»ΠΈΡ‡Π΅ΡΠΊΠΈΠΉ ΡˆΠ°Ρ€ радиуса R ΠΎΠΊΡ€ΡƒΠΆΠ΅Π½ концСнтричСским диэлСктриком. ΠΎΠ±ΠΎΠ»ΠΎΡ‡ΠΊΠ° с Π²Π½ΡƒΡ‚Ρ€Π΅Π½Π½ΠΈΠΌ радиусом R ΠΈ внСшним радиусом 3R / 2. Π­Ρ‚ΠΎ ΠΎΠΊΡ€ΡƒΠΆΠ΅Π½ΠΎ концСнтричСская тонкая мСталличСская ΠΎΠ±ΠΎΠ»ΠΎΡ‡ΠΊΠ° радиуса 2R (см. рисунок 27.10). Π’ диэлСктричСская ΠΏΡ€ΠΎΠ½ΠΈΡ†Π°Π΅ΠΌΠΎΡΡ‚ΡŒ ΠΎΠ±ΠΎΠ»ΠΎΡ‡ΠΊΠΈ ΠΊΠ°ΠΏΠΏΠ°. Какая Ρƒ этого Π΅ΠΌΠΊΠΎΡΡ‚ΡŒ Ρ…ΠΈΡ‚Ρ€ΠΎΠ΅ ΠΈΠ·ΠΎΠ±Ρ€Π΅Ρ‚Π΅Π½ΠΈΠ΅?

    ΠŸΡ€Π΅Π΄ΠΏΠΎΠ»ΠΎΠΆΠΈΠΌ, Ρ‡Ρ‚ΠΎ заряд Π½Π° Π²Π½ΡƒΡ‚Ρ€Π΅Π½Π½Π΅ΠΉ сфСрС Ρ€Π°Π²Π΅Π½ Q бСсплатно .ЭлСктричСский ΠΏΠΎΠ»Π΅ Π²Π½ΡƒΡ‚Ρ€ΠΈ диэлСктрика ΠΌΠΎΠΆΠ½ΠΎ ΠΎΠΏΡ€Π΅Π΄Π΅Π»ΠΈΡ‚ΡŒ, ΠΏΡ€ΠΈΠΌΠ΅Π½ΠΈΠ² Π·Π°ΠΊΠΎΠ½ Гаусса для диэлСктрика (ΡƒΡ€Π°Π²Π½Π΅Π½ΠΈΠ΅ (27.50)) ΠΈ ΠΈΡΠΏΠΎΠ»ΡŒΠ·ΡƒΡ Π² качСствС объСма интСгрирования сфСру радиуса r (Π³Π΄Π΅ R

    (27.51)

    Π‘Π»Π΅Π΄ΠΎΠ²Π°Ρ‚Π΅Π»ΡŒΠ½ΠΎ, элСктричСскоС ΠΏΠΎΠ»Π΅ Π² этой области Ρ€Π°Π²Π½ΠΎ

    . (27.52)

    Рисунок 27.10. ΠŸΡ€ΠΎΠ±Π»Π΅ΠΌΠ° 27.25. ЭлСктричСскоС ΠΏΠΎΠ»Π΅ Π² области ΠΌΠ΅ΠΆΠ΄Ρƒ 3R / 2 ΠΈ 2R ΠΌΠΎΠΆΠ΅Ρ‚ Π±Ρ‹Ρ‚ΡŒ ΠΏΠΎΠ»ΡƒΡ‡Π΅Π½Π½Ρ‹ΠΉ Π°Π½Π°Π»ΠΎΠ³ΠΈΡ‡Π½Ρ‹ΠΌ ΠΎΠ±Ρ€Π°Π·ΠΎΠΌ, ΠΈ Ρ€Π°Π²Π΅Π½

    (27.53)

    Π˜ΡΠΏΠΎΠ»ΡŒΠ·ΡƒΡ элСктричСскоС ΠΏΠΎΠ»Π΅ ΠΈΠ· ΡƒΡ€Π°Π²Π½Π΅Π½ΠΈΠΉ (27.52) ΠΈ (27.53), ΠΌΡ‹ ΠΌΠΎΠΆΠ΅ΠΌ ΠΎΠΏΡ€Π΅Π΄Π΅Π»ΠΈΡ‚ΡŒ Ρ€Π°Π·Π½ΠΎΡΡ‚ΡŒ ΠΏΠΎΡ‚Π΅Π½Ρ†ΠΈΠ°Π»ΠΎΠ² [Π”Π΅Π»ΡŒΡ‚Π°] V ΠΌΠ΅ΠΆΠ΄Ρƒ Π²Π½ΡƒΡ‚Ρ€Π΅Π½Π½Π΅ΠΉ ΠΈ внСшнСй сфСрой:

    (27,54)

    Π•ΠΌΠΊΠΎΡΡ‚ΡŒ систСмы ΠΌΠΎΠΆΠ΅Ρ‚ Π±Ρ‹Ρ‚ΡŒ ΠΏΠΎΠ»ΡƒΡ‡Π΅Π½Π° ΠΈΠ· уравнСния (27.54) с использованиСм ΠΎΠΏΡ€Π΅Π΄Π΅Π»Π΅Π½ΠΈΠ΅ Смкости Ρ‡Π΅Ρ€Π΅Π· заряд Q ΠΈ ΠΏΠΎΡ‚Π΅Π½Ρ†ΠΈΠ°Π» Ρ€Π°Π·Π½ΠΈΡ†Π° [Delta] V:

    (27,55)

    ЭлСктричСская ΠΏΠΎΡ‚Π΅Π½Ρ†ΠΈΠ°Π»ΡŒΠ½Π°Ρ энСргия кондСнсатора, Π½Π΅ содСрТащСго диэлСктрика ΠΈ ΠΈΠΌΠ΅ΡŽΡ‰Π΅Π³ΠΎ заряд +/- Q Π½Π° Π΅Π³ΠΎ пластинах Ρ€Π°Π²Π΅Π½

    (27.56)

    Π³Π΄Π΅ V 1 ΠΈ V 2 — ΠΏΠΎΡ‚Π΅Π½Ρ†ΠΈΠ°Π»Ρ‹ Π΄Π²ΡƒΡ… пластин. ЭлСктричСская ΠΏΠΎΡ‚Π΅Π½Ρ†ΠΈΠ°Π»ΡŒΠ½Π°Ρ энСргия Ρ‚Π°ΠΊΠΆΠ΅ ΠΌΠΎΠΆΠ΅Ρ‚ Π±Ρ‹Ρ‚ΡŒ Π²Ρ‹Ρ€Π°ΠΆΠ΅Π½Π° Ρ‡Π΅Ρ€Π΅Π· Π΅ΠΌΠΊΠΎΡΡ‚ΡŒ Π‘ кондСнсатора

    (27,57)

    Π­Ρ‚Π° Ρ„ΠΎΡ€ΠΌΡƒΠ»Π° Π²Π΅Ρ€Π½Π° ΠΈ для кондСнсатора с диэлСктриком; свойства диэлСктрика Π²Ρ…ΠΎΠ΄ΠΈΡ‚ Π² эту Ρ„ΠΎΡ€ΠΌΡƒΠ»Ρƒ Ρ‡Π΅Ρ€Π΅Π· Π΅ΠΌΠΊΠΎΡΡ‚ΡŒ C.

    ΠŸΡ€ΠΈΠΌΠ΅Ρ€: Π·Π°Π΄Π°Ρ‡Π° 27.40

    Π”Π΅ΡΡΡ‚ΡŒ ΠΈΠ΄Π΅Π½Ρ‚ΠΈΡ‡Π½Ρ‹Ρ… кондСнсаторов ΠΏΠΎ 5 ΠΌΠΊΠ€ ΠΏΠΎΠ΄ΠΊΠ»ΡŽΡ‡Π΅Π½Ρ‹ ΠΏΠ°Ρ€Π°Π»Π»Π΅Π»ΡŒΠ½ΠΎ ΠΊ источнику питания 240 Π’. аккумулятор.Π—Π°Ρ‚Π΅ΠΌ заряТСнныС кондСнсаторы ΠΎΡ‚ΠΊΠ»ΡŽΡ‡Π°ΡŽΡ‚ΡΡ ΠΎΡ‚ аккумулятора ΠΈ ΠΏΠΎΠ²Ρ‚ΠΎΡ€Π½ΠΎ ΠΏΠΎΠ΄ΠΊΠ»ΡŽΡ‡Π΅Π½Ρ‹ ΠΏΠΎΡΠ»Π΅Π΄ΠΎΠ²Π°Ρ‚Π΅Π»ΡŒΠ½ΠΎ, ΠΏΠΎΠ»ΠΎΠΆΠΈΡ‚Π΅Π»ΡŒΠ½Ρ‹ΠΉ Π²Ρ‹Π²ΠΎΠ΄ ΠΊΠ°ΠΆΠ΄ΠΎΠ³ΠΎ кондСнсатора ΠΏΠΎΠ΄ΠΊΠ»ΡŽΡ‡Π°Π΅Ρ‚ΡΡ ΠΊ ΠΎΡ‚Ρ€ΠΈΡ†Π°Ρ‚Π΅Π»ΡŒΠ½ΠΎΠΉ ΠΊΠ»Π΅ΠΌΠΌΠ΅ ΡΠ»Π΅Π΄ΡƒΡŽΡ‰Π΅Π³ΠΎ. Π’ Ρ‡Π΅ΠΌ Ρ€Π°Π·Π½ΠΈΡ†Π° ΠΏΠΎΡ‚Π΅Π½Ρ†ΠΈΠ°Π»ΠΎΠ² ΠΌΠ΅ΠΆΠ΄Ρƒ ΠΎΡ‚Ρ€ΠΈΡ†Π°Ρ‚Π΅Π»ΡŒΠ½Ρ‹ΠΉ Π²Ρ‹Π²ΠΎΠ΄ ΠΏΠ΅Ρ€Π²ΠΎΠ³ΠΎ кондСнсатора ΠΈ ΠΏΠΎΠ»ΠΎΠΆΠΈΡ‚Π΅Π»ΡŒΠ½Ρ‹ΠΉ Π²Ρ‹Π²ΠΎΠ΄ послСдний кондСнсатор? Если эти ΠΊΠ»Π΅ΠΌΠΌΡ‹ ΠΏΠΎΠ΄ΠΊΠ»ΡŽΡ‡Π΅Π½Ρ‹ Ρ‡Π΅Ρ€Π΅Π· внСшнюю Ρ†Π΅ΠΏΡŒ, ΠΊΠ°ΠΊ большая Ρ‡Π°ΡΡ‚ΡŒ заряда Π±ΡƒΠ΄Π΅Ρ‚ Ρ‚Π΅Ρ‡ΡŒ ΠΏΠΎ этой Ρ†Π΅ΠΏΠΈ ΠΏΡ€ΠΈ ΠΏΠΎΡΠ»Π΅Π΄ΠΎΠ²Π°Ρ‚Π΅Π»ΡŒΠ½ΠΎΠΌ разрядС. ? Бколько энСргии выдСляСтся ΠΏΡ€ΠΈ разрядС? Π‘Ρ€Π°Π²Π½ΠΈΡ‚Π΅ это ΠΎΠ±Π²ΠΈΠ½Π΅Π½ΠΈΠ΅ ΠΈ это энСргия с зарядом ΠΈ энСргиСй, хранящСйся Π² исходном, ΠΏΠ°Ρ€Π°Π»Π»Π΅Π»ΡŒΠ½ΠΎΠΌ располоТСнии, ΠΈ ΠΎΠ±ΡŠΡΡΠ½ΠΈΡ‚Π΅ Π»ΡŽΠ±Ρ‹Π΅ нСточности.

    Заряд Π½Π° ΠΊΠ°ΠΆΠ΄ΠΎΠΌ кондСнсаторС послС ΠΏΠΎΠ΄ΠΊΠ»ΡŽΡ‡Π΅Π½ΠΈΡ ΠΊ Π±Π°Ρ‚Π°Ρ€Π΅Π΅ 240 Π’ составляСт Ρ€Π°Π²Π½ΠΎ

    (27,58)

    Π Π°Π·Π½ΠΎΡΡ‚ΡŒ ΠΏΠΎΡ‚Π΅Π½Ρ†ΠΈΠ°Π»ΠΎΠ² Π½Π° ΠΊΠ°ΠΆΠ΄ΠΎΠΌ кондСнсаторС останСтся Ρ€Π°Π²Π½ΠΎΠΉ 240 Π’ послС кондСнсаторы Π²ΠΊΠ»ΡŽΡ‡Π΅Π½Ρ‹ ΠΏΠΎΡΠ»Π΅Π΄ΠΎΠ²Π°Ρ‚Π΅Π»ΡŒΠ½ΠΎ. ΠžΠ±Ρ‰Π°Ρ Ρ€Π°Π·Π½ΠΎΡΡ‚ΡŒ ΠΏΠΎΡ‚Π΅Π½Ρ†ΠΈΠ°Π»ΠΎΠ² ΠΏΠΎ Π΄Π΅ΡΡΡ‚ΡŒ кондСнсаторов, Ρ‚Π°ΠΊΠΈΠΌ ΠΎΠ±Ρ€Π°Π·ΠΎΠΌ, Ρ€Π°Π²Π½Ρ‹

    (27,59)

    Если Π΄Π²Π° ΠΊΠΎΠ½Ρ†Π΅Π²Ρ‹Ρ… Π²Ρ‹Π²ΠΎΠ΄Π° кондСнсаторной сСти соСдинСны, заряд 1.2 мкКл Π±ΡƒΠ΄Π΅Ρ‚ Ρ‚Π΅Ρ‡ΡŒ ΠΎΡ‚ ΠΏΠΎΠ»ΠΎΠΆΠΈΡ‚Π΅Π»ΡŒΠ½ΠΎΠΉ ΠΊΠ»Π΅ΠΌΠΌΡ‹ ΠΊ ΠΎΡ‚Ρ€ΠΈΡ†Π°Ρ‚Π΅Π»ΡŒΠ½ΠΎΠΉ (см. Рисунок 27.11).

    Рисунок 27.11. ΠŸΡ€ΠΎΠ±Π»Π΅ΠΌΠ° 27.40. ЭлСктричСская энСргия, запасСнная Π² кондСнсаторной сСти ΠΏΠ΅Ρ€Π΅Π΄ разряд Ρ€Π°Π²Π΅Π½

    (27.60)

    ЭнСргия, запасСнная Π² ΠΊΠ°ΠΆΠ΄ΠΎΠΌ кондСнсаторС послС заряда Π΄ΠΎ 240 Π’, Ρ€Π°Π²Π½Π° Π½Π° Π½ΠΎΠΌΠ΅Ρ€

    (27.61)

    ΠžΡ‡Π΅Π²ΠΈΠ΄Π½ΠΎ, Ρ‡Ρ‚ΠΎ ΠΏΡ€ΠΈ Π·Π°ΠΌΠ΅Π½Π΅ кондСнсатора энСргия Π½Π΅ тСряСтся. конфигурация с ΠΏΠ°Ρ€Π°Π»Π»Π΅Π»ΡŒΠ½ΠΎΠ³ΠΎ Π½Π° ΠΏΠΎΡΠ»Π΅Π΄ΠΎΠ²Π°Ρ‚Π΅Π»ΡŒΠ½Ρ‹ΠΉ.

    ΠŸΡ€ΠΈΠΌΠ΅Ρ€: Π·Π°Π΄Π°Ρ‡Π° 27.39

    Π’Ρ€ΠΈ кондСнсатора ΠΏΠΎΠ΄ΠΊΠ»ΡŽΡ‡Π΅Π½Ρ‹, ΠΊΠ°ΠΊ ΠΏΠΎΠΊΠ°Π·Π°Π½ΠΎ Π½Π° рисункС 27.12. Π˜Ρ… Смкости ΡΠΎΡΡ‚Π°Π²Π»ΡΡŽΡ‚ C 1 = 2,0 ΠΌΠΊΠ€, C 2 = 6,0 ΠΌΠΊΠ€ ΠΈ C 3 = 8,0 ΠΌΠΊΠ€. Если Π½Π° Π΄Π²Π° свободных ΠΊΠ»Π΅ΠΌΠΌΡ‹, ΠΊΠ°ΠΊΠΎΠΉ Π±ΡƒΠ΄Π΅Ρ‚ заряд Π½Π° ΠΊΠ°ΠΆΠ΄ΠΎΠΌ кондСнсаторС? Π§Ρ‚ΠΎ Π±ΡƒΠ΄Π΅Ρ‚ элСктричСская энСргия ΠΊΠ°ΠΆΠ΄ΠΎΠ³ΠΎ?

    Рисунок 27.12. ΠΏΡ€ΠΎΠ±Π»Π΅ΠΌΠ° 27.39.

    ΠŸΡ€Π΅Π΄ΠΏΠΎΠ»ΠΎΠΆΠΈΠΌ, Ρ‡Ρ‚ΠΎ напряТСниС Π½Π° кондСнсаторС C 1 составляСт Π’ 1 , ΠΈ НапряТСниС Π½Π° кондСнсаторС (C 2 + C 3 ) составляСт Π’ 2 .Если заряд кондСнсатора C 1 Ρ€Π°Π²Π΅Π½ Q 1 , Ρ‚ΠΎΠ³Π΄Π° Заряд Π½Π° ΠΏΠ°Ρ€Π°Π»Π»Π΅Π»ΡŒΠ½ΠΎΠΌ кондСнсаторС Ρ‚Π°ΠΊΠΆΠ΅ Ρ€Π°Π²Π΅Π½ Q 1 . ΠŸΠΎΡ‚Π΅Π½Ρ†ΠΈΠ°Π» Ρ€Π°Π·Π½ΠΈΡ†Π° Π² этой систСмС Ρ€Π°Π²Π½Π°

    (27,62)

    Π’Π°ΠΊΠΈΠΌ ΠΎΠ±Ρ€Π°Π·ΠΎΠΌ, заряд кондСнсатора 1 опрСдСляСтся Ρ€Π°Π·Π½ΠΎΡΡ‚ΡŒΡŽ ΠΏΠΎΡ‚Π΅Π½Ρ†ΠΈΠ°Π»ΠΎΠ² [Π”Π΅Π»ΡŒΡ‚Π°] V

    (27,63)

    НапряТСниС V 23 Π½Π° кондСнсаторС (C 2 + C 3 ) связано с расходом Q 1

    (27.64)

    Заряд кондСнсатора Π‘ 2 Ρ€Π°Π²Π΅Π½

    (27,65)

    Заряд кондСнсатора Π‘ 3 Ρ€Π°Π²Π΅Π½

    (27,66)

    ЭлСктричСская ΠΏΠΎΡ‚Π΅Π½Ρ†ΠΈΠ°Π»ΡŒΠ½Π°Ρ энСргия, запасСнная Π² ΠΊΠ°ΠΆΠ΄ΠΎΠΌ кондСнсаторС, Ρ€Π°Π²Π½Π°

    . (27,67)

    Для Ρ‚Ρ€Π΅Ρ… кондСнсаторов Π² этой Π·Π°Π΄Π°Ρ‡Π΅ элСктричСская ΠΏΠΎΡ‚Π΅Π½Ρ†ΠΈΠ°Π»ΡŒΠ½Π°Ρ энСргия Ρ€Π°Π²Π½Π° Π½Π° Π½ΠΎΠΌΠ΅Ρ€

    (27.68)

    (27,69)

    (27,70)


    ΠžΡ‚ΠΏΡ€Π°Π²Π»ΡΠΉΡ‚Π΅ ΠΊΠΎΠΌΠΌΠ΅Π½Ρ‚Π°Ρ€ΠΈΠΈ, вопросы ΠΈ / ΠΈΠ»ΠΈ прСдлоТСния ΠΏΠΎ элСктронной ΠΏΠΎΡ‡Ρ‚Π΅ Π½Π° адрСс [email protected] ΠΈ / ΠΈΠ»ΠΈ посСтитС домашнюю страницу Ѐрэнка Π’ΠΎΠ»ΡŒΡ„ΡΠ°.

    ΠžΡΠ½ΠΎΠ²Ρ‹ Смкости | EC&M

    Π’ ΠΏΡ€ΠΎΡˆΠ»ΠΎΠΉ ΠΊΠΎΠ»ΠΎΠ½ΠΊΠ΅ объяснялось, Ρ‡Ρ‚ΠΎ ΠΏΠ΅Ρ€Π΅ΠΌΠ΅Π½Π½Ρ‹ΠΉ Ρ‚ΠΎΠΊ, ΠΏΡ€ΠΎΡ‚Π΅ΠΊΠ°ΡŽΡ‰ΠΈΠΉ Ρ‡Π΅Ρ€Π΅Π· ΠΈΠ½Π΄ΡƒΠΊΡ‚ΠΈΠ²Π½ΠΎΡΡ‚ΡŒ, достигаСт своСй ΠΏΠΈΠΊΠΎΠ²ΠΎΠΉ Π°ΠΌΠΏΠ»ΠΈΡ‚ΡƒΠ΄Ρ‹ послС напряТСния. Π—Π°ΠΏΠ°Π·Π΄Ρ‹Π²Π°ΡŽΡ‰ΠΈΠΉ Ρ‚ΠΎΠΊ Ρ…Π°Ρ€Π°ΠΊΡ‚Π΅Ρ€Π΅Π½ для ΠΈΠ½Π΄ΡƒΠΊΡ‚ΠΈΠ²Π½ΠΎΠΉ Ρ†Π΅ΠΏΠΈ. Однако Π² Смкостных цСпях Π΅ΡΡ‚ΡŒ Ρ‚ΠΎΠΊ, ΠΊΠΎΡ‚ΠΎΡ€Ρ‹ΠΉ ΠΎΠΏΠ΅Ρ€Π΅ΠΆΠ°Π΅Ρ‚ напряТСниС.


    Π˜ΡΡ‚ΠΎΡ‡Π½ΠΈΠΊΠΈ Смкости. Π”Π²Π° ΠΏΡ€ΠΎΠ²ΠΎΠ΄Π½ΠΈΠΊΠ°, Ρ€Π°Π·Π΄Π΅Π»Π΅Π½Π½Ρ‹Π΅ изоляционным ΠΌΠ°Ρ‚Π΅Ρ€ΠΈΠ°Π»ΠΎΠΌ ΠΈΠ»ΠΈ диэлСктриком, ΠΎΠ±Ρ€Π°Π·ΡƒΡŽΡ‚ кондСнсатор. ΠŸΡ€ΠΈΠ»ΠΎΠΆΠ΅Π½ΠΈΠ΅ элСктричСского заряда ΠΊ ΠΏΡ€ΠΎΠ²ΠΎΠ΄Π½ΠΈΠΊΠ°ΠΌ создаст элСктричСскоС ΠΏΠΎΠ»Π΅, пСрпСндикулярноС ΠΏΡ€ΠΎΠ²ΠΎΠ΄Π½ΠΈΠΊΠ°ΠΌ Ρ‡Π΅Ρ€Π΅Π· диэлСктрик. Π•ΠΌΠΊΠΎΡΡ‚ΡŒ измСряСтся Π² Ρ„Π°Ρ€Π°Π΄Π°Ρ… (Π€) ΠΈΠ»ΠΈ Π²Π΅Π»ΠΈΡ‡ΠΈΠ½Π΅ Π½Π°ΠΊΠΎΠΏΠ»Π΅Π½Π½ΠΎΠ³ΠΎ заряда Π½Π° Π²ΠΎΠ»ΡŒΡ‚.

    Π‘ΠΈΠ»ΠΎΠ²Ρ‹Π΅ кондСнсаторы состоят ΠΈΠ· Π±Π»ΠΎΠΊΠΎΠ² кондСнсаторов. ΠžΡ‚Π΄Π΅Π»ΡŒΠ½Ρ‹Π΅ ΠΏΠ°ΠΊΠ΅Ρ‚Ρ‹ ΡΠΊΠ»Π°Π΄Ρ‹Π²Π°ΡŽΡ‚ΡΡ вмСстС ΠΈ ΡΠΎΠ΅Π΄ΠΈΠ½ΡΡŽΡ‚ΡΡ ΠΏΠ΅Ρ€Π΅ΠΌΡ‹Ρ‡ΠΊΠ°ΠΌΠΈ, образуя ΠΏΠ°Ρ€Π°Π»Π»Π΅Π»ΡŒΠ½Ρ‹Π΅ ΠΈΠ»ΠΈ ΠΏΠΎΡΠ»Π΅Π΄ΠΎΠ²Π°Ρ‚Π΅Π»ΡŒΠ½Ρ‹Π΅ Π±Π»ΠΎΠΊΠΈ.ΠšΠ°ΠΆΠ΄Ρ‹ΠΉ ΠΏΠ°ΠΊΠ΅Ρ‚ состоит ΠΈΠ· мСталличСских пластин, часто ΠΈΠ· алюминиСвой Ρ„ΠΎΠ»ΡŒΠ³ΠΈ, Ρ€Π°Π·Π΄Π΅Π»Π΅Π½Π½Ρ‹Ρ… ΠΏΠΎΠ»ΠΈΠΏΡ€ΠΎΠΏΠΈΠ»Π΅Π½ΠΎΠ²Ρ‹ΠΌ диэлСктриком. Π‘ΡƒΠΌΠ°Π³Ρƒ ΠΈΠ½ΠΎΠ³Π΄Π° Ρ‚Π°ΠΊΠΆΠ΅ ΠΈΡΠΏΠΎΠ»ΡŒΠ·ΡƒΡŽΡ‚ Π² качСствС диэлСктрика. Π­Ρ‚ΠΈ сэндвичи ΠΈΠ· Ρ„ΠΎΠ»ΡŒΠ³ΠΈ ΠΈ ΠΏΠΎΠ»ΠΈΠΏΡ€ΠΎΠΏΠΈΠ»Π΅Π½Π° свСрнуты ΠΈ ΠΏΠΎΠΌΠ΅Ρ‰Π΅Π½Ρ‹ Π² Ρ€Π΅Π·Π΅Ρ€Π²ΡƒΠ°Ρ€, Π·Π°ΠΏΠΎΠ»Π½Π΅Π½Π½Ρ‹ΠΉ маслом. ΠŸΠΎΠ΄ΠΎΠ±Π½Ρ‹Π΅ кондСнсаторы ΠΎΠ±Ρ‹Ρ‡Π½ΠΎ ΠΈΡΠΏΠΎΠ»ΡŒΠ·ΡƒΡŽΡ‚ΡΡ для ΠΊΠΎΡ€Ρ€Π΅ΠΊΡ†ΠΈΠΈ коэффициСнта мощности ΠΈ Ρ„ΠΈΠ»ΡŒΡ‚Ρ€Π°Ρ†ΠΈΠΈ Π² систСмах ΠΎΡ‚ Π½ΠΈΠ·ΠΊΠΎΠ³ΠΎ напряТСния Π΄ΠΎ самых высоких напряТСний ΠΏΠ΅Ρ€Π΅Π΄Π°Ρ‡ΠΈ.

    Π‘ΠΈΠ»ΠΎΠ²Ρ‹Π΅ кондСнсаторы — Π½Π΅ СдинствСнныС устройства, Π΄Π΅ΠΌΠΎΠ½ΡΡ‚Ρ€ΠΈΡ€ΡƒΡŽΡ‰ΠΈΠ΅ СмкостныС свойства. Π˜Π·ΠΎΠ»ΠΈΡ€ΡƒΡŽΡ‰ΠΈΠ΅ Π²Π²ΠΎΠ΄Ρ‹ оборудования, ΠΈΠ·ΠΎΠ»ΠΈΡ€ΠΎΠ²Π°Π½Π½Ρ‹ΠΉ кабСль ΠΈ Π΄Π°ΠΆΠ΅ Π»ΠΈΠ½ΠΈΠΈ ΠΏΠ΅Ρ€Π΅Π΄Π°Ρ‡ΠΈ ΠΈ распрСдСлСния с Ρ€Π°Π·ΠΎΠΌΠΊΠ½ΡƒΡ‚Ρ‹ΠΌΠΈ ΠΏΡ€ΠΎΠ²ΠΎΠ΄Π°ΠΌΠΈ выглядят ΠΊΠ°ΠΊ кондСнсаторы, соСдинСнныС ΠΎΡ‚ Π»ΠΈΠ½ΠΈΠΈ ΠΊ Π·Π΅ΠΌΠ»Π΅.ЗначСния Смкости этих устройств, ΠΊΠ°ΠΊ ΠΏΡ€Π°Π²ΠΈΠ»ΠΎ, большС ΠΏΡ€ΠΈ Π±ΠΎΠ»Π΅Π΅ высоких Ρ€Π°Π±ΠΎΡ‡ΠΈΡ… напряТСниях.

    Как Ρ€Π°Π±ΠΎΡ‚Π°Π΅Ρ‚ кондСнсатор?

    ΠšΠΎΠ½Π΄Π΅Π½ΡΠ°Ρ‚ΠΎΡ€Ρ‹ Π½Π°ΠΊΠ°ΠΏΠ»ΠΈΠ²Π°ΡŽΡ‚ ΡΠ½Π΅Ρ€Π³ΠΈΡŽ Π² элСктричСском ΠΏΠΎΠ»Π΅, установлСнном Π² ΠΈΡ… диэлСктриках, ΠΏΠΎΠ΄ΠΎΠ±Π½ΠΎ Ρ‚ΠΎΠΌΡƒ, ΠΊΠ°ΠΊ ΠΈΠ½Π΄ΡƒΠΊΡ‚ΠΎΡ€Ρ‹ Π½Π°ΠΊΠ°ΠΏΠ»ΠΈΠ²Π°ΡŽΡ‚ ΡΠ½Π΅Ρ€Π³ΠΈΡŽ Π² своих ΠΌΠ°Π³Π½ΠΈΡ‚Π½Ρ‹Ρ… полях. ЭлСктричСскоС ΠΏΠΎΠ»Π΅ создаСтся элСктричСским зарядом Π½Π° пластинах кондСнсатора. Π’ Ρ„ΠΈΠ»ΡŒΡ‚Ρ€Π°Ρ… ΠΈΡΠΏΠΎΠ»ΡŒΠ·ΡƒΡŽΡ‚ΡΡ кондСнсаторы, Ρ‡Ρ‚ΠΎΠ±Ρ‹ Β«Π·Π°ΠΏΠΎΠ»Π½ΠΈΡ‚ΡŒΒ» ΠΏΡ€ΠΎΠΌΠ΅ΠΆΡƒΡ‚ΠΊΠΈ Π² Ρ„ΠΎΡ€ΠΌΠ΅ Π²ΠΎΠ»Π½Ρ‹, отдавая Π½Π°ΠΊΠΎΠΏΠ»Π΅Π½Π½Ρ‹ΠΉ заряд элСктричСской Ρ†Π΅ΠΏΠΈ, ΠΈΠ»ΠΈ Β«ΡΠ±Ρ€ΠΈΡ‚ΡŒΒ» всплСски Π² Ρ„ΠΎΡ€ΠΌΠ΅ Π²ΠΎΠ»Π½Ρ‹, поглощая ΠΈΠ·Π±Ρ‹Ρ‚ΠΎΡ‡Π½ΡƒΡŽ ΡΠ½Π΅Ρ€Π³ΠΈΡŽ ΠΈ сохраняя Π΅Π΅ Π² элСктричСском ΠΏΠΎΠ»Π΅.

    ΠšΠΎΠ½Π΄Π΅Π½ΡΠ°Ρ‚ΠΎΡ€Ρ‹, ΠΏΠΎΠ΄ΠΊΠ»ΡŽΡ‡Π΅Π½Π½Ρ‹Π΅ ΠΎΡ‚ Π»ΠΈΠ½ΠΈΠΈ ΠΊ Π½Π΅ΠΉΡ‚Ρ€Π°Π»ΠΈ, ΠΎΠ±Π΅ΡΠΏΠ΅Ρ‡ΠΈΠ²Π°ΡŽΡ‚ Ρ€Π΅Π°ΠΊΡ‚ΠΈΠ²Π½ΡƒΡŽ ΠΌΠΎΡ‰Π½ΠΎΡΡ‚ΡŒ Π² Ρ†Π΅ΠΏΠΈ ΠΏΠ΅Ρ€Π΅ΠΌΠ΅Π½Π½ΠΎΠ³ΠΎ Ρ‚ΠΎΠΊΠ°, которая измСряСтся Π² Ρ€Π΅Π°ΠΊΡ‚ΠΈΠ²Π½Ρ‹Ρ… Π²ΠΎΠ»ΡŒΡ‚-Π°ΠΌΠΏΠ΅Ρ€Π°Ρ… (ВАР). Для Π΄Π°Π½Π½ΠΎΠ³ΠΎ значСния Смкости ΠΏΡ€ΠΈ ΡƒΠ²Π΅Π»ΠΈΡ‡Π΅Π½ΠΈΠΈ Ρ€Π°Π±ΠΎΡ‡Π΅Π³ΠΎ напряТСния подаСтся большС VAR. ΠŸΠΎΡΠΊΠΎΠ»ΡŒΠΊΡƒ ΠΎΠ½ΠΈ ΡΠ²Π»ΡΡŽΡ‚ΡΡ источником Ρ€Π΅Π°ΠΊΡ‚ΠΈΠ²Π½ΠΎΠΉ мощности, кондСнсаторы ΠΌΠΎΠ³ΡƒΡ‚ ΠΎΠ±Π΅ΡΠΏΠ΅Ρ‡ΠΈΠ²Π°Ρ‚ΡŒ ΠΏΠ΅Ρ€Π΅ΠΌΠ΅Π½Π½Ρ‹Π΅ мощности, потрСбляСмыС ΠΈΠ½Π΄ΡƒΠΊΡ‚ΠΈΠ²Π½Ρ‹ΠΌΠΈ Π½Π°Π³Ρ€ΡƒΠ·ΠΊΠ°ΠΌΠΈ. ΠŸΠΎΡΠΊΠΎΠ»ΡŒΠΊΡƒ ΠΏΠ΅Ρ€Π΅ΠΌΠ΅Π½Π½Ρ‹Π΅ мощности, обСспСчиваСмыС кондСнсатором, Π½Π΅ Π΄ΠΎΠ»ΠΆΠ½Ρ‹ ΠΏΠΎΠ΄Π°Π²Π°Ρ‚ΡŒΡΡ ΠΈΠ· систСмы, коэффициСнт мощности систСмы увСличиваСтся ΠΈΠ·-Π·Π° ΠΏΠΎΠ΄ΠΊΠ»ΡŽΡ‡Π΅Π½Π½ΠΎΠ³ΠΎ кондСнсатора.Π­Ρ‚ΠΎ называСтся ΠΊΠΎΡ€Ρ€Π΅ΠΊΡ†ΠΈΠ΅ΠΉ коэффициСнта мощности ΠΈ Π±ΡƒΠ΄Π΅Ρ‚ рассмотрСно Π² столбцС ΡΠ»Π΅Π΄ΡƒΡŽΡ‰Π΅Π³ΠΎ мСсяца.

    РСзонанс.

    ВсС ΠΊΠΎΠΌΠΏΠΎΠ½Π΅Π½Ρ‚Ρ‹ систСмы питания ΠΈΠΌΠ΅ΡŽΡ‚ элСктричСскоС сопротивлСниС ΠΈ Π±ΠΎΠ»ΡŒΡˆΠΈΠ½ΡΡ‚Π²ΠΎ ΠΈΠ· Π½ΠΈΡ… ΠΎΠ±Π»Π°Π΄Π°ΡŽΡ‚ ΠΈΠ½Π΄ΡƒΠΊΡ‚ΠΈΠ²Π½Ρ‹ΠΌΠΈ характСристиками. Π”ΠΎΠ±Π°Π²Π»Π΅Π½ΠΈΠ΅ кондСнсаторов Π² систСму питания вносит Π²ΠΎΠ·ΠΌΠΎΠΆΠ½ΠΎΡΡ‚ΡŒ рСзонанса. Если Ρ€Π΅Π°ΠΊΡ‚ΠΈΠ²Π½ΠΎΠ΅ сопротивлСниС ΠΈΠ½Π΄ΡƒΠΊΡ‚ΠΈΠ²Π½ΠΎΠΉ ΠΈ Смкостной частСй Ρ†Π΅ΠΏΠΈ Ρ€Π°Π²Π½ΠΎ, энСргия Π±ΡƒΠ΄Π΅Ρ‚ ΠΊΠΎΠ»Π΅Π±Π°Ρ‚ΡŒΡΡ ΠΌΠ΅ΠΆΠ΄Ρƒ ΠΈΠ½Π΄ΡƒΠΊΡ‚ΠΈΠ²Π½ΠΎΡΡ‚ΡŒΡŽ ΠΈ Π΅ΠΌΠΊΠΎΡΡ‚ΡŒΡŽ, создавая высокиС напряТСния, ΠΊΠΎΡ‚ΠΎΡ€Ρ‹Π΅ ΠΌΠΎΠ³ΡƒΡ‚ ΠΏΡ€Π΅Π²Ρ‹ΡˆΠ°Ρ‚ΡŒ возмоТности изоляции систСмы.

    Π Π΅Π°ΠΊΡ‚ΠΈΠ²Π½ΠΎΠ΅ сопротивлСниС ΠΊΠ°Ρ‚ΡƒΡˆΠΊΠΈ индуктивности ΠΈ кондСнсатора зависит ΠΎΡ‚ частоты. ЦСпь, содСрТащая сопротивлСниС, ΠΈΠ½Π΄ΡƒΠΊΡ‚ΠΈΠ²Π½ΠΎΡΡ‚ΡŒ ΠΈ Π΅ΠΌΠΊΠΎΡΡ‚ΡŒ, Ρ€Π΅Π·ΠΎΠ½ΠΈΡ€ΡƒΠ΅Ρ‚ Π½Π° ΠΎΠ΄Π½ΠΎΠΉ ΠΎΠΏΡ€Π΅Π΄Π΅Π»Π΅Π½Π½ΠΎΠΉ частотС. Если эта частота Π±Π»ΠΈΠ·ΠΊΠ° ΠΊ основной частотС (60 Π“Ρ† Π² БША) ΠΈΠ»ΠΈ Ρ†Π΅Π»ΠΎΠΌΡƒ ΠΊΡ€Π°Ρ‚Π½ΠΎΠΌΡƒ этой Ρ€Π°Π±ΠΎΡ‡Π΅ΠΉ частоты, Π²ΠΎΠ·Π½ΠΈΠΊΠ½ΡƒΡ‚ ΡΠ΅Ρ€ΡŒΠ΅Π·Π½Ρ‹Π΅ ΠΏΡ€ΠΎΠ±Π»Π΅ΠΌΡ‹.

    alexxlab

    Π”ΠΎΠ±Π°Π²ΠΈΡ‚ΡŒ ΠΊΠΎΠΌΠΌΠ΅Π½Ρ‚Π°Ρ€ΠΈΠΉ

    Π’Π°Ρˆ адрСс email Π½Π΅ Π±ΡƒΠ΄Π΅Ρ‚ ΠΎΠΏΡƒΠ±Π»ΠΈΠΊΠΎΠ²Π°Π½. ΠžΠ±ΡΠ·Π°Ρ‚Π΅Π»ΡŒΠ½Ρ‹Π΅ поля ΠΏΠΎΠΌΠ΅Ρ‡Π΅Π½Ρ‹ *